Vous êtes sur la page 1sur 109

Sussex Mathematics Lecture Notes

Year 2009
Monetary Theory Analysis (G5078MTA)
taught by Omar Lakkis

http://www.maths.sussex.ac.uk/Staff/OL/teaching.html
o.lakkis@sussex.ac.uk

Contents
Whats this?
Disclaimer
Course synopsis

5
5
5

Chapter 1. Theory of interest rate


1.1. Discrete interest rate
1.2. From discrete time to continuous time
1.3. Present value
1.4. Valuing cash flows, equation of value and yield
1.5. Interest income
1.6. Compound Interest Functions
1.7. Annuities-certain: present values and accumulation
1.8. The general loan or mortgage schedule
1.9. Cash flow
Exercises and problems on Theory of Interest Rate

1
1
2
6
8
10
11
12
14
15
16

Chapter 2. Nominal rate of interest and fractionally convertible annuities


2.1. Interest payable fractionally
2.2. Annuity payable fractionally
2.3. Annuities payable at time r > 1
2.4. The loan schedule for a fractional annuity
Exercises and problems on Rates convertible p-thly

19
19
22
25
26
26

Chapter 3. Applications of Interest Rate Theory


3.1. The valuation of some securities
3.2. Price and yield relation
3.3. Perpetuity
3.4. Makehams formula
3.5. The effect of the term to redemption on the yield
3.6. Real returns, inflation and index-linked stocks
Exercises and problems on Application of interest rate

29
29
31
34
35
38
40
42

Chapter 4. Simple market model


4.1. Building a simple market model
4.2. Basic assumptions
4.3. No-Arbitrage Principle (NAP)
4.4. One-step binomial model
Exercises and problems on Simple market model

45
45
47
48
49
51

Chapter 5. Arbitrage and pricing


5.1. Terminology and notation
5.2. Redundant Assets
5.3. Contingent Claims and Derivative Assets
5.4. Insurable portfolios and states
5.5. Dominance and arbitrage

53
53
55
56
56
58

MTA (G5078) Autumn 2009 Complete course


5.6. Pricing under NAP
5.7. Riskless Issues
5.8. Applications
Exercises and problems on Arbitrage and portfolios

60
62
64
67

Appendix A. A review of optimisation


1.1. Homotheticity and homogeneity
1.2. General optimisation (also known as nonlinear programming)
1.3. Linear programming (LP) and duality

69
69
69
72

Appendix B. Brush-up of Probability Theory


2.1. Probability space and measure
2.2. Random variables
2.3. Distribution and density
2.4. Moments and characteristic functions
2.5. Normal distribution
2.6. Limit Theorems
2.7. Central limit-theorem
2.8. Examples of random variables
2.9. Stochastic Processes
2.10. Regression and analysis of variance

77
77
78
80
86
89
89
90
90
92
93

Appendix C. Common indices in Economics and the Housing Market


3.1. FTSE 100 Index (also known as Footsie)
3.2. Halifax House Price Index
3.3. Nationwide House price Index
3.4. Calculating the price of a typical house
3.5. Pooled Property Fund Indices

95
95
95
97
99
100

Appendix. Bibliography

101

Appendix. Index

103

MTA (G5078) Autumn 2009 0. Contents

Whats this?
This booklet bears the essential contents of the course Monetary Theory Analysis
(MTA), code G5078, as taught at the University of Sussex by Omar Lakkis in 2009.
It contains the (most of) the material covered in class as well as the exercises and
their solutions. If you are a University of Sussex maths student, or prospective student, you are allowed to use this document only after you have taken the Monetary
Theory Analysis course with me. Otherwise you must have explicit permission from
the tutor o.lakkis@sussex.ac.uk to use it.
Disclaimer
A part from a couple of places, these notes are largely the result of summarising
the reading from published sources [CZ03, MCS86, mainly]. I therefore have no
pretense at originality, except for the errors that you may find in the manuscript. If
you do find what you think is an error/typo/mistake, I will be grateful if you sent
me an email about it (subject: MTA G5078 notes typo) so that things can be
fixed for future releases of these notes.
Much of the material in the notes is copyrighted by the authors of the aforementioned
sources. If you publish or broadcast this material, or otherwise use it for purposes
not directly related to this particular course you may be acting illegaly at your own
risk.
Course synopsis
This course focuses on the basic mathematical aspects of money dynamics.
Interest rate theory and manipulations are at the base of the course, there we learn
about mathematical (and where necessary computational) methods in finance such
as cash flows, the equation of value and the methods for calculating mortgages.
We then introduce various applications of interest theory by looking at examples
where surrounding interest rates and inflation may influence investments. We
learn how to calculate return and introduce the concept of yield, for fixed and
variable income securities.
A learning outcome of the course is to know how to evaluate, analyse and disseminate a real world investment project by writing an easy to understand financial
report, which implies that the assessment comes in the form of an Essay covering
a particular aspect of the course.
For the Essay, you will need to be:
? comfortable mathematics methods to solve financial problems;
? preparted to do autonomous reading in the field;
? able to communicate mathematical ideas to an audience.

CHAPTER 1

Theory of interest rate


1.1. Discrete interest rate
1.1.1. Simple interest. If an amount C is deposited in an account which pays
simple interest at the rate of i 100% per annum and the account is closed after
n years - there being no intervening payments to or from the account - then the
amount paid to the investor when the account is closed will be C(1 + ni)
This payment consists of a return of the initial deposit C, together with interest of
amount niC.
1.1.2. Exercise. An investor puts 1, 000 in a savings account that pays 10%
simple interest at the end of each year. Compare how much the investor would have
after 2 years if the money was:
(i) invested continuously for 2 years
(ii) invested for 1 year, then immediately reinvested for a further year.
1.1.3. Compound interest. The essential feature of compound interest is that
interest itself earns interest. Assuming that the annual interest is i 100% per
annum and the account is closed after n years, then the amount paid to the investor
when the account is closed will be C(1 + i)n .
This payment consists of a return of the initial deposit, C, together with accumulated
interest (i.e., interest which, if n > 1, has itself earned further interest) of amount:
C((1 + i)n 1) .
The compound interest rate in this case is i.
This can be extended to variable interest rate case: let the interest rate for year k
to be i(k), k = 0, 1, 2, . . . n 1. If an investment has been held for n years, then the
accumulation of investment C from year 0 to year n is
C(1 + i(0))(1 + i(1)) . . . (1 + i(n 1)).
In general, let an investment of 1 for a time period of 1 time unit commencing at
time t is returned 1 + i(t) at time t + 1, we call i(t) the rate of interest for the period
t to t + 1. i(t) is also referred to as the effective rate of interest for the period.
1.1.4. Definition (nominal and effective rates of interest). Consider transactions
for a term of length h times units where h > 0 is a real number. Although not
necessary, it is useful to think h < 1.
Define ih (t) as the nominal rate of interest per unit time on transaction of term h
beginning at time t. The effective rate of interest over this period is thus defined as
hih (t). Thus if the amount C is invested at time t for a term of length h, the sum
to be received at time t + h is
C(1 + hih (t)).
If h = 1, then i1 (t) = i(t). When ih (t) does not depend on t we write it simply as
ih and we speak of a constant interest rate.
The typical choice for h is 1/p with p a positive integer (e.g., 12 for the number of
months in a year, or 4 for the number of quarters/trimesters in a year). If h = 1/p

MTA (G5078) Autumn 2009 Lecture Notes

and p integer, we write ih = i(p) , which also means i1/p = i(p) . According to this
definition, an investment of 1 for a period of length 1/p produces a return of 1+i(p) /p.
1.1.5. Example (nominal rates).
Problem. Working in time units of years, we will let the nominal rate of interest
per year on monthly transactions over the next year be 12. If 100 is invested at
time 0, how much is it worth at the end of the first month and at the end of the
third month?
Solution. We have defined: i1/12 = 12% and so the effective monthly interest rate
is i1/12 /12 = 1%.
At the end of the first month we have 100(1 + 0.01) = 101.
At the end of the third month we have 100 1.013 = 103.03
1.1.6. Example.
Problem. The nominal interest rate of interest per annum quoted in the financial
press for local authority deposits on a particular day are as follows:
Term
Nominal rate of interest (%)
1 day
11.75
1 week
11.5
1 month
11.375
1 quarter
11.25
Find the accumulation of an investment at this time of 100 for
(a) for 2 days,
(b) for 3 weeks,
(c) for 1 month,

(d) for 7 days, and compare with 1 week,


(e) for 90 days, and compare with 1
quarter.
Draw conclusions.

Solution. To express the above information in terms of our notation, we draw up


the following table, our time unit is one year and the particular time is t0 :
Term h 1/365 1/52
1/12
1/4
ih (t0 ) 0.117 5 0.115 0.113 75 0.112 5
The accumulations are:
(a) for 2 days, 100(1 + 0.117 5/365)2 = 100.064 394,
(b) for 3 weeks, 100(1 + 0.115/52)3 = 100.663 104,
(c) for 1 month, 100(1 + 0.113 75/12) = 100.947 917
(d) for seven days, 100(1 + 0.117 5/365)7 = 100.225 560,
and for a week, 100(1 + 0.115/52) = 100.221 154.
(e) for 90 days, 102.939 159 and, for a quarter, 102.812 500.
It is apparent that the rates have been rounded up a bit too much. An enterprising
bank clerk could take advantage of this situation to perform some salami slicing:
she invests the money for 90 days for the customer wanting it for a quarter. She
then shaves off the a salami slice of 102.939 159 102.812 500 = 0.13, on each
such customer.
1.2. From discrete time to continuous time
A continuous time model is useful as it allows us to use the many rules of calculus
that are available. In this section we relate our discrete-time vision, described above,
to one where time is continuous. In mathematical words, we want to see how to
express interest phenomena when h 0.

MTA (G5078) Autumn 2009 1.2. From discrete time to continuous time3
1.2.1. Accumulation factors. Let time be measured in suitable units (e.g. years).
For t1 t2 we define A(t1 , t2 ) to be the accumulation at time t2 of an investment of
1 at time t1 for a term of t2 t1. Thus A(t1 , t2 ) is the amount which will be repaid
at time t2 in return for an investment of 1 at time t1 .
Expressing this in terms of capital, suppose our capital at time t1 was C(t1 ) and at
time t2 , C(t2 ), then the accumulation factor is given by
A(t1 , t2 ) =

C(t2 )
.
C(t1 )

(1.2.1)

Since hih (t) is the effective rate for the period of length h beginning at time t and
so, by the definition of ih (t), for all t and for all h > 0,
A(t, t + h) = 1 + hih (t)

(1.2.2)

ih (t) = (A(t, t + h) 1)/h for h > 0

(1.2.3)

and hence that


We also define A(t, t) = 1 for all t. The number A(t1 , t2 ) is often called the accumulation factor, since the accumulation at time t2 of an investment of C at time t1 is,
by proportion,
CA(t1 , t2 ).
1.2.2. Theorem (Principle of Consistency). If t1 t2 . . . tn, then
A(t1 , tn ) = A(t1 , t2 )A(t2 , t3 ) . . . A(tn1 , tn ).

(1.2.4)

Proof The proof is easily achieved by induction on n and observing that


C(tn ) = C(tn1 )A(tn1 , tn ).
The details are left to the student as an exercise.

(1.2.5)


1.2.3. Example. For integer times, 50 are invested at time 2 and the accumulated
amount at time 7 is 100. Find i5 (2) and i(2).
Solution. The accumulation factor is A(2, 7) = 100/50 = 2 and, using (1.2.3), we
have i5 (2) = (2 1)/5 = 0.2 = 20%.
1.2.4. Exercise. Calculate the equivalent effective annual rate of interest (assumed
to be a constant over the period) for the investment in the above example, assuming
that time is in years.
1.2.5. Exercise. 5000 is invested at time 0 and the proceeds at time 10 are 9000.
Calculate A(6, 10) if A(0, 9) = 1.8, A(2, 4) = 1.1, A(2, 6) = 1.32, A(4, 9) = 1.45.
1.2.6. Force of interests. We consider the case where the interest is paid continuously throughout the time period: a nominal interest rate convertible very frequently
(e.g., every second), then this fund steadily accumulates over the period as interest
is earned and added.
In the limiting case, i.e., as h 0, the amount of the fund can be considered to be
subject to a constant force causing it to grow. This leads us to the concept of a
force of interest per unit time at time t: (t). It is mathematically defined as
lim ih (t) = lim h 0+ (A(t, t + h) 1)/h = (t).

h0+

(t) is also know as the nominal rate of interest per unit time at time t convertible
momently.

MTA (G5078) Autumn 2009 Lecture Notes

1.2.7. Continuous accumulation. The relationship of the force of interest to


accumulation is intimately related to the properties of the exponential.
Consider the situation where money accumulates at the force of interest on an
interval [t, s]. Subdivide this interval into n equal-length intervals [tk1 , tk ], for
k = 1, . . . , n, where
tk = t + kh, for k = 0, . . . , n.
(1.2.6)
By the Principle of Consistency (1.2.4), we have
A(t, s) = A(t0 , t1 )A(t1 , t2 ) A(tn1 , tn ).

(1.2.7)

Recalling the definition of ih this implies


A(t, s) = (1 + hih (t0 ))(1 + hih (t1 )) (1 + hih (tn1 )) ,

(1.2.8)

which is a long product. A very useful property of exp (and of its inverse log) is
that it transforms sums into products (and viceversa). Using these properties we
may write the above as
A(t, s) = exp(log(1 + hih (t0 )) + log(1 + hih (t1 )) + + log(1 + hih (tn1 ))) .
(1.2.9)
Now, recalling Taylors formula of order 2 we know that
log(1 + ) = + (), for 1/2 < < 1/2,

(1.2.10)

where the remainder function satisfies


|()| c0 2

(1/2, 1/2) .

(1.2.11)

Supposing then that there is a uniform bound on ih , i.e.,


ih ( ) i

, h > 0,

(1.2.12)

it follows that, for h < 1/2i , we can apply the Taylor formula and regroup terms
to get



A(t, s) = exp hih (t0 ) + + hih (tn1 ) + (hih (t0 )) + + (hih (tn1 ))
= exp

n1
X

hih (tk ) + (h),

(1.2.13)

k=0

where
(h) := (hih (t0 )) + + (hih (tn1 )).
(1.2.14)
Note that identity (1.2.13) is true for all h > 0 and h < 1/2i . Taking h 0 (and
using uniform convergence of step functions) we have
" n1
# Z
s
X
lim
hih (tk ) =
( ) d.
(1.2.15)
h0

k=0

To see what happens to the second term on the right-hand side of (1.2.13), we use
the properties of the remainders in Taylors formula (1.2.11) and the uniform bound
(1.2.12), we get
0 |(h)|

n1
X
k=0

|(hih (tk ))| c0 hi

n1
X

h = c0 i 2 (s t)h 0, as h 0.

k=0

(1.2.16)
Thus (h) 0 and, modulo minor technical points, we have shown that
Z s
A(t, s) = exp
( ) d.
t

(1.2.17)

MTA (G5078) Autumn 2009 1.2. From discrete time to continuous time5
1.2.8. Theorem (accumulation factor and force of interest relation). Given an
accumulation factor (t, s) 7 A(t, s), then

Z s
( ) d ,
(1.2.18)
A(t, s) = exp
t

for all t s.
Proof 1
Fix t R and let s be variable. Define
Z

( ) d.

R(s) := log(A(t, s)) and L(s) :=

(1.2.19)

We want to show that R(s) = L(s) for all s t. For starters we have
R(t) = 0 = L(t)

(t is frozen) .

(1.2.20)

Then for s t, we have (by the Chain Rule and the definition of A and ) that
d
1
d
R(s) =
A(t, s)
ds
A(t, s) ds
1
A(t, s + h) A(t, s)
=
lim
h0
A(t, s)
h
1
A(t, s)(A(s, s + h) 1)
=
lim
A(t, s) h0
h
A(s, s + h) 1
= lim
h0
h
= (s).

R0 (s) =

(1.2.21)

On the other hand, by the Fundamental Theorem of Calculus, we have that


L0 (s) = (s).

(1.2.22)

Thus R0 (s) = L0 (s), for all s t and L(t) = R(t), so by the integrating both sides
over [t, s] we obtain
Z s
Z s
0
L(s) = L(t) +
L = R(t) +
R0 = R(s), s t.
(1.2.23)
t


1.2.9. Example. Assume (a) (t) = , or (b) (t) = a + bt.
Find the formulas for the accumulation of 1 unit from t1 to t2 in each case.
1.2.10. Solution. Using (1.2.18) we have
(a) A(t1 , t2 ) = exp((t2 t1 )), and
(b) A(t1 , t2 ) = exp(a(t2 t1 ) + 21 b(t22 t21 )).
1.2.11. Exercise. A bank credits interest on deposit using accumulation factors
based on a variable force of interest. On 1 July 1999, a customer deposited 100, 000
pounds with the bank. On 1 July 2001, his deposit has grown to 120, 000. Assuming that the force of interest per annum was bt (1 July 1999 = 0, unit = year)
during the period, find the force of interest per annum on 1 July 2000.
1Technical

note. This proof, as presented, is a more rigorous, but perhaps less insightful,
alternative to the one in 1.2.7. Also this proof will work only for differentiable accumulation
functions A. The argument in 1.2.7 can be made perfectly rigorous for that is Lebesgueintegrable, so it may even have jumps. See [MCS86, Appendix 1] for another proof.

MTA (G5078) Autumn 2009 Lecture Notes

1.2.12. Solution. Using Theorem 1.2.8 we have




1 2
2
A(t1 , t2 ) = exp b(t2 t1 ) = 120 000/100 000 = 1.2
2
Since we have t1 = 0 for year 1999, and thus t2 = 2 for year 2001, we get
1
exp(2b) = 1.2, hence b = ln(1.2).
2
The force of interest per annum at t = 1 (year 2000) is ln(1.2)/2.

(1.2.24)

(1.2.25)

1.2.13. Exercise. If a large pension fund with a value of 2000m is assumed to


grow steadily subject to a constant force of interest of 10% per annum, how much
interest is earned every second? (Assume that there are 365 days in a year.)
1.2.14. Exercise. A bank credits interest on deposit using accumulation factors
based on a variable force of interest. On 1 July 1999, a customer deposited 100, 000
pounds with the bank. On 1 July 2001, his deposit has grown to 120, 000. Assuming that the force of interest per annum was a + bt (1 July 1999 = 0, unit = year)
during the period, find the force of interest per annum on 1 July 2000 and the force
of interest per annum on 1 Jan 2001.
1.3. Present value
1.3.1. Example (discounted present value). Due to interest rate effect cash flows
have different real meanings at different times. A payment of 100 pounds to us now
means 100 pounds now. Does it have the same value if the 100 pounds is promised
to us but in fact paid one year later?
Let us assume that an interest rate of 7% per annum is available to us. If someone
promises to pay us 100 in a year that includes 7% of some initial quantity X,
which they can invest, at the beginning of the year.
X(1.07) = 100 and thus X = 100/1.07 = 93.46.
That is, that 100 to be paid in a year means only about 93.50 now.
We say that 93.50 is the discounted present value of 100 in a year, at a 7%
interest rate.
1.3.2. Definition (discounted present value and past
Let t1 < t2 ,
 accumulation).

R t2
it follows that an investment of C/A(t1 , t2 ) = C exp t1 (t) dt at time t1 will
produce a return of C at time t2 .
We say that the discounted value at time t1 of value C due at time t2 is
 Z t2

C/A(t1 , t2 ) = C exp
(t) dt
t1

When t1 = 0 (the present time) and t2 = t 0, the discounted value of C due at t,


is called its discounted present value (or more briefly, its present value). The present
value of C is thus equal to
 Z t2

C exp
(t)dt
(1.3.1)
t1

Normalize (by getting rid of C) and define the function of time


 Z t

v(t) := exp
(t)dt .
0

? When t 0, v(t) is the discounted present value of 1 due at time t.

(1.3.2)

MTA (G5078) Autumn 2009 1.3. Present value

? When t < 0, v(t) is the accumulation of 1 from (negative) time t to time 0.


Hence the discounted present value of C due at a (non-negative) time t 0 is Cv(t).
Under a constant force of interest assumption, i.e.,
(t) = , t R,

(1.3.3)

for some fixed value R, we have


v(t) = v t , t R

(1.3.4)

v = v(1) = exp().

(1.3.5)

where
1.3.3. Example.
Problem. Measuring time in years from present, suppose that (t) = 0.06(0.9)t for
all t 0. Find a simple expression for v(t) and find the discounted present value of
100 due in 3.5 years time.
Solution.

 Z t
0.06(0.9t 1)
s
0.06(0.9) ds = exp
v(t) = exp
.
ln(0.9)
0
Hence the present value of 100 due in 3.5 years is
100 exp[0.06(0.93.5 1)/ ln(0.9)] = 83.89.

(1.3.6)

1.3.4. Exercise. Calculate the present value on 1 September 2002 of payments of


280 due on 1 September 2004 and 360 due on 1 March 2005. Interest is 10% per
annum effective.
1.3.5. Exercise. Write a 1000-2000 words essay on the activity of house purchasing
with a mortgage and explain the value of a house.
1.3.6. Present Values of Discrete Cash Flows. The present values of the
amounts c1 , . . . , cn , due at times 0 < t1 < . . . < tn , respectively, is given by the
sum of each single present value, relative to its time and amount,
c1 v(t1 ) + + cn v(tn ).
This is called the summation principle.
1.3.7. Present Values of Continuously Payable Cash Flows. Let M (t) denote the total payment made between time 0 and time t, then the rate of payment
at time t is,
(t) = M 0 (t),
(1.3.7)
i.e., the derivative of M at time t.
During the infinitesimal time period (t, t + dt), the amount of repayment is
(t)dt.

(1.3.8)

Its infinitesimal discounted value at time t is then


v(t)(t) dt.

(1.3.9)

Following the summation principle (and taking the limit, were we to make a fully
rigorous argument) we obtain the following expression for the discounted value between 0 and t given by
Z t
v( )( ) d.
(1.3.10)
0

MTA (G5078) Autumn 2009 Lecture Notes

1.3.8. Present Values of Discrete and Continuously Payable Cash Flows.


When we have both types of payments: discrete ones coming at bursts in time
and continuous one, which flow with a certain rate then we use the following
continuous-discrete description of present value:
Z t
v( ) ( ) d.
c1 v(t1 ) + + cn v(tn ) + +
0

1.4. Valuing cash flows, equation of value and yield


1.4.1. Discrete time description. Consider the following transaction, in return
for outlays of amount a1 , a2 , . . . , an at times t1 < t2 < < tn , an investor will
receive payments of b1 , b2 , . . . , bn at these times respectively. (In most situations
only one of aj and bj will be non-zero for each j = 1, . . . , n.) At what force or rate
of interest does the series of outlays have the same value as the series of receipts?
At force of interest the two series are of equal value if and only if:

 X
 Z tr
 Z tr
n
n
X
( ) d .
(1.4.1)
( ) d =
br exp
ar exp
r=1

r=1

Note that in spite of using a discrete description for the process we are using a
continuous time description for the interest rate, i.e., in terms of the force of interest
function .
This equation may be written as:
 Z tr 
n
X
cr exp
=0
(1.4.2)
0

r=1

where cr = br ar is the amount of the net cash flow at time tr . (We adopt
the convention that a negative cash flow corresponds to a payment by the investor
and a positive
 R cash
 flow represents a payment to the investor.) Recalling that
t
v(t) = exp 0 we may rewrite (1.4.2) as
n
X

cr v(tr ) = 0.

(1.4.3)

r=1

Equation (1.4.2), which expresses algebraically the condition that, at force of interest
, the total value of the net cash flows is 0, is called the equation of value for the
force of interest implied by the transaction.
Under the constant force of interest assumption we have
1
e = 1 + i = and v(t) = v t ,
(1.4.4)
v
and the equation of value (1.4.3) may be then written as
!
n
n
X
X
tr
tr
cr (1 + i)
or
cr v
= 0.
(1.4.5)
r=1

r=1

1.4.2. Continuous time description. In relation to continuous payment streams,


if we let 1 (t) and 2 (t) be the rates of paying and receiving money at time t respectively, we call (t) = 2 (t) 1 (t) the net rate of cash flow at time t. The equation
of value for the force of interest is:
Z
(t) exp(t) dt = 0.
0

MTA (G5078) Autumn 2009 1.5. Valuing cash flows, equation of value
and yield
9
1.4.3. Hybrid description. If the cash flows involve both discrete and continuous
transactions, we simply add the two contibutions.
1.4.4. Example.
Problem. A businessman is owed the following amounts: 1000 on 1 January 1986,
2500 on 1 January 1987 and 3000 on 1 July 1987. Assume a constant force of
interest of 0.06 per annum, find the value of these payments:
(a) on 1 January 1984,
(b) on 1 March 1985.
Solution. Let time be measured in years from 1 January 1984. The value of the
debts at that date is
1000v(2) + 2500v(3) + 3000v(3.5)
= 1000 exp(0.12) + 2500 exp(0.18) + 3000 exp(0.21) = 5406.85. (1.4.6)
The value at 1 March 1985 of the same debts is
5406.85 exp(0.06(14/12)) = 5798.89.

(1.4.7)

1.4.5. Example.
Problem. Suppose that time is measured in years and that , the force of interest
per unit time, is given by:
0.121 890
(t) = 0.076 961 +
(1.4.8)
1 + 0.5 exp(0.121 890 t)
(a) Find the single payment which, if invested at time 10, will accumulate to
30 000 at time 20.
(b) Find the accumulated amount after ten years of ten annual payments each of
1 000, the first payment being made at time 0.
Solution. (a) This is simply
30, 000 v(20) = xv(10)
Hence

30, 000 v(20)


.
v(10)
Now
 Z 20

2
1
(t) dt = (1.22)20 + (0.08)20 ,
v(20) = exp
3
3
0
2
1
v(10) = (1.22)10 + (0.08)10
3
3
and thus x = 10, 259.
(b) We have
1000(v(0) + v(1) + . . . + v(9)) = xv(10)
Hence x = 22, 822.
x=

1.4.6. Exercise. An investor owns a block of shares which are expected to pay a
dividend of amount D in one years time and dividends in each future year that are
100j % higher than in the previous year. Suppose that the bank deposit interest rate
is i, Show that the present value of the proceeds from this investment is D/(i j)
assuming the shares will be held indefinitely. (Hint: bank interest rate will reduce
the money value.)

10

MTA (G5078) Autumn 2009 Lecture Notes


1.5. Interest income

1.5.1. A lazy son of a rich. Keep capital at C and receive interest as income.
This is achieved by withdrawing continuously money from the account as to keep
it at C. Note that there is no inflation involved in this model, and money does not
lose value in time.
Fix a time t > t0 . The interest received at times
t0 + h, t0 + 2h, . . . , t0 + nh = t,

(1.5.1)

where h = (tt0 )/n and n is a positive integer, can be calculated as follows. Interest
payable at time t0 + (j + 1)h for the period (t0 + jh, t0 + (j + 1)h) is
Chih (t0 + jh),
and the total of all payments is
C

n1
X

hih (t0 + jh).

j=1

As n , we know that ih (t) (t), and we obtain the continuous version


C

n1
X

Zt
hih (t0 + jh) C

j=1

(s)ds =: I(t).
t0

(Here I and play the role of M and , respectively, in 1.4.2.)


If we invest at t0 = 0, and fix T as a final time to base our calculation on, the present
RT
value of our income that we will receive from time 0 up to time T is C (t)v(t)dt
0

and the final capitals present value is Cv(T ). On the other hand the present value
is C, therefore we obtain the relation
Z T
(1.5.2)
C
(t)v(t) dt + Cv(T ) = C.
| {z }
0
{z
} pr.val. of capital
|
pr.val. of income

A mathematical description of this fact can be retrieved by integrating



 Z t
Z T
Z T
(s) ds dt
(t)v(t) dt =
(t) exp
0
0
0
 Z t 
Z T
d
=
exp
dt
dt
0
0
  Z t t=T
= exp

  Z
= exp

t=0
T


1

= 1 v(T ).
Multiplying by C we recover
Z
C=C

(t)v(t) dt + Cv(T )
0

as we expected from our earlier reasoning.


1.5.2. Exercise. What happens in (1.5.1) when T ?

MTA (G5078) Autumn 2009 1.6. Interest income

11

1.5.3. Remark (Capital gains and losses). So far, we have described the difference
between money returned at the end of the term and the cash originally invested as
interest.
In practice, this quantity may be divided into interest income and capital gains. A
capital loss is used for negative capital gain.
1.5.4. Exercise. Use buy-to-let concept to explain the concept of interest income
and capital gain.
1.6. Compound Interest Functions
In the following, we will work under the constant force of interest assumption. We
assume, thus that (t) is a constant, which we denote also by .2 We review in
this short section the main properties of compound interest with constant force of
interest.
1.6.1. Effective discount rates. The value at time s of 1 due at time s + t is
s+t

Z
exp (r) dr = exp(t),
s

which is independent of s. Hence


v(t) = et = v t = (1 d)t ,
where the discount rate is defined as d := 1 v.
The number d is the effective discount rate per time unit. This is a loan of 1 at
which interest d is deducted in advance.
Similarly, the accumulated amount at time s + t of 1 invested at time s is
s+t

Z
exp (r) dr = exp(t) = (1 + i)t
s

which is independent of s.
1.6.2. Exercise. Find all the relationship between , v, i and d.
Solution.
v=

1
= 1 d = e .
1+i

(1.6.1)

1.6.3. Remark (interest, discount and the (wrong) law of universal linearity).
The interest rate i and the discount rate d have an interesting dual relationship,
summarised by
(1 + i)(1 d) = 1.
(1.6.2)
One way to remember this relation is to think i as the percentage gain that compensates for the percentage loss d.
It should be clear to the reader by now that d 6= i. This is a common mistake (known
as the law of universal linearity) whereby, for instance, many people think that if
the FTSE goes down by 5% one day and by 5% up then it has recovered. What is
your guess?
2We

trust the reader will not be confused when reading expressions like (t s)

12

MTA (G5078) Autumn 2009 Lecture Notes


Figure 1. Series of equal payments from time 1 to time n.
payments
1
1
1
1
1
1

n2n1

i
h
annuity value at
various times

ane a
ne

n+1

time

sne sne

The reason why many people commit this mistake (and the reason behind the mistakes nickname) is the Neumann series expansion (which is just another name of
the Taylor expansion)
1
= 1 x + x2 x3 + .
1+x

(1.6.3)

1 d = 1 i + i2 i3 + ,

(1.6.4)

d = i i2 + i3 + .

(1.6.5)

Using x = i we obtain
and hence
For very small i, say of the order of 1%, the remainder term (nonlinear part of the
expansion) is negligible, and one may assume, for all practical purposes that d = i.
But as soon as i becomes bigger, one should be more careful.
1.6.4. Problem. A lender bases his short-term transactions on a rate of commercial
discount D, where 0 < D < 1. For 0 < t 1, in return for a repayment of X after
a period t, he will lend X(1 Dt) at the start of the period.
For such a transaction over an interval of length t, (0 < t 1) derive an expression
in terms of D and t for d, the effective rate of discount per unit time. Hence show
that, regarded as a function of t, d is increasing on the interval 0 < t 1.
1.7. Annuities-certain: present values and accumulation
1.7.1. Annuity-certain, annuity-due, perpetuity. Consider a series of n payments, each of amount 1, made at time interval of 1 unit, the 1st payment is made
at time t0 + 1. We sketch the situation in Figure 1.
The value of this series of payments at time t0 (which is the present value if t0 = 0)
is denoted by ane , hence v = 1/(1 + i), and, for n 1, we get
ane = v + v 2 + . . . + v n =

1 vn
1 vn
=
.
1/v 1
i

(1.7.1)

If n = 0 (no payment ever done) then we define ane := 0. We refer the quantity
ane the present value of an immediate annuity-certain (annuity paid in arrear).
By contrast, the value of this series of payment at the time the first payment is
made is denoted by a
ne . Thus
a
ne = 1 + v + v 2 + + v n1 =

1 vn
1 vn
=
.
1v
d

The name of a
ne is called the present value of an (annuity paid in advance).

MTA (G5078) Autumn 2009 1.8. Annuities-certain: present values and


accumulation
13
The value of the series of payments at the time of the last payment is denoted by,
sne and therefore
(1 + i)n 1
.
i
The value of the series of payment one unit time after the last payment is made is
denoted by sne and we have
sne = (1 + i)n1 + (1 + i)n2 + + 1 = (1 + i)n an] =

(1 + i)n 1
.
d
When n , i > 0, the limit of an annuity (certain or due) is called a perpetuity
(certain or due, respectively):
sne = (1 + i)n + (1 + i)n1 + + (1 + i) = (1 + i)n a
n] =

ane ae and a
ne a
e .
Hence we have

1 vn
1
=
n
i
i

ae = lim
and

1 vn
1
= .
n
d
d
1.7.2. Problem. A loan of 2, 400 is to be repaid by 20 equal annual instalments.
The rate of interest for the transaction is 10% per annum. Find the amount of
each annual repayment, assuming that payments are made (a) in arrear and (b) in
advance.
a
e = lim

1.7.3. Problem. On 10 Jan in each of the years 1964 to 1979 inclusive, an investor
deposited 500 in a special bank savings account. On 10 Jan 1983, the investor
withdrew his savings. Given that over the entire period the bank used an annual
interest rate of 7% for its special savings accounts, find the sum withdrawn by the
investor.
1.7.4. Problem. A borrower agrees to repay a loan of 3000 by 15 annual repayments of 500, the first repayment being due after 5 years. Find the annual yield
(i.e., the interest rate the creditor has to apply) for this transaction.
1.7.5. Exercise (deferred annuity-certain). Let m and n be non-negative integers,
draw a cash flow chart (see 1.9) for a series of n payments, each of amount 1, due
at times (m + 1), (m + 2), . . . , (m + n). Find the value of these payments at time 0
(this is known as deferred annuity(-certain) and is denoted by m |ane .
1.7.6. Exercise (deferred annuity-due). Define deferred annuity-due

ane ?
m |

1.7.7. Exercise (continuous time annuity). Let t be a nonnegative real number,


using a constant force of interest , find the value of an annuity payable continuously
between time 0 and time t, where the rate of payment per unit time is the constant
1 (this is known as continuously payable annuities and is denoted by a
te ).
1.7.8. Exercise. Let n be a non-negative integer, draw the cash flow chart for a
series of n payments, each of amount Xj , due at times tj , where j = 1, . . . , n. Find
the value of these payments at time 0 and time tn .
Find a simple formula in the particular case when Xj = j, tj = j (this is known as
an increasing annuity and is denoted by (Ia)ne .
Hint. Multiply (Ia)ne by 1/v and compare with (Ia)ne , and use the fact that v =
1/(1 + i) where i is the effective interest rate for unit time.

14

MTA (G5078) Autumn 2009 Lecture Notes


1.8. The general loan or mortgage schedule

1.8.1. The basic . Assume that a sum L is lent for a series of n yearly payments.
The r-th payment, of amount xr , being due at time r. Let the effective annual
interest rate for r-th year be ir . Then we have
L = x1 (1 + i1 )1 + x2 (1 + i1 )1 (1 + i2 )1 + + xn (1 + i1 )1 (1 + in )1 . (1.8.1)
However, it maybe useful to split the payment in terms of capital and interest
useful when tax needs to be calculated.
Let
F0 = L,
Ft loan outstanding immediately after payment at time t,
xt be the loan repaid at t,
Then the interest due at time t, is given by
it Ft1 ,

(1.8.2)

and, for each t = 1, . . . , n we have


Ft =Ft1 (xt it Ft1 )
=(1 + it )Ft1 xt
=
=(1 + i1 )(1 + i2 ) (1 + it )L (1 + i2 )(1 + i3 ) (1 + it )x1
(1 + i3 ) (1 + it )x2 (1 + it )xt1 xt
t
t
t
Y
X
Y
(1 + ir ).
xs
=L (1 + is )
s=1

s=1

r=s+1

This result can be proven rigorously, by induction on t.


1.8.2. Exercise. Show that Ft is the value at time t of the outstanding payment:
Ft = (1 + it+1 )1 xt+1 + (1 + it+1 )1 (1 + it+2 )1 xt+2
+ . . . + (1 + it+1 )1 (1 + it+1 )1 (1 + in )1 xn

(1.8.3)

If i is the interest rate per unit time, a loan of amount sne is made at time 0 in
return for n repayments, each of amount 1, to be made at t = 1, . . . , n.
We split the payments into capital and interest: after the t-th payment, there remain
nt outstanding payments and the outstanding loan is then Ft = ante . The amount
of loan repaid at time t is
ft = Ft1 Ft = ant+1e ante = v nt+1 .

(1.8.4)

1.8.3. Exercise (producing an ). Make the schedule for a level annuity (amount of
loan ane ) by filling the following table appropriately.
Interest content of
Loan outstanding afPayment
Capital repaid
payment
ter payment
1
2
...
...
...
...
t
...
...
...
...
n1
n

MTA (G5078) Autumn 2009 1.1. Cash flow

15

1.8.4. Problem. A loan of 10, 000 pounds is to be repaid over 10 years by a level
annuity payable monthly in arrear. The amount of the monthly payment is calculated on the basis of an interest rate of 1% per month effective. Find
(a) The monthly repayment.
(b) The total capital repaid and interest paid in the first and last year respectively.
(c) After which monthly repayment does the outstanding loan become first less
than 5, 000?
(d) For which monthly repayment the capital repaid first exceeds the interest?
1.9. Cash flow
We have mentioned cash flows in the last few sections. We now give some more
ideas concerning cash flow.
1.9.1. Cash flow model terminology. The is a mathematical projection of the
payments arising from a financial transaction, e.g., a loan, a life insurance contract
or a capital project. Payments received are referred to as income and are shown
as positive cash flows. Payments made are referred to as outgo and are shown as
negative cash flows. The difference at a single point in time (income less outgo) is
called the at that point in time.
Where there is uncertainty about the amount or timing of cash flows, an actuary
can assign probabilities to both the amount and the existence of a cash flow.
1.9.2. Example (fixed interest security). An investor might buy a 10-year fixed
interest security of nominal amount 1, 000. This means that the face value of the
loan is 1, 000. The investor is unlikely to pay exactly 1, 000 for this security
but will pay a price that is acceptable to both parties. This may be higher or
lower than 1, 000. The investor will then receive a lump sum payment in 20 years
time. This lump sum is most commonly equal to the nominal amount, in this case
1, 000, but could be a pre-specified amount higher or lower than this. The investor
will also receive regular payments throughout the 20 years of, say, 50 pa. These
regular payments could be made at the end of each year or half year or at different
intervals.
1.9.3. Example (Index linked securities). The initial negative cash flow is followed
by a series of unknown positive cash flows and a single larger unknown positive cash
flow, all on specified dates. However, it is known that the amounts of the future
cash flows relate to the inflation index. Hence these cash flows are said to be known
in real terms.
1.9.4. Definition (real terms and nominal terms). Real terms means taking into
account inflation, whereas nominal means ignoring inflation.
1.9.5. Essay. Using internet, newspaper and financial journals to find information
of the following investment vehicles and analyse their cash flows respectively:
5. repayment loan, or
1. equity shares, also 2. cash on deposit;
3.
annuities;
mortgage;
known as shares or
equities in Britain 4. endowment assurance, 6. motor insurance;
or an interest-only 7. index linked bonds.
and
as
common
loan;
stock in the USA;

16

MTA (G5078) Autumn 2009 Lecture Notes


Exercises and problems on Theory of Interest Rate

Exercise 1.1 (accumulation factor). For integer times, 50 are invested at time
2 and the accumulated amount at time 7 is 100. Find i5 (2) and i(2).
Problem 1.2 (commervial vs. effective rate). A lender bases his short-term transactions on a rate of commercial discount D, where 0 < D < 1. For 0 < t 1, in
return for a repayment of X after a period t, he will lend X(1 Dt) at the start of
the period.
For such a transaction over an interval of length t, (0 < t 1) derive an expression
in terms of D and t for d, the effective rate of discount per unit time. Hence show
that, regarded as a function of t, d is increasing on the interval 0 < t 1.
Problem 1.3 (annuities). A loan of 2, 400 is to be repaid by 20 equal annual
instalments. The rate of interest for the transaction is 10% per annum. Find the
amount of each annual repayment, assuming that payments are made (a) in arrear
and (b) in advance.
Problem 1.4 (annuities). On 10 Jan in each of the years 1964 to 1979 inclusive,
an investor deposited 500 in a special bank savings account. On 10 Jan 1983, the
investor withdrew his savings. Given that over the entire period the bank used an
annual interest rate of 7% for its special savings accounts, find the sum withdrawn
by the investor.
Problem 1.5 (annuities). A borrower agrees to repay a loan of 3000 by 15 annual
repayments of 500, the first repayment being due after 5 years. Find the annual
yield (i.e., the interest rate the creditor has to apply) for this transaction.
Exercise 1.6 (accumulation factor). 5000 are invested at time 0 and the proceeds
at time 10 are 9000. Calculate A(6, 10) if A(0, 9) = 1.8, A(2, 4) = 1.1, A(2, 6) =
1.32, A(4, 9) = 1.45.
Exercise 1.7 (force of interest). If a large pension fund with a value of 2000m is
assumed to grow steadily subject to a constant force of interest of 10% per annum,
how much interest is earned every second? (Assume that there are 365 days in a
year.)
Exercise 1.8 (force of interest). A bank credits interest on deposit using accumulation factors based on a variable force of interest. On 1 July 1999, a customer
deposited 100,000 pounds with the bank. On 1 July 2001, his deposit has grown
to 120,000. Assuming that the force of interest per annum was a + bt (1 July 1999
= 0, unit = year) during the period, find the force of interest per annum on 1 July
2000.
Exercise 1.9 (discount factor). Calculate the present value on 1 September 2002
of payments of 280 due on 1 September 2004 and 360 due on 1 March 2005.
Interest is constantly 10% p.a. effective.
Exercise 1.10 (dividend gain versus interest gain). An investor owns a block of
shares which are expected to pay a dividend of amount D in one years time and
dividends in each future year that are j = 100j% higher than in the previous year.
Suppose that an alternative investment opportunity is a bank deposit whose interest
rate is i.
Show that the present value of the proceeds from the share investment is D/(i j)
assuming the shares will be held indefinitely.
Hint. The bank interest rate will act as an inflation rate and reduce the money value
in time.

MTA (G5078) Autumn 2009 1.21. Exercises and problems on Theory of


Interest Rate
17
Problem 1.11 (simple interest is simplistic). An investor puts 1,000 in a
savings account that pays 10% simple interest at the end of each year. Compare
how much the investor would have after 2 years if the money was:
(i) invested continuously for 2 years;
(ii) invested for 1 year, then immediately reinvested for a further year.
Problem 1.12 (Consistency Principle). Let  be a given integrable function. For
each t, s R define
Z s
B(t, s) = exp
( ) d.
(1.12.1)
t

Show that the function B obeys the Principle of Consistency, i.e.,


B(t0 , tn ) = B(t0 , t1 ) B(tn1 , tn ).
Exercise 1.13 (lazy son of a rich). What happens in C = C
when T ?

(1.12.2)
RT

(t)v(t)dt + Cv(T )

Problem 1.14. Use buy-to-let concept to explain the concept of interest income
and capital gain.
Exercise 1.15 (interest basic relations). Find all the possible relationships among
, v, i and d.
Exercise 1.16 (deferred annuity-certain). Let m and n be non-negative integers,
draw a cash flow chart (see 1.9) for a series of n payments, each of amount 1, due
at times (m + 1), (m + 2), . . . , (m + n). Find the value of these payments at time 0
(this is known as deferred annuity(-certain) and is denoted by m |ane .
Exercise 1.17 (deferred annuity-due). Define deferred annuity-due

ane ?
m |

Exercise 1.18 (continuously payable annuity). Let t be a nonnegative real number, using a constant force of interest , find the value of an annuity payable continuously between time 0 and time t, where the rate of payment per unit time is the
constant 1 (this is known as continuously payable annuities and is denoted by a
te ).
Exercise 1.19 (cash flow). Let n be a non-negative integer, draw the cash flow
chart for a series of n payments, each of amount Xj , due at times tj , where j =
1, . . . , n. Find the value of these payments at time 0 and time tn .
Find a simple formula in the particular case when Xj = j, tj = j (this is known as
an increasing annuity and is denoted by (Ia)ne .
Hint. Multiply (Ia)ne by 1/v and compare with (Ia)ne , and use the fact that v =
1/(1 + i) where i is the effective interest rate for unit time.
Exercise 1.20 (variable discount rate in discrete time). Show that Ft is the value
at time t of the outstanding payment:
Ft = (1 + it+1 )1 xt+1 + (1 + it+1 )1 (1 + it+2 )1 xt+2
+ . . . + (1 + it+1 )1 (1 + it+1 )1 (1 + in )1 xn

(1.20.1)

Problem 1.21 (cash flow). A loan of 10, 000 pounds is to be repaid over 10 years
by a level annuity payable monthly in arrear. The amount of the monthly payment
is calculated on the basis of an interest rate of 1% per month effective. Find
(a) The monthly repayment.
(b) The total capital repaid and interest paid in the first and last year respectively.

18

MTA (G5078) Autumn 2009 Lecture Notes

(c) After which monthly repayment does the outstanding loan become first less
than 5, 000?
(d) For which monthly repayment the capital repaid first exceeds the interest?

CHAPTER 2

Nominal rate of interest and fractionally convertible


annuities
Throughout this chapter we assume that the force of interest per unit time (t), is
constant for all times t and denote it by .
2.1. Interest payable fractionally
2.1.1. Summary of rates of interest and discount. Let i and d be effective
rates of interest and discount, respectively, over a period of unit time corresponding
to the force of interest . We showed that d payable at time 0, i payable at time 1,
and payable continuously at a constant rate over the time interval [0, 1] all have the
same value (on the basis of a constant force of interest ). Each of these payments
may be regarded as the interest for the period [0, 1] payable on a loan of 1 made at
time 0.
This can be summarised as follows, from the borrowers view-point: I take 1 from
the lender at time 0, with the understanding that
(i) I pay the borrowed ammount (i.e., 1) at time t,
(ii) I pay the interest: (a) immediately, (b) at time t = 1, or (c) continuously at
a constant rate throughout [0, 1].
The situation (b) is the easiest to describe, the interest to be payed is i at t = 1 and
it can be summarised in the simple table
time earn pay (capital) pay (interest)
0
1
0
0
1
0
1
i
Let us ignore situation (c) and come back to it when we understand (a) and (b).
The situation (a) can be described in two ways. The first way, which is the definition
of d seen in Chapter 1, can be summarised by the following table
time earn pay (capital) pay (interest)
0
1
0
d
1
0
1
0
Using description (b) to talk about (a) we get the following table
time earn pay (capital) pay (interest)
0 1d
0
0
1
0
1d
(1 d)i
This is basically telling us that
(1 + i)(1 d) = 1,

(2.1.1)

which the relationship from Chapter 1.


In this section we want to generalise the modality of paying the interest in (a) and
(b), by spreading it to more than one repayment. Let us begin with a simple case.
2.1.2. Example.

20

MTA (G5078) Autumn 2009 Lecture Notes

Problem. Suppose a borrower-lender agreement stipulates that the repayment can


be spread throghout the period [0, 1] in 4 payments, the first of which is at the end of
the first quarter (i.e., a fourth). Namely, they agree to follow the following schedule
time earn pay (capital) pay (interest)
0
1
0
0
1/4
0
0

1/2
0
0

3/4
0
0

1
0
1

Calculate such that the equivalent annual (time-unit)interest is i.


Solution. To calculate we note that
1+i
| {z }

owed ammount

= (1 + ) +
| {z }
last payment

(1 + i)1/4
| {z }

interest from t = 3/4

(1 + i)1/2
| {z }

interest from t = 1/2

(1 + i)3/4
| {z }

interest from t = 1/4

(2.1.2)
A quick calculation using the geometric-telescopic identity,
(1 + r + + rn )(1 r) = (1 rn+1 ),

(2.1.3)

true for all r R and n N, shows that


(1 + i) 1
,
(1 + i)1/4 1

(2.1.4)

= (1 + i)1/4 1.

(2.1.5)

i=
and thus

A good name for is the (effective) interest rate payable quarterly (or fourthly).
One could be also interested in the (total) nominal interest rate payable quarterly
which is given by 4.
2.1.3. Definition (rate of interest convertible fractionally). A borrower, who is lent
1 at time 0 for repayment at time 1, agrees to pay the capital 1 at time 1 and to
spread the payment of the interest on his loan in p equal instalments over the time
interval [0, 1] (e.g., if 1 time unit stands for a year and we want to repay monthly
then p = 12). Supposing that the money is borrowed at an effective rate i over the
period [0, 1], which is compounded p-thly, how much interest should he pay at each
installment?
Denote by the amount of interest to be payed at the end of each instalment and
i(p) to be the total amount of interest of such a payment.1
So, the borower pays the lender the amount at times 1/p, 2/p, 3/p, . . . , 1. The
interest rate has the same value as each of the interest payment.
Let us express i(p) in terms of i. Since i(p) is the total interest paid, the p-thly
interest rate is constant, then each interest payment is of amount = i(p) /p. On
the other hand, the effective rate being i, the borrower owes 1 + i at time 1, (1 for
the capital repayment and i as interest). Each repayments at then end of each
interval [(t 1 p)/p, (t p)/p), with a fixed t = 1, . . . , p, bears a contribution of
(1 + i)(tp)/p

(2.1.6)

towards the final settlement.


1Note

that this notation clashes with the one introduced in 1.1.4 of Chapter 1. We should be
using another notation, but this is the one that is found in most textbooks and so we stick with
it, with a warning.

MTA (G5078) Autumn 2009 2.1. Interest payable fractionally


Thus

p
X
i(p)
t=1

21

(1 + i)(pt)/p = i.

Excluding the trivial case where i = 0 = i(p) , and changing index t 7 p t, we have
i=

p1
X

(1 + i)1/p

t

t=0

i(p) (1 + i) 1
,
p (1 + i)1/p 1

whence the relation


and its inverse


i(p) = p (1 + i)1/p 1

(2.1.7)

p

i(p)
1.
i= 1+
p

(2.1.8)

Likewise we define d(p) to be the total amount of interest, payable in equal instalments at the start of each of the p subintervals, i.e., at times 0, 1/p, 2/p, . . . , (p1)/p.
Also here, the interest rate is taken to be constant with respect to the payment interval. It is thus a consequence of the definition of d(p) that
n
X
d(p)
(1 d)(t1)/p = d
p
t=1
or, if d 6= 0,
d(p) 1 (1 d)
=d
p 1 (1 d)1/p
Hence
d(p) = p 1 (1 d)1/p
and

p

d(p)
1
=1d
p

(2.1.9)
(2.1.10)

It is customary to refer to i(p) and d(p) as nominal rates of interest and discount
convertible p-thly.
2.1.4. Continuous time repayment. The continuous repayment option (c), with
a constant force of interest cannot be expressed in terms of a table; but we can still
think of the borrower, in order to earn 1 at time 0, to have to pay 1 at time 1 and
to spread the interest repayment by paying a constant infinitesimal amount dt at
each time t between 0 and 1. Thus the payment of dt at time t will provide an
interest of (1 + i)1t dt up to time 1. The total repayment is thus given by
Z 1
1+
(1 + i)1t
dt
| {z }
| {z }
0
interest factor from time t infinitesimal payment at t
(2.1.11)
Z 1
i
=1+
exp(log(1 + i)t) dt = 1 +
.
log(1 + i)
0
This amount must match the one from repayment option (a), hence
1+i=1+

i
,
log(1 + i)

(2.1.12)

which implies the well known relationship


e = 1 + i.

(2.1.13)

22

MTA (G5078) Autumn 2009 Lecture Notes

2.1.5. Example. If we speak of a rate of interest of 12% per annum convertible


quarterly (i.e., 4-thly), we have i(4) = 0.12 (with one year as the unit of time). Since
(1 + i) = (1 + i(4) /4)4 , this means that i = 0.125509. Thus the equivalent annual
rate of interest is 12.5509%.
Here i is the equivalent rate per unit time as an effective rate. Thus, if the nominal rate of interest convertible quarterly is 12%, the effective rate per annum i is
12.5509%.
Equivalent interest repayments: (a) one settlement in advance, (a) p-thly in advance, (b) in arrear, (b) p-thly in arrear, (c) continuously at a constant rate.
time
0
1/p
2/p
3/p
. . . (p 1)/p
1
(a)
d
(p)
(a) d /p d(p) /p d(p) /p d(p) /p . . .
d(p) /p
(p)
(p)
(p)
(b)
i /p i /p i /p . . .
i(p) /p
i(p) /p
(b)
i
(c)
dt
dt
dt
dt
...
dt
dt
Note that description of (c) is a bit strange in a table. In fact, we could not do this
rigorously and it has to taken as a heuristic description, whereby we think of 1/p
being equal to dt and p infinite. This is a useful intuitive physical description
of a continous payment in continuously many infinite instalments, but it has to be
handled with care as it is not very rigorous.
2.1.6. Example (effective rates to nominal rates). Given that = 0.1, find the
values of
(a) i, i(4) , i(12) , i(52) , i(365)
(b) d, d(4) , d(12) , d(52) , d(365)
Solution. Using (2.1.7) and (2.1.9) respectively we can draw the following table
p
(p)

i
d(p)

1
4
12
52
365
0.105171 0.101260 0.100418 0.100096 0.100014
0.095163 0.098760 0.099584 0.099904 0.099986

2.1.7. Example. Given that i = 0.08, find the values of i(12) , d(4) , and .
Solution.
d(4)


i(12) = 12 (1 + i)1/12 1 = 0.077208,


= 4 1 (1 d)1/4 = 4 1 (1 + i)1/4 = 0.076225,
= log(1 + i) = 0.076961.

2.1.8. Problem. Suppose that l and m are positive integers. Express i(m) in terms
of l, m, and d(l) . Hence find i(12) when d(4) = 0.057847.
Hint. Use to link the two quantities.
2.2. Annuity payable fractionally
We have learned about annuities in 1.7 of Chapter 1. The purpose of this section
is to extend the ways of payment of an annuity from one yearly instalment to p
p-thly instalments per year (assuming the year to be the period of the annuity).
A typical example where this is important are mortgages on a house purchase, where
the interest rate is given in a yearly percentage, but the payments occur monthly
(p = 12).

MTA (G5078) Autumn 2009 2.2. Annuity payable fractionally

23

2.2.1. Definition of an annuity payable fractionally. We call annuity-certain


payable p-thly a sequence of payments, each worth 1/p, performed p times per unit
time, and over a given number, say n of time units (usually years).
Unless otherwise stated, the usual unit for time is a year, thus the model we have
in mind is that of an annuity which, instead of being paid once a year (say at the
end of the year), is paid in p instalments (say in arrears), for example at the end of
each month when p = 12.
2.2.2. Basic relations for an annuity payable fractionally in arrear. If p and
(p)
n are positive integers, ane denotes the value at time 0 of a level annuity payable
p-thly in arrear, over [0, n]. In detail, there are np payments in total with each
instalment worth 1/p per unit time being paid at the end of each subperiod of
length 1/p, over the time interval [0, n].
For this annuity the payments are made at times 1/p, 2/p, 3/p, ..., n and the amount
of each payment is 1/p.
It is a simple matter to derive an expression for a(p) e from first principles. However,
we first use the following argument which has widespread application.
By definition, a series of p payments, each of amount i(p) /p in arrear at p-thly
subintervals over any unit time interval, has the same value as a single payment of
amount i at the end of the interval as obtained in the discussion leading (2.1.7). By
proportion, p payments, each of amount 1/p in arrear at p-thly subintervals over
any unit time interval, have the same value as a single payment of amount i/i(p) at
the end of the interval.
(p)
Consider now that annuity for which the present value is ane the p payments after
time r 1 and not later than time r have the same value as a single payment of
amount i/i(p) at time r. This is true for r = 1, 2, . . . , n, so the annuity has the same
value as a series of n payments, each of amount i/i(p) , at times 1, 2, . . . , n. This
means that
i
1 vn
(p)
ane = (p) ane = (p) .
i
i
2.2.3. Exercise. Prove the second equality above.
2.2.4. Annuity payable fractionally in advance. Similarly we can define an
annuity-due payable p-thly whereby the payments are performed at the start of
(p)
each subperiod of length 1/p. This quantity is denoted by a
ne to be the present
value of a level annuity-due payable p-thly at the rate of 1 per unit time over the
time interval [0, n]. (The annuity payments, each of amount 1/p,are made at times
0, 1/p, 2/p, ..., n 1/p.)
By definition, a series of p payments, each of amount d(p) /p, in advance at p-thly
subintervals over any unit time interval has the same value as a single payment
of amount i at the end of the interval. Hence, by proportion, p payments, each
of amount 1/p in advance at p-thly subintervals, have the same value as a single
payment of amount i/d(p) at the end of the interval. This means (by an identical
argument to that above) that
1 vn
.
d
d(p)
2.2.5. Exercise. Prove the second equality above.
(p)

a
ne =

(p)

2.2.6. Exercise. Prove that ane =


rate of i(p) to the payments.

a =
(p) ne

1v n
i(p)

from first principles, by applying a nominal

24

MTA (G5078) Autumn 2009 Lecture Notes


(p)

(p)

2.2.7. Annuity. Similarly, we define sne and sne to be the accumulated amounts
of the corresponding p-thly immediate annuity and annuity-due respectively. Thus
i
i
(p)
(p)
sne = (1 + i)n ane = (1 + i)n (p) ane = (p) sne
i
i
and
i
i
(p)
(p)
sne = (1 + i)n a
ne = (1 + i)n (p) ane = (p) sne .
d
d
The above proportional arguments may be applied to other varying series of payments. Consider, for example, an annuity payable annually in arrear for n years, the
payment in the tth year being xt . The present value of this annuity can be obtained
as
n
X
a=
xt v t .
t=1

Consider now a second annuity, also payable for n years with the payment in the
t-th year, again of amount xt being made in p equal instalments in arrear over that
year. If a(p) denotes the present value of this second annuity, by replacing the p
payments for year t (each of amount xt /p) by a single equivalent payment at the
end of the year of amount xt i/i(p) , we immediately obtain
a(p) = a

i
i(p)

where a is given above.


2.2.8. Definition (perpetuity payable fractionally). An annuity payable fractionally in arrear, under which the payments continue for ever, is called a perpetuity
payable fractionally.
When the rate of payment is constant and equal to 1 per unit time, the present
(p)
value of such a perpetuity is denoted by ae . If the payments are in advance, we
(p)

have a perpetuity-due, with the corresponding present value denoted by a


e .
(p)

(p)

2.2.9. Exercise. Find a relationship between a


e and ae .
2.2.10. Exercise. The present values of an immediate annuity and an immediate
annuity-due, payable p-thly at the rate of 1 per unit time for n time units and
(p)
(p)
ane . Find the relation
deferred for m time units are denoted by m |ane and m |
between

(p)
m |ane

(p)

and ane , and

(p)
ane
m |

(p)

and a
ne .

2.2.11. Example.
Problem. An investor wishes to purchase a level annuity of 120 per annum
payable quarterly in arrear for five years. Find the purchase price, given that it
is calculated on the basis of an interest rate of 12% per annum:
(a) Effective
(b) Convertible half-yearly
(c) Convertible quarterly
(d) Convertible monthly.
Solution. (a) The value is
(4)

120a5e at 12% = 120

a5e
i(4)

= 451.583.

(2.2.1)

MTA (G5078) Autumn 2009 2.4. Annuity payable fractionally

25

(b) Since the rate of interest is a nominal one convertible half-yearly, we take the
half-year as our unit of time and 6% as our rate of interest. The annuity is payable
twice per half-year for ten half-years at the rate of 60 per half-year. Hence its
value is
i
(2)
60a10e at 6% = 60 (2) a10e = 448.134
i
(c) We take the quarter-year as the unit of time and 3% as the rate of interest.
The value is thus
30a20e at 3% = 446.324.
(d) We take the month as the unit of time and 1% as the rate of interest. The
annuity payments of amount 30, at the end of every third month can be replaced
by a series of equivalent monthly payments, each of 30/s3e (at 1%). The value is thus
30
a60e at 1% = 445.084.
s3e
2.3. Annuities payable at time r > 1
In Section 2.2 we showed how, by replacing a series of payments to be received by
an equivalent series of payments of equal value, we could immediately write down
an expression for the value of a fractional annuity.
2.3.1. Extension to higher integers. The same technique, i.e., the use of equivalent payments of the same value, may be used to value a series of payments of
constant amount payable at intervals of time length r, where r is some integer
greater than 1.
For example, suppose that k and r are integers greater than 1 and consider a series
of payments, each of amount X,due at times r, 2r, 3r, . . . , kr. What is the value of
this series at time 0 on the basis of an interest rate i per unit time?
Let us replace the first payment of X due at time r by a series of r payments, each
of amount Y , due at times 1, 2, . . . , r, where Y is chosen to make these r equivalent
payments of the same total value as the single payment they replace. This means
that Y and X are related by Y sre = X at rate i.
Similarly each payment of amount X can be replaced by r equivalent payments of
amount Y of the same value. Thus the series of payments of X, due every rth time
interval, has the same value as a series of kr payments, each of Y = X/sre due at
unit time intervals.
Hence the value of the annuity equals
akre
Y akre = X
(2.3.1)
sre
at rate i.
By analogy with the p-thly payments we may extend the definition of annuity payable
p-thly in advance to fractional values of p = 1/r, r integer, by replacing n with kr
and X by r
rane
(1/r)
.
(2.3.2)
ane :=
sre
2.3.2. Exercise. Prove this result by using the first principles (i.e., by summing
the appropriate geometric progression).

26

MTA (G5078) Autumn 2009 Lecture Notes


2.4. The loan schedule for a fractional annuity

We outline here how to present a fractional annuity in practical terms. No new


principles are involved with respect to the general load schedule discussed in 1.8.
For a loan repayable by a level annuity payable p-thly in arrear over n time units and
based on an interest rate j per unit time, the schedule is best derived by working
with an interest rate i(p) /p per time interval of length 1/p. Thus the interest due
at time r/p(r = 1, . . . , np) is i(p) /p times the loan outstanding at time (r 1)/p
(immediately after the repayment then due has been received).
(p)
For example, in relation to a loan of ane (at rate j) we have that the capital repaid
in the r-th annuity payment (r = 1, 2, . . . np) is (1/p)v n(r1)/p and that the loan
(p)
outstanding immediately after the r-th payment has been received is a a(nr/p)e (at
rate i). (This is simply the value of the outstanding payments.)
2.4.1. Example.
Problem. A loan of 1000 is repayable by a level annuity payable half-yearly in
arrear for three years and calculated on the basis of an interest rate of 15% per
annum effective.
Construct the lenders schedule showing the subdivision of each payment into capital
and interest and the loan outstanding after each repayment.
Solution. At 15%, i(2) /2 = 0.072 381, so the interest due at the end of each halfyear is 7.2381% of the loan outstanding at the start of the half-year. The amount
(2)
of the annual repayment is 1000/a3e at 15%, i.e. 422.68. The half-yearly payment
is thus 211.34. The schedule can then be drawn up as follows
(1)
(2)
(3)
(4)
Loan outstanding at Interest due at end of Capital repaid at end
of n-th half-year ()
n start of n-th half- n-th half-year ()
year ()
(0.072381 (2))
(211.34 (3))
1
1000.00
72.38
138.96
2
861.04
62.32
149.02
3
712.02
51.54
159.80
4
552.22
39.97
171.37
5
380.85
27.57
183.77
6
197.08
14.26
197.08
Exercises and problems on Rates convertible p-thly
Problem 2.1. Suppose that l and m are positive integers. Express i(m) in terms
of l, m, and d(l) . Hence find i(12) when d(4) = 0.057847.
Hint. Use to link the two quantities.
(p)

Exercise 2.2. Prove that ane =


rate of i(p) to the payments.

1v n
i(p)

from first principles, by applying a nominal

Exercise 2.3. Give a mathematical proof of the formula


iane
(p)
ane = (p) .
i
Exercise 2.4. Prove that
1 vn
(p)
a
ne =
.
d(p)
(p)

(p)

Exercise 2.5. Prove that ane = v 1/p a


ne .

(2.4.1)

MTA (G5078) Autumn 2009 2.7. Exercises and problems on Rates


convertible p-thly
(p)

27

(p)

Exercise 2.6. Find a relationship between a


e and ae .
Exercise 2.7 (annuities and deferred annuities). The present values of an immediate annuity and an immediate annuity-due, payable p-thly at the rate of 1 per
(p)
unit time for n time units and deferred for m time units are denoted by m |ane and
(p)
ane .
m |

Find a relation between

(p)
m |ane

(p)

and ane , and

(p)
ane
m |

(p)

and a
ne .

CHAPTER 3

Applications of Interest Rate Theory


3.1. The valuation of some securities
Securities are a common type of investment that investors may buy on a personal
basis from corporations, governments or other type of companies. These are usually
associated with some form of economic production. For example, it could be a
share of a manufacturing company. The deal consists in paying a certain lumpsum at issue and then reaping the benefits by collecting a given amount for a fixed
number of years, after which the investor is given the choice to either sell back
or redeem her share or to reinvest it for another fixed amount of years. We
will look at the situation where the investor redeems the capital at the securities
expiration.
3.1.1. Cash flow model for securities. Generally speaking a security investment
consists of three types of transactions:
1. an to purchase the security (or stock) at its issue,
2. a regular at given n intervals in time,
3. a final payment to sell back the security.
We summarise this cash-flow with a diagram in Figure 1.
3.1.2. Terminology and notation for security investment.
Security: name of the type of contract, in specialised fields securities are given
special names bonds (government), debentures, equities (medium sized companies), etc.
Investor: also called buyer or (stock/share-)holder
Holding: N is the number of total security units held.
Coupon rate: D is the rate of interest p.a., known as coupon rate (D may vary
with time).
(a) Fixed interest securities such as s, are usually ed by governments or giant
corporations. In these securities D is usually constant throughout the
holding period.
(b) Equities are large to medium sized company issued securities.
Redemption price: R is redemption price per unit nominal.
Payment frequency: p the frequency of interest payment per time unit (usually
p.a.).
Nominal amount: The nominal amount is the amount given to the stock at
issue, we say that a security is

30

MTA (G5078) Autumn 2009 Lecture Notes


Figure 1. Diagram representing a securitys cash-flow model, described in 3.1.1, with time in abscissa and payments (positive for
inflow, negative for outflow) in ordinate.

DN DN DN

0
1

C = RN = redemption price

DN DN DN coupon
payments
time
n2n1

n+1

A = P N = purchase price
(a) above par, or at a premium, if a bond of 100 nominal is worth more than
100 (if nothing else is specified, this is meant to happen at redemption)
(b) below par, or at a discount, if a bond of 100 nominal is worth less than
100,
(c) at par, if a bond of 100 nominal is worth 100.
Price per unit: P is the (purchase) price per unit nominal. In practise, if nothing else is specified, stocks are quoted per 100 nominal.
Price: A = N P is the price of purchase of the holding.
Redemption price: C = N R is the cash received at redemption.
3.1.3. Ordinary shares or equities. These securities are issued by commercial
companies. The holders are entitles to all the net profits after deduction of payment
of interests on loans and fixed interest stocks.
If a share is bought ex-dividend, then the seller collects the dividend. If it is bought
cum dividend, then the buyer receives the dividend. This is different from fixed
interest securities where interest is shared according to the number of days owned
by the seller and buyer.
In most countries, taxation laws require a levy on the income from equities (or
securities in general). This is known as the dividend tax (even for ex-dividend
shares). We will denote the percentage of this share by and be careful not to
confuse it with the time variable t.
3.1.4. Example (Compound interest valuation of ordinary shares).
Problem. A pension fund, which is subject to 20% dividend tax but no capital
gains tax, has a large holding of a single UK company share with current market
value of 14 700 000. The current rate of dividend payment per year is 620 000p
(payable at the end of the year). The pension fund wishes to value its holding, at
an effective interest rate of 6% per annum effective, on the assumption that
(i) both dividend income from the share and its market value will increase at the
rate of 2% per annum,
(ii) the shares will be sold in 30 years time.
What value should the pension fund place on the shares?
Solution. Let us start by identifying the terms. We have = 0.2 (the tax rate),
the payment effected each year is Xk , based on the initial payment of X0 = 620 000
per year which must be multiplied by 1.02 for each additional year, i.e., Xk =
Xk1 (1.02) = X0 (1.02)k . Accounting for an effective interest rate of 0.06, we get

MTA (G5078) Autumn 2009 3.2. The valuation of some securities

31

that the discount rate is v = 1/1.06. The number of years is n = 30. So, the capital
value at redemption time is C = 14 700 000 (1.02)n .
Thus, using the Equation of Value, the (present) value placed on the holding must
be

(1 )X0 (1.02)v + (1.02)2 v 2 + + (1.02)v n + Cv n
1 (1.02)n v n
= (1 )X0
(1.02)v + Cv n
1 (1.02)v

30
(1 (1.02/1.06)30 )1.02
1.02
= 620 000 0.8
+ 14 700 000
1.06 1
1.06
= 13 295 152.

(3.1.1)

3.2. Price and yield relation


In this section we address the following problems.
(i) What price A, or P per unit nominal, should be paid by an investor to secure
a net yield of i per annum?
(ii) Given that the investor pays a price A, or P per unit nominal, what net yield
per annum will he obtain?
We define yield of a security, that interest rate which makes the following price-yield
relation true.
3.2.1. The price-yield relation. The solution to problem (i) can be summarised
in the following relation:
present value, at rate
present value, at rate i
A = i p.a., of net interest + p.a., of net capital re- =: F (i),
payments
demption

(3.2.1)

where F is a certain function of the yield i.


Relation (3.2.1) provides a value for the price A, by directly evaluating F , for each
chosen yield value i and answers question (i). The price per unit nominal is then
obtained via P = A/N, where N is the nominal amount of stock to which the
payments relate.
3.2.2. Definition (net vs. gross). A net amount means after deduction of taxes,
fees, etc from the gross amount.
If the investor is not subject to taxation, or any other form of fees, then net equals
gross and the yield is sometimes referred to as a gross yield.
The yield quoted in the press for a fixed-interest security is often the gross nominal
yield per annum, convertible half-yearly.
3.2.3. The yield-price relation. To answer to problem (ii) is obtained by inverting the function F appearing in the relation (3.2.1).1
3.2.4. Remark (redemption yield vs. flat/running yield). If an investor sells her
holding before redemption, or if she is subject to taxation, her actual yield will in
general be different from that quoted.
1Inverting

a nonlinear function is not a trivial matter, mathematically speaking, and nonlinear


r
solver implementations (such as Newtons iteration, called via fsolve in Octave [Eat] or Matlab
)
will help. Before the widespread use of computers actuaries used tables and linear interpolation to
calculate these inverses; you may find in many of the books written up to the 1980s extensive use
of such techniques which are now considered obsolete. They are still useful though, if you wanted
to do some back-of-the-envelope calculations.

32

MTA (G5078) Autumn 2009 Lecture Notes

The yield on a security is sometimes referred to as the yield to redemption or the


redemption yield to distinguish it from the flat (or running) yield, which is defined
as D/P, the ratio of the coupon rate to the price per unit nominal of the stock.
3.2.5. Example (debenture).
Problem. A certain debenture (i.e., a fixed-interest stock issued by a commercial
company) was redeemable at par on 1 September 1967. The stock bore interest at
6% per annum, payable half-yearly on 1 March and 1 September.
(a) What price percent should have been offered for this stock on 1 July 1945 to
secure a yield of 5% per annum for a tax-free investor?
(b) What yield per annum did this stock offer to a tax-free investor who bought it
at 117% on 1 July 1945?
Solution. (a) In this example we have R = 1, N = 100, C = 100, D = 0.06, and
p = 2. The yield being i = 0.05, means that the present value has to be calculated
using this i and yearly present value v = 1/(1 + i).
The price A which should be offered on 1 July 1945 to secure a yield of 5% pa is
A = v 1/6 A
where A is the price which should be offered on 1 September 1945 (the v 1/6 comes
from the fact that the value A is discounted by 2 months and v is the yearly present
value.) To calculate A we note that
(p)

A = N D/p +
| {z }
paym. 1.9.45

N Dane
| {z }

pres. val. of int. paym. 1.3.461.9.67

+ N
Rv n}
| {z

redemption

(3.2.2)

(2)

=3 + 6a22e + 100v 22 at 5%.


Thus A = v

1/6

3+

(2)
6a22e

+ 100v

22

at 5%, which implies A = 116.9.

(b) The condition implies that




(2)
117 = v 1/6 3 + 6a22e + 100v 22 =: F (i ),

(3.2.3)

where i is the value we are after. To obtain i we must solve the nonlinear equation
F (i ) 117 = 0. This can be achieved with the help of a computational package
(or, less efficiently, by interpolation and trial-and-error). We obtain
i = 0.049437.

(3.2.4)

r
The following series of printouts is an example of Octave (or Matlab
) files that
2
solve this problem for us.

2The

r
only difference between Octave and Matlab
, in this case, is the endfunction keyword

r
which you should comment for Matlab use.

MTA (G5078) Autumn 2009 3.2. Price and yield relation


Printout of file Code/equity1.m

function istar = equity1 ()


% prepare the graphics
close all ;
% a suitable range for plotting
i =[0:.01:1];
% plot the function equity1fun
plot (i , equity1fun ( i ) ) ;
hold on ;
% make the plot more meaningful by embellishing
plot ([0 ,1] ,[0 ,0] , b ) ;
ylabel ( F ( i ) ) ;
xlabel ( i ) ;
% the first part of problem and plot suggest the seed
i0 =0.05;
% fsolve uses a Newton method to find a root of polynomial
istar = fsolve ( equity1fun , i0 ) ;
istar *100;
endfunction

Printout of file Code/equity1fun.m

function F = equity1fun ( i )
v = vfromi ( i ) ;
F = v .^(1/6) .*(3+6* acpfromip (i ,22 ,2) +100* v .^22) -117;
endfunction

Printout of file Code/ipthly.m

function ip = ipthly (i , p )
ip = p .*((1+ i ) .^(1/ p ) -1) ;
endfunction

Printout of file Code/acpfromip.m

function a = acpfromip (i ,n , p )
a = (1 - vfromi ( i ) .^ n ) ./ ipthly (i , p ) ;
endfunction

33

34

MTA (G5078) Autumn 2009 Lecture Notes


Printout of file Code/vfromi.m

function v = vfromi ( i )
v = 1./(1+ i ) ;
endfunction

3.2.6. Example.
Problem. A newly issued stock bears interest at 7 21 % per annum, payable annually
in arrear, and is redeemable at par in 20 years time. Find the net yield per annum
to an investor, liable to income tax at 33 13 %, who buys a quantity of this stock at
80% of the nominal price.
Solution. We have coupon rate D = 0.075, price paid per unit nominal P = 0.8,
redemption price per unit nominal R = 1, rate of income tax = 13 , and term to
redemption n = 20. The equation of value is
P = D(1 )ane + Rv n at rate i,
i.e.,
0.8 = 0.05a20e + v 20 .
By bisection-interpolation (or using a Newton method on a computer), we find that
i = 6.8686% up to 4 digits.
3.3. Perpetuity
3.3.1. Why use perpetuity? If a security is undated, i.e., if it has no final redemption date, or if it runs for a very long time, it is regarded and valued as a .
(A practical example of such securities are loans on land purchase in Great Britain,
where the land property is sometimes leased for 999 years. These numbers are as
high as infinity for all practical purposes and a perpetuity calculation will work.)
3.3.2. How to use a perpetuity? Assume that the first interest payment is due
at time t years (t not necessarily integer) from the present, and that interest is at
rate D per annum per unit nominal, payable p times per annum. The price P per
unit nominal to give a net yield of i per annum, and liable to income tax at rate ,
may be found from the equation
D(1 )v t
at rate i.
d(p)
This equation is obtained by taking n in the relation
(p)

P = D(1 )v t a
e =
(p)

(p)

P = D(1 )v t1/p ane + v n+t = D(1 )v t a


ne + v n+t1/p ,
and recalling, from Chapter 2, that 0 < v < 1 and
1
(p)
(p)
lim a
ne =: a
e = (p) .
n
d
3.3.3. Example (perpetual loan).
Problem. Assume that the interest on 3 21 % perpetual loan is payable on 1 June and
1 December each year, find (a) the effective yield per annum and (b) the nominal
yield per annum, convertible half-yearly, to a tax-free investor on 22 August 1983,
when the price was 34.875%.

MTA (G5078) Autumn 2009 3.4. Perpetuity

35

Solution. We have P = 0.34875, D = 0.035, p = 2 and t = 101/365 = 0.27671


(101 are the days from the purchase date 22 August 1983 to the first payment date
1 December 1983). We therefore solve the equation
0.34875 = 0.035[v 0.27671 /d(2) ]

(3.3.1)

for i. This gives i = 10.53% and hence,



i(2) = 2 (1 + i)1/2 1 = 10.27%.

(3.3.2)

3.4. Makehams formula


In practise, it is often desirable to evaluate the price of a security in terms of its
redemption value C at time n and the present value of C evaluated at the rate
equalling the yield i. Namely, let the capitals present value be K := Cv n = C/(1 +
i)n , how can we express A in terms of C and K directly? Makehams formula, which
we derive next, answers this question.
3.4.1. Derivation. Consider a loan, of nominal amount N , which is to be repaid
after n years at a price of R per unit nominal, and let C = N R. Thus C is the cash
payable on redemption. Let the coupon rate (i.e., the annual interest in money
terms, not percentage terms - per unit nominal) be D, and assume that interest is
payable p-thly in arrear. Thus each interest payment is of amount DN/p = gC/p
where
g = DN/C = D/R.
(3.4.1)
Note that g quantifies the annual rate of interest per unit of redemption price.
Consider an investor, liable to income tax at rate who wishes to purchase the
loan at a price such as to provide an effective net yield of i per annum. Let the
price he should pay be A. (We assume that n is an integer multiple of 1/p and that
any interest now due will not be received by the purchaser.) The price is simply
the present value (at rate i) of the redemption proceeds and the future net interest
payments. Thus
A =pres. val. of redemption + pres. val. of net interest payments
(p)

=N Rv n + (1 )DN ane at rate i


(p)

=Cv n + (1 )gCane

(3.4.2)

1 vn
i(p)
n
Recalling that K = Cv and rearranging terms we obtain
(1 )g
A=K+
(C K).
(3.4.3)
i(p)
Note that K is the present value of the capital repayment, and(C K) g(1 )/i(p)
is the present value of the net interest payments.
Relation (3.4.3) is known as Makehams formula and is of great importance.
=Cv n + (1 )gC

3.4.2. Remark (validity of Makehams formula). Note that Makehams formula is


valid only when
(a) g, and R are constant throughout the term of the loan; and
(b) n is an integer multiple of 1/p.
Makehams formula remains true when the loan is repayable by instalments, provided
that the coupon rate D, the rate of income tax and the redemption price R per
unit nominal remain constant in time. Extension of this formula to more general
situations is possible [MCS86].

36

MTA (G5078) Autumn 2009 Lecture Notes

Derive Makehams formula for the above case.


3.4.3. Makehams formula established by general reasoning. Consider a
second loan of the same total nominal amount N as the loan described above.
Suppose that, as before, interest is payable p-thly in arrear. But in the second loan,
suppose that the redemption will happen in phases: namely, a nominal amount Nj
will be redeemed at time nj , for j = 1, . . . , m, at the price of R per unit nominal.
The capital repayments for this second loan are thus identical to those of the original
loan in total, except they happen in instalments.
Suppose that for this second loan the net annual rate of interest per unit of redemption price is i(p) . The total indebtedness (i.e., capital to be repaid) for either loan
is C = N R.
Since, by hypothesis, the net annual rate of interest payment per unit indebtedness
for the second loan is i(p) , the value at rate i of this loan is clearly C.
Let K be the value of the capital payments of this second loan. (Of course, K is
also the value of the capital payments of the first loan.) Then the value of the net
interest payments for the second loan must be (C K).
The difference between the two loans lies simply in the rate of payment of net
interest. The net annual rate of interest per unit redemption price is g(1 ) for
the original loan and i(p) for the second loan. By proportion, therefore, the value of
the net interest payments for the first loan is g(1 )/i(p) times the value of the net
interest payments for the second loan, i.e.,
g(1 )
(C K).
(3.4.4)
i(p)
The value of the first loan, being the value of the capital plus the value of the net
interest, is thus
g(1 )
K+
(C K).
(3.4.5)
i(p)
A good grasp of proportionality arguments, as the one above, can simplify the
solution of many problems.
3.4.4. Example.
Problem. A loan of 75000 is to be issued bearing interest at the rate of 8% per
annum payable quarterly in arrear. The loan will be repaid at par in 15 equal annual
instalments, the first instalment being repaid five years after the issue date.
Find the price to be paid on the issue date by a purchaser of the whole loan who
wishes to realize a yield of (a) 10% per annum effective, and (b) 10% per annum
convertible half-yearly. (Ignore taxation.)
Solution. The capital repayments are each of amount 5000. The first repayment
is after five years and the final repayment is after 19 years.
(a) Choose one year as the basic unit of time. The required yield per unit time is
10% so i = 0.10. Using the notation above, we have C = 75000 (since redemption
is at par). The value of the capital repayment is
K = 5000(a19e a4e ) at 10% = 25975.27.
Note that, since redemption is at par, g = 0.08 and interest is paid quarterly
(i.e., four times per time unit) so p = 4. From Makehams formula we obtain the
required price as
0.08
(75000 25975.27) = 66636.60
25975.27 +
0.10(4)

MTA (G5078) Autumn 2009 3.4. Makehams formula

37

Since 66636.60/75000 = 0.8885, this price may be quoted as 88.85 for 100
nominal.
(b) Choose six months as the basic unit of time. The required yield per unit time
is 5%. Thus i = 0.05. Note now that interest is paid twice per time unit, so in the
notation above p = 2. Also, per time unit the amount of interest payable is 4% of
the outstanding loan, so now we have g = 0.04. The capital repayments occur at
times 10, 12, 14, ..., 38, so
5000
K=
(a40e a10e ) at 5% = 25377.27.
a2e
Hence the value of the entire loan is
0.04
25377.27 +
(75000 25377.27) = 65565.63
0.05(2)
or 87.42 for 100 nominal.
3.4.5. Example.
Problem. In relation to the loan described in Problem 3.4.4, find the price to be
paid on the issue date by a purchaser of the entire loan who is liable to income tax
at the rate of 40% and wishes to realise a net yield of 7% per annum effective.
Solution. The capital payments have value
K = 5000(a19e a4e ) at 7% = 34741.92.
Hence the price to provide a net yield of 7% per annum effective is
0.08(1 0.4)
(75000 34741.92) = 63061.89,
0.07(4)
i.e., after dividing the result by 100C, 84.08 for 100 nominal.
34741.92 +

3.4.6. Example.
Problem. A loan of nominal amount 100 000 is redeemable at 105% in four equal
instalments at the end of 5, 10, 15, and 20 years. The loan bears interest at the rate
of 5% p.a. payable half-yearly.
An investor, liable to income tax at the rate of 30%, purchased the entire loan on
the issue date at a price to obtain a net yield of 8% pa effective. What price did he
pay?
Solution. Note that the total indebtedness, C, is 100000 1.05 = 105000.
Each year the total interest payable is 5% of the outstanding nominal loan, so
that the interest payable each year is g times the outstanding indebtedness, where
g = 0.05/1.05.
Choose one year as the unit of time, then i = 0.08 and at the issue date the capital
payments have value
K = 25000 1.05(v 5 + v 10 + v 15 + v 20 ) at 8% = 43931.12.
Using the value of g described above, we obtain the price paid by the investor as
43, 931.12 +

0.05 (1 0.3)
(105, 000 43, 931.12)
1.05 0.08(2)

0.05 0.7
(105, 000 43, 931.12) = 69, 875.92
1.05 0.07846
which is equivalent to about 70 for 100 nominal.
= 43, 931.12 +

38

MTA (G5078) Autumn 2009 Lecture Notes

3.4.7. Exercise. A loan of nominal amount 1, 000, 000 is to be issued bearing


interest of 10% p.a. payable half-yearly. At the end of each year part of the loan
will be redeemed at 105%, The nominal amount redeemed at the end of the first
year will be 1, 000 and each year thereafter the nominal amount redeemed will
increase by 10, 000 until the loan is finally repaid. The issue price of the loan is
98.80 for 100 nominal.
Find the net effective annual yield to an investor liable to income tax at 40%, who
purchases the entire loan on the issue date.
3.4.8. Exercise. 5 years ago a loan was issued bearing interest payable annually
in arrear at the rate of 8% per annum. The terms of issue provided that the loan
would be repaid by a level annuity of 1 000 over 25 years.
An annuity payment has just been made and an investor is considering the purchase
of the remaining instalments. The investor will be liable to income tax at the rate
of 40% on the interest content (according to the original loan schedule) of each
payment. What price should the investor pay to obtain a net yield of 10% per
annum effective?
3.5. The effect of the term to redemption on the yield
Consider a loan of nominal amount N which has interest payable p-thly at the annual
rate of D per unit nominal. Suppose that the loan is redeemable after n years at a
price of R per unit nominal. An investor, liable to income tax at rate , wishes to
purchase the loan at a price to obtain a net effective annual yield of i.
3.5.1. Three useful relations. As before, let g = D/R and C = N R, so that
gC = DN . The price to be paid by the investor, as a function of n an i when all
the other parameters are kept fixed, is given by the relation
(p)

i) :=(1 )DN a + Cv n
A = A(n,
ne


(p)
(p)
=(1 )Gane + C 1 i(p) ane

(3.5.1)

(p)

=C + [(1 )g i(p) ]Cane

at rate i. The first two relations are obtained from the definition of the price A
and Makehams formula, respectively. The last relation can be obtained by general
reasoning: if the net annual rate of interest per unit indebtedness were i(p) , the value
of the loan would be C. In fact the net annual rate of interest per unit indebtedness
is (1 )g, so the second term in the right-hand side of the last equation is the value
of net interest in excess of the rate i(p) .
The following are immediate conse3.5.2. Behaviour of the price function A.
quences of equations in (3.5.1):
i) = C, i.e., the function A(,
i)
(a) If i(p) = (1 )g, then, for any value of n, A(n,
is a constant equal to the capital at redemption C.
i) is increasing.
(b) If i(p) < (1 )g, then, the function A(,
i) is decreasing.
(c) If i(p) > (1 )g, then, the function A(,
) is decreasing.
(d) For any fixed n, the function A(n,
3.5.3. Term to redemption variation and comparison. Consider now two
loans, each of which is as described in the first paragraph of this section except that
the first loan is redeemable after n1 years and the second loan after n2 years, where

MTA (G5078) Autumn 2009 3.5. The effect of the term to redemption
on the yield
39
n1 < n2 . Suppose that an investor, liable to income tax at a fixed rate, wishes to
purchase one of the loans for a price A. Then
(a) If A < C (where C = N R), the investor will obtain a higher net yield by
purchasing the first loan (i.e., the loan which is repaid earlier).
(b) If A > C, the investor will obtain a higher net yield by purchasing the second
loan (i.e., the loan which is repaid later).
(c) If A = C, the net yield will be the same for either loan.
3.5.4. Exercise. Give an intuitive explanation to the above conclusion.
3.5.5. Partial redemption by instalments. When an investor purchases part
of a loan redeemable by instalments, the yield he will obtain depends on the actual
date (or dates) at which his holding is chosen for redemption. In relation to such
a loan, issued in bonds of equal nominal amount, suppose that a nominal amount
Nr is redeemable at time nr t = 1, . . . , k where n1 < n2 < < nk . Suppose that
(for each bond) the purchase and redemption prices per unit nominal are P and R
respectively.
Consider a purchaser of one bond, subject to income tax at rate , and let the net
yield per annum which he will obtain if his bond is redeemed at time n, be denoted
by ir , r = 1, . . . , k.
If the bonds redeemed at anyone time are drawn by lot, the probability of obtaining
a particular yield ir is equal to
Nr
pr = Pk
r=1

Nr

and the expected value of the yield, in the probabilistic sense, is therefore
Pk
k
X
Nr ir

pr ir = Pr=1
.
i =
k
r=1 Nr
r=1
It should be noted that this quantity is not in general equal to the net yield i on
the whole loan, but in most practical cases i and i will be quite close to each other.
It is also clear that both i and i will lie somewhere between i1 and ik .
3.5.6. Problem (partial redemption). A loan of nominal amount 80, 000 is redeemable at 105% in four equal instalments at the end of 5, 10, 15, and 20 years.
The loan bears interest at the rate of 10% p.a. payable half-yearly.
Suppose that an investor, who is subject to income tax at 30%, purchases one bond
of 100 nominal on the issue date for 95.82. Find the net yield per annum he will
obtain, assuming redemption after 5, 10, 15, and 20 years, and plot these net yields
on a graph. Find also the probability that the net yield will exceed 9% p.a.. Find
the expected value of his yield and show that it is not identical with the net yield
which he would obtain if he purchased the entire issue at the same price.
3.5.7. Problem (partial redemption). A loan of nominal amount 100 000 is to
be issued in bonds of nominal amount 100 bearing interest of 6% per annum
payable half-yearly in arrear. The loan will be repaid over 20 years, 50 bonds being
redeemed at the end of each year at a price of 120 per 100 nominal (also denoted
by 120%). The bonds redeemed in anyone year will be drawn by lot (so the buyer
of one particular bond ignores its redemption time but knows the probabilities of it
being redeemed in a certain year). The issue price of the loan is 94.32%.
An investor, liable to income tax at the rate of 25% is considering the purchase of
all or part of the loan.

40

MTA (G5078) Autumn 2009 Lecture Notes

(a) Show that if he purchases the entire loan, his net annual yield on the transaction
will be 7%.
(b) Show that if he purchases only one bond his net annual yield could be as high
as 32.36% or as low as 5.61% and find the probability that he will achieve a net
annual yield of (i) at least 8% and (ii) between 6% and 8%.
3.5.8. Exercise. A loan of nominal amount 1000 is to be issued in bonds of
nominal amount 10 bearing interest of 4% per annum payable quarterly in arrear.
The loan will be repaid over 10 years, 10 bonds being redeemed at the end of each
year at a price of 110 per 100 nominal. The bonds redeemed in anyone year will
be drawn by lot. The issue price of the loan is 90%.
An investor, liable to income tax at the rate of 20% is considering the purchase of
all or part of the loan.
(a) Calculate his net annual yield if he purchases the entire loan at the outset.
(b) If he purchases only one bond, find the probability that he will achieve a net
annual yield of (i) at least 6% and (ii) between 6% and 10%.
3.6. Real returns, inflation and index-linked stocks
3.6.1. Price indexes and inflation. Inflation may be defined as a fall in the
purchasing power of money. It is usually measured with reference to an index
representing the cost of certain goods and (perhaps) services. For example, in the UK
the index used most frequently is the Retail Prices Index (RPI), which is calculated
monthly by the Central Statistical Office.
Real investment returns, as opposed to the money (or cash) returns we have so far
considered, take into account changes in the value of money, as measured by the
RPI or another such index. It is possible for all calculations relating to discounted
cash flow, yields on investments, etc. to be carried out using units of real purchasing
power rather than units of ordinary currency.
A price index is a number that quantifies the amount of currency needed to purchase
one unit of average goods. This number is relative to some fixed time t0 where we
consider the index to be 100% (i.e., 1 unit of good costs 1 unit of currency). Having
fixed t0 , let us define Q(t) to be price of one unit of goods at time t.
For example, the following table describes the RPI
Calendar year 1980 1981 1982 1983
Value of RPI for January 245.3 277.3 310.6 325.9
Annual inflation, calculated over a period of 12 months, at time t is defined as the
rate (expressed in percentile) in the price index increase over a year, i.e., that number
j(t) such that
Q(t)(1 + j) = Q(t 1),
(3.6.1)
for each fixed t and when the time unit is a year. So in relation to the example we
have that the annual inflation in Jan 1981, with respect to 12 months earlier, is of
j=

Q(1)
277.3
1=
1 = 0.1304 = 13.04%.
Q(0)
245.3

(3.6.2)

As an exercise calculate the annual inflation in 1982 and 1983, with respect to 12
months earlier.
Note the relative nature of the price index Q: changing the unit of goods will
change the values of Q, but not their ratios. For example, an equally good table as
the one above is given by

MTA (G5078) Autumn 2009 3.6. Real returns, inflation and


index-linked stocks

41

Calendar year 1980 1981 1982 1983


Value of RPI for January 88.5 100.0 112.0 117.5
Although inflation is almost universally expressed as annual, it is possible to calculate annual inflation, over periods smaller (or bigger) than 12 months.
3.6.2. Essay (annual inflation over a period shorter than a year). Define the annual
inflation at time t, calculated over a period h in years (e.g., h = 1/12), to be the
number jh such that
Q(t) = Q(t h)(1 + jh (t))h .
(3.6.3)
Define the nominal inflation at time t, calculated over the period h to be the number
lh such that
Q(t) = Q(t h)(1 + hlh (t)) .
(3.6.4)
(a) Fix h > 0 and suppose Q(t) is measured at regular intervals of time
t0 < t1 := t0 + h < < tr := t0 + hr < < tn := t0 + hn.

(3.6.5)

Find an expression of jh (t) and lh (t) in terms of Q and h, for t = t1 , . . . , tn .


(b) Still fixing h, what can you say about jh (tr ) and j, when jh (tr ) is constant with
respect to tr ? What kind of function of time is Q(tr )?
(c) Suppose h is not fixed and let Q(t) be measured at each time t (not only at
integer multiple of h intervals). Define (t) = limh0 lh (t) and show that this limit
exists if Q(t) is a differentiable function of t.
Hint. Show that (t) = d log Q(t)/ dt.
(d) Define j(t) = limh0 jh (t) and show, assuming that Q is differentiable, that the
limit must exist.
(e) Find a relationship between (t) and j(t).
3.6.3. Evaluating a cash flow with inflation. Suppose now that a transaction
involves cash flows c1 , c2 , . . . , cn , the r-th cash flow occurring at time tr . (Note that
the cash flows are monetary amounts.) If the appropriate index has value Q(t) at
a generic time t, the cash flow cr at time tr will purchase cr /Q(tr ) units of the
index. By the real yield on the transaction we mean the yield calculated on the
basis that the investors receipts and outlays are measured in units of index (rather
than monetary units). The real internal rate of return (or yield) on the transaction,
measured in relation to the index Q, is thus that value of i for which
n
X
cr
(1 + i)tr = 0.
(3.6.6)
Q(tr )
r=1
For a fixed k = 1, . . . , n, this equation is equivalent to
n
X
Q(tk )
cr
(1 + i)tr = 0.
Q(tr )
r=1

(3.6.7)

This is the equation of value for the transaction, measured in units of purchasing
power at a particular time tk . It means that the values of Q can be taken as relative
values against any arbitrarily fixed reference purchasing power (namely, Q(tk )).
3.6.4. Problem (accounting for inflation). On 16 January 1980 a bank lent 25, 000
to a businessman. The loan was repayable three years later, and interest was payable
annually in arrear at 10% per annum. Ignoring taxation and assuming that the RPI
for any month relates to the middle of that month, find the real annual rate of

42

MTA (G5078) Autumn 2009 Lecture Notes

return, or yield, on this transaction. Values of the RPI for the relevant months are
summarised in the following table.
Calendar year 1980 1981 1982 1983
Value of RPI for January 245.3 277.3 310.6 325.9
Exercises and problems on Application of interest rate
Problem 3.1 (fixed income investment: priceyield relation). A loan of nominal
amount 550 000 is to be issued bearing interest of 10% pa payable half-yearly. At
the end of each year part of the loan will be redeemed at 105%, The nominal amount
redeemed at the end of the first year will be 10 000 and each year thereafter the
nominal amount redeemed will increase by 10 000 until the loan is finally repaid.
The issue price of the loan is 90 for 100 nominal.
Find the net effective annual yield to an investor liable to income tax at 40%, who
purchases the entire loan on the issue date.
Problem 3.2 (fixed income investment: priceyield relation). 5 years ago a loan
was issued bearing interest payable annually in arrear at the rate of 8% per annum.
The terms of issue provided that the loan would be repaid by a level annuity of
1 000 over 25 years.
An annuity payment has just been made and an investor is considering the purchase
of the remaining instalments. The investor will be liable to income tax at the rate
of 40% on the interest content (according to the original loan schedule) of each
payment. What price should the investor pay to obtain a net yield of 10% per
annum effective?
Problem 3.3 (Makehams formula). A loan of nominal amount 1, 000, 000 is to
be issued bearing interest of 10% p.a. payable half-yearly. At the end of each year
part of the loan will be redeemed at 105%, The nominal amount redeemed at the
end of the first year will be 1, 000 and each year thereafter the nominal amount
redeemed will increase by 10, 000 until the loan is finally repaid. The issue price
of the loan is 98.80 for 100 nominal.
Find the net effective annual yield to an investor liable to income tax at 40%, who
purchases the entire loan on the issue date.
Problem 3.4 (partial redemption). A loan of nominal amount 80, 000 is redeemable at 105% in four equal instalments at the end of 5, 10, 15, and 20 years.
The loan bears interest at the rate of 10% p.a. payable half-yearly.
Suppose that an investor, who is subject to income tax at 30%, purchases one bond
of 100 nominal on the issue date for 95.82. Find the net yield per annum he will
obtain, assuming redemption after 5, 10, 15, and 20 years, and plot these net yields
on a graph. Find also the probability that the net yield will exceed 9% p.a.. Find
the expected value of his yield and show that it is not identical with the net yield
which he would obtain if he purchased the entire issue at the same price.
Problem 3.5 (partial redemption). A loan of nominal amount 100 000 is to
be issued in bonds of nominal amount 100 bearing interest of 6% per annum
payable half-yearly in arrear. The loan will be repaid over 20 years, 50 bonds being
redeemed at the end of each year at a price of 120 per 100 nominal (also denoted
by 120%). The bonds redeemed in anyone year will be drawn by lot (so the buyer
of one particular bond ignores its redemption time but knows the probabilities of it
being redeemed in a certain year). The issue price of the loan is 94.32%.
An investor, liable to income tax at the rate of 25% is considering the purchase of
all or part of the loan.

MTA (G5078) Autumn 2009 3.8. Exercises and problems on


Application of interest rate

43

(a) Show that if he purchases the entire loan, his net annual yield on the transaction
will be 7%.
(b) Show that if he purchases only one bond his net annual yield could be as high
as 32.36% or as low as 5.61% and find the probability that he will achieve a net
annual yield of (i) at least 8% and (ii) between 6% and 8%.
Problem 3.6. A loan of nominal amount 1000 is to be issued in bonds of nominal
amount 10 bearing interest of 4% per annum payable quarterly in arrear. The loan
will be repaid over 10 years, 10 bonds being redeemed at the end of each year at a
price of 110 per 100 nominal. The bonds redeemed in anyone year will be drawn
by lot. The issue price of the loan is 90%.
An investor, liable to income tax at the rate of 20% is considering the purchase of
all or part of the loan.
(a) Calculate his net annual yield if he purchases the entire loan at the outset.
(b) If he purchases only one bond, find the probability that he will achieve a net
annual yield of (i) at least 6% and (ii) between 6% and 10%.
Problem 3.7 (accounting for inflation). On 16 January 1980 a bank lent 25, 000
to a businessman. The loan was repayable three years later, and interest was payable
annually in arrear at 10% per annum. Ignoring taxation and assuming that the RPI
for any month relates to the middle of that month, find the real annual rate of
return, or yield, on this transaction. Values of the RPI for the relevant months are
summarised in the following table.
Calendar year 1980 1981 1982 1983
Value of RPI for January 245.3 277.3 310.6 325.9
Problem 3.8 (inflation). On 1 January 1970 a bank lent 1, 000 to a businessman.
The loan was repayable three years later, and interest was payable annually in arrear
at 5% per annum. Ignoring taxation and assuming that the RPI for any month
relates to the middle of that month, find the real annual rate of return, or yield,
on this transaction. Values of the RPI for the relevant months are reported in the
following table.
Calendar year 1970 1971 1972 1973
Value of RPI for January 225.3 257.3 290.6 305.9

CHAPTER 4

Simple market model


The lecture notes are essentially taken from the first chapter in the book of Capi
nski
& Zastawniak [CZ03], to which we refer the reader for more details.
4.1. Building a simple market model
4.1.1. Assets. In financial jargon a market is a (virtual) place where one can buy,
sell, or trade, assets. An asset is anything that has an identifiable value, e.g., a
house, a company (or a share of it), a bicycle, a plot of land, etc. Like ice-cream,
assets come in a rich variety of flavours, but they can be categorized in two main
asset types (or submarkets):
Risk-free assets: for example, bonds. As we shall see risk free is a misnomer,
but it does convey the idea that a risk-free asset is more secure in time. In fact,
some risk-free assets are also called securities.
A risk-free asset is characterised, by a guaranteed return on the price, i.e.,
if you buy some risk-free asset, say a bond, today for 100 you and the issuer
of the bond sign a contract which guarantees that in a certain time, say a year,
the issuer has the obligation of redeeming it from you at a higher price, say
107. In Chapter 3 we have seen many examples of real-life securities; but
here we will deal with a much simpler model which encapsulates all the main
properties of a security.
Risky assets: for example, stocks. A stock is a share of a company. If the
company is doing good business the stock will increase in value, but if the
company underperformsand this cannot be predicted with certainty, hence
the adjective riskyit may decrease in value. In some sectors, risky assets
are also called equities.
Equities are harder to represent mathematically than securities, but Probability Theory (see Appendix B) comes to help with random variables (also
known as random numbers). Note that random variable, as a category, includes all deterministic (nonrandom) variables; in fact, a deterministic variable (e.g., the redemption price of a bond in a years time above) can be seen
as a random variable that has 100% chance of having a certain value (e.g.,
107). An example of genuinely (also known as uncertain) random variable is,
for example, and in a very simplified setting, the price of stock S , of price
1.5 today, in a years time which could be 1.8 if the share goes up, say with
60% chance and 1.1, with 40%. Such a random variable, that takes only 2
possible values, is called a binomial random variable. Clearly, a binomial random variable is a very simplistic model for the future price of a stock, which
will more realistically range across a theoretically infinite range of values with
a probablity density, rather than a discrete valued one. But realism comes at
the cost of complicating understanding, so we will stick to binomial variables
in this chapter.
4.1.2. A two-asset model. To make it as simple as possible, without loosing
meaning altogether, let us assume that in our market model there are only two

46

MTA (G5078) Autumn 2009 Lecture Notes


Table 1. The two-by-two (asset and time), or one-step binomial, or
simple market, model.
asset

time 0

S(0) = certain

B(0) = certain

time 1
(
S u with chance p
S(1) =
S d with chance (1 p)
B(1) = certain

assets: a risk-free one, which we call B (for bond) and a risky one which we call S
(for stock).
4.1.3. A two-instant model. To get to a reasonable model, we need to add time.
As we have learned in previous chapters, time can be modelled in two ways: continuous and discrete. The simplest form of discrete time can be modelled with a sequence
of equally spaced time instants, usually normalised to 1 time-unit, i.e., 0, 1, 2, . . . , N
where N is some final time for the model. Such models entail the knowledge (or the
pretension thereof) of the asset prices at all instants. By knowledge we mean a
stochastic type of knowledge for the random variables, which means knowledge of
the chances rather than the outcomes.
To make things the simplest possible we will use only 2 times, i.e., take N = 1
above. So we have an initial time 0 and a final time 1. Think of these as the
beginning of the year and the end of the year. This is the so-called two-instant
(or one-step) model, and, after combining it with a two-asset model it becomes
the two-by-two market model, also called simple market model. Finally, we will
consider the uncertain asset (i.e., the stock) to have a binomial price at the end
time; this leads to the so-called one-step binomial market model. We can summarise
the situation in Table 1.
4.1.4. Definition (return on an asset). An important financial quantity (maybe
the most important one) is the return on a given asset over a given time interval.
In our simple market model we have two returns:
B(1) B(0)
S(1) S(0)
KB :=
and KS :=
.
(4.1.1)
B(0)
S(0)
Note that while KB is certain, KS is uncertain (i.e., genuinely random).
4.1.5. Example (simple market). Consider the following simple market
asset time 0
time 1
(
125 with chance 45%
S
S(0) = 100 S(1) =
95
with chance 55%
B
B(0) = 100 B(1) = 110
The return on B is easy,
110 100
KB :=
= 10%,
(4.1.2)
100
whereas the return on S takes just a bit more math (and we use vector notation to
compress the repetitive calculations)
 
125



100 
95
25%
45%
KS :=
=
with chance
.
(4.1.3)
5%
55%
100

MTA (G5078) Autumn 2009 4.2. Building a simple market model

47

4.2. Basic assumptions


We now set to make some blanket1 structural assumptions on the (two-by-two)
simple market model.
4.2.1. Hypothesis (Positivity of prices). An asset cannot have 0 or negative price.
A zero price is useless, whereas a negative price becomes positive by inverting the
role of buyer and seller (for example, debt is an asset, which is negative in nature
and traded by financial institutions, but the roles are exchanged so that debt is
effectively traded as credit).
In our simple market, positivty of prices means
 u  
S
0
S(0), B(0), B(1) > 0 and S(1) = d >
.
(4.2.1)
0
S
4.2.2. Hypothesis (Uncertainty of stock prices). We assume that stock prices
have at least two possible outcomes, each with positive probability. In the two-bytwo model context, with reference to the notation of Table 1, this means that the
chance of the market going up has to satisfy
0 < p < 1.

(4.2.2)

This is, of course, equivalent to 0 < 1 p < 1, for those who have a glass half-empty
view of life.
4.2.3. Definition (portfolio). A portfolio is a certain quantity of the possible assets that we (or the investor, if you prefer) holds. Mathematically a portfolio is
|
arranged in a (column) vector, say w = x, y , where x is the quantity of S and y
is the quantity of B held (portfolio means wallet in rennaissance Italian, whence
the symbol w).
4.2.4. Hypothesis (Divisiblity, liquidity and position of assets). We assume that
assets can be held and traded in arbitrary quantities. Mathematically, this means
x, y (and any variation thereof) belongs to R. Negative values of x or y mean that we
(investor) are short on (also known as in a short position), i.e., we owe to someone,
the asset S or B, respectively, while postive values mean that we are long on (also
known as in a long position), in the sense that we actually possess, the asset S or
B, respectively.
4.2.5. Definition (value of a portfolio and market matrix).
| Suppose we hold a
|
portfolio w = x, y in a one-step binomial market S , B , then the value of w
at time t {0, 1} is defined as
 

 x
V (t) := xS(t) + yB(t) = S(t), B(t)
= [M ]t w,
(4.2.3)
y
where we use the market vector [M ]t , as being the t-th row of the market matrix


S(0) B(0)
M=
,
(4.2.4)
S(1) B(1)
which is a random matrix because the entry S(1) is random.
4.2.6. Hypothesis (admissible porfolio and solvency). Given a simple market with
matrix M and a portfolio w, we say that w is an admissible portfolio for M if and
only if
V (t) = [M ]t w > 0
1Our

t {0, 1} , with probability 100%.

assumptions hold for the whole chapter.

(4.2.5)

48

MTA (G5078) Autumn 2009 Lecture Notes

If we have an admissible portfolio, we say that we are solvent. This means that we
are able to close any short position in the portfolio (by using the long ones). If one
holds a portfolio for which V (t) 0 for some t and positive probability, we say that
the investor is insolvent or bankrupt.
4.2.7. Exercise. Consider the simple market
asset time 0
time 1
(
52 with chance p
S
S(0) = 50 S(1) =
48 with chance 1 p
B
B(0) = 100 B(1) = 110
.
(1) Calculate the returns KB and KS . 
|
(2) Consider the portfolio w = 20, 10 , and calculate its value, as an algebraic
expression, at the generic time t.
(3) We define the (total) return on the portfolio w as
V (1) V (0)
KV :=
.
(4.2.6)
V (0)
Calculate KV .
(4) Is it true that
KV = xKS + yKB ?
(4.2.7)
(5) Show that
S(0)
.
(4.2.8)
KV = 0 KS + (1 0 )KB , where 0 = x
V (0)
(6) Deduce that (4.2.7) is generally wrong, but the following is true
min {KS , KB } KV max {KS , KB } .

(4.2.9)

4.3. No-Arbitrage Principle (NAP)


Arbitrage , from French for refereeing or influencing, in finance means, roughly
speaking the ability to create a positive profit from no (zero) investement. Mathematically this amounts to infinite return, which is not possible in any reasonable
economy. Indeed, suppose someone did have the opportunity to make profit without investing anything, then as soon as they start doing it, others will follow suit
and everybody would end up making a profit, which is clearly absurd. If someone
makes money from nothing somewhere, someone must be putting money for nothing
somewhere else.
4.3.1. Example (an arbitrage opportunity). Mr X in New York buys Euros at the
rate 1.42$/. Ms Y in Paris sells Euros at the rate 1.40$/.
Clearly the difference in rates must give us the opportunity of making a profit. The
idea is to follow the common sense and buy cheap and sell expensive. One Euro is
clearly cheaper in Paris than in New York, so we may buy 1000 from Y, promising
to pay her $1400 for them, and then immediately sell this to X in New York who
hands us $1420. After settling our $1400 debt with Y, we are thus left with $20
profit, which we have pulled from no investment at all.
4.3.2. Example (another arbitrage opportunity). Suppose you are offered to exchange GBP in USD for $1.58 a , in one years time (i.e., at time 1), while the
exchange rate now (i.e., at time 0) is 1.60$/. Furthermore, suppose the interest
rates are
i$ = 4% and j = 6%,
(4.3.1)

MTA (G5078) Autumn 2009 4.4. No-Arbitrage Principle (NAP)

49

for borrowing $s and investing respectively. Then there is an arbitrage opportunity.


Indeed, we could borrow an amount in USD at i$ = 4%, say $16 000 and buy 10 000
with it. Investing those 10 000 at j we reap 10 600 after one year. Then we take
advantage of the offer to exchange at $1.58 and repay the USD debt:
10 600 1.58 16 000 1.04 = 108.

(4.3.2)

We have thus produced $108 out of nought: another arbitrage opportunity.


4.3.3. Problem (an arbitrage opportunity?). On 19 July 2002 the exchange rates
were as follows
NYSE buys
sells
LSE buys
sells
1
$1.0202 $1.0284
1 0.6324 0.6401
1
$1.5718 $1.5844
$1
0.6299 0.6375
Can you spot an arbitrage opportunity?
4.3.4. Remark (spotting arbitrage opportunities in a simple market). So far, we
have given only a vague (though effective) definition of arbitrage. To make it more
rigorous, we need to make some reasonable argumentations. As it can be seen from
Problem 4.3.3, it is generally not very easy to spot an arbitrage opportunity, but in
a simple market, it is possible to derive a simple rule. In fact, to create money from
nothing means that there must be a way to form a portfolio w with no money at
time 0 (i.e., whose total value V (0) is 0), which yields a postive gain at time 1 (i.e.,
V (1) > 0). This prompts the following definition.
4.3.5. Definition (arbitrage opportunity in a simple market). A simple market
with assets S and B as in Table 1, is said to have an arbitrage opportunity if and
only if there exists an admissible portfolio w such that V (0) = 0 and V (1) 0 with
V (1) > 0 with some positive probability.
4.3.6. Exercise (simple market with arbitrage). Consider the binomial simple market given by
asset time 0
time 1
(
125 with chance p
S
S(0) = 100 S(1) =
110 with chance 1 p
B
B(0) = 100 B(1) = 105
Find an arbitrage opportunity.
4.3.7. Hypothesis (No-Arbitrage Principle (NAP)). We shall assume that the
simple market satisfies the No-Arbitrage Principle (NAP), i.e., it has no arbitrage
opportunity, or, equivalently, there is no admissible portfolio w for which 0 = V (0)
V (1) and V (1) > 0 with non-zero chance.
4.4. One-step binomial model
Under the assumptions listed in 4.2 and NAP, given in 4.3.7, we will now derive
a series of results concerning a binomial simple market.
4.4.1. Remark (simple market with arbitrage). In Exercise 4.3.6, we notice that
B(1) = 105 is smaller than S d = 110, while B(0) = S(0). This is essentially what
allows for the arbitrage opportunity to arise: in fact, no one in their mind would
ever invest in B in such a scenario, because S gives you a higher return, no matter

50

MTA (G5078) Autumn 2009 Lecture Notes

what. In other words the bond in Exercise 4.3.6 is dearer2 than the stock. Indeed,
we will now prove this in a more general setting.
4.4.2. Theorem (intermediate bond-price property). Consider a binomial simple
market that operates under the blanket assumptions 4.2 and the NAP 4.3.7. If
B(0) = S(0)3 then
S d < B(1) < S u .
(4.4.1)
Proof Without loss of generality let us assume that B(0) = 100, hence S(0) = 100.
Suppose that B(1) S d , then the buy-cheap-sell-expensive and the fact that B(1)
is cheap in the future, makes it expensive. Let us go short on B then for starters:
borrow 1 of B at time 0 (i.e., set y = 1), and use the 100 to buy S (i.e., set
x = 1). With this portfolio we have V (0) = 0. Furthermore at time t = 1 we obtain
 u
   u
  
S B(1)
S B(1)
1
0
V (1) = d
= d

,
(4.4.2)
0
S B(1) 1
S B(1)

[]:

Check!

thus V (1) 0 surely and V (1) > 0 with some positive probability, which amounts
to an arbitrage opportunity.
If we suppose B(1) S u then it is enough to invert the roles, x = 1 and y = 1 to
find an arbitrage opportunity.[] Therefore a no-arbitrage assumption implies that
S d < B(1) and B(1) < S u .

4.4.3. Corollary (intermediate bond-return property). Consider a binomial simple
market that operates under the blanket assumptions 4.2 and the NAP 4.3.7..
Then, using the returns defined in 4.1.4, we have
min KS < KB < max KS .

(4.4.3)

where, recalling that KS is a binomial random variable, we put


min KS = KS d :=

S d S(0)
S u S(0)
and max KS = KS u :=
.
S(0)
S(0)

(4.4.4)

Proof left as exercise Extend the Intermediate Bond-Price Theorem 4.4.2 to the
case where B(0) 6= S(0) and then take the returns.
Hint. Normalise, apply the theorem and then denormalise back.

4.4.4. Risk and return (yield). Consider the binomial simple market given by
asset time 0
time 1
(
100 with chance 80%
S
S(0) = 80 S(1) =
60
with chance 20%
B
B(0) = 100 B(1) = 110
and suppose you have 10 000 to be invested and you decide to go with a portfolio
of
x = 50 of S and y = 60 of B.
(4.4.5)
(Check that V (0) = 10 000.) Then we have

  



100 110 50
11600
80%
V (1) =
=
with chance
.
(4.4.6)
60 110 60
9600
20%
2We

use dear here and below as opposite of cheap.


loss of generality one may assume that B(0) = S(0) = 100 by rescaling appropriately
the units of x and y.
3Without

MTA (G5078) Autumn 2009 4.3. One-step binomial model

51

The portfolios return is then



 



V (1) V (0)
16%
80%
4 1600
=
with chance
.
= 10
KV =
400
4%
20%
V (0)
The expected return can be calculated as the expectation4 of KV , i.e.,
Z
E KV =
KV ()P ( d) = KV u P u + KV d P d




 16%
= 80% 20%
= (12.8 0.8)% = 12%.
4%

(4.4.7)

(4.4.8)

This give us the expected return, but it does not tell us how much we are risking.
In fact, though small, the probability of making a loss on S is not negligible. The
risk can be quantified by using the variance of the return KV or, more appropriately
in terms of units, the variances square root (also known as standard deviation). We
define the risk V associated to our portfolio as follows
p
V := var KV .
(4.4.9)
In our context, we evaluate


var KV = E (KV E KV )2 = E[KV ]2 (E KV )2
= KV2 u P u + KV2 d P d (0.12)2 = 64 104
and hence the risk associated with this portfolio is

V = 64 104 = 8 102 = 8%.

(4.4.10)

(4.4.11)

Exercises and problems on Simple market model


Exercise 4.1. Extend the Intermediate Bond-Price Theorem 4.4.2 to the case
where B(0) 6= S(0) and then take the returns.
Hint. Normalise, apply the theorem and then denormalise back.
Exercise 4.2. Suppose A is a fixed-income (risk-free) asset and S is a risky asset
with prices given as follows
asset time 0
time 1
(
30 with chance p
S
S(0) = 25 S(1) =
20 with chance 1 p
B
B(0) = 90 B(1) = 100
where 0 < p < 1.
(a) For a portfolio holding x = 10 units of S and y = 15 units of B, calculate
V (0), V (1) and KV .
(b) Find a portfolio whose value at time 1 is
(
1 160 if the stock rises,
V (1) =
(4.2.1)
1 040 if the stock declines.
What is the value of this portfolio at time 0.
the understated use of the probability space (, F , P ). In the financial context
is the space of all possible events that can influence the market. In the binomial model, it is
sufficient to think of as being partitioned into two disjoint events: u = {the market goes up}
and d = {the market goes down}
4Note

52

MTA (G5078) Autumn 2009 Lecture Notes

Exercise 4.3. Consider two assets, one risky and one risk-free, denoted S and
A respectively. Suppose their prices are
asset time 0
time 1
(
S u with chance p
S
S(0) = 34 S(1) =
S d with chance 1 p
B
B(0) = 100 B(1) = 112
Is it possible to find an arbitrage opportunity if the forward price of stocks is F =
38.60 with delivery date 1?
Exercise 4.4. Let the bond B and stock S prices be
asset time 0
time 1
(
120 with chance p
S
S(0) = 100 S(1) =
80
with chance 1 p
B
B(0) = 100 B(1) = 110
Price (i.e., find C(0) for) a call option with exercise time 1 and
(a) strike price F = 90,
(b) strike price F = 110.

CHAPTER 5

Arbitrage and pricing


In a standard consumption-investment problem, investors have two types of controls
available. In the first period, they invest wealth in investment assets after consumption. In the second period, they sell these assets to buy the various consumption
goods. Therefore, investment consists of a series of transfers.
5.1. Terminology and notation
5.1.1. Assets and states. An is a possible form of investment (e.g., stock of a
manufacturer, a building, shares in a telecom company, etc.). Each asset is considered to be in certain (e.g., the price of the manufacturers stock, the return on the
buildings rent, the telecoms company share value).
The number of states and assets is considered finite (though it may be very large).
States are indexed by s = 1, . . . , S and assets by i = 1, . . . , N .
The one period investment problem is characterized by the table Y , of end-of-period
prices on the N assets in the S states
|
Y = Y 1, . . . , Y S
(5.1.1)
where each row of Y , say the s-th row, can be explictly written as

Y s = Ys1 , . . . , Ysi , . . . , YsN ,

(5.1.2)

for s = 1, . . . , S.
5.1.2. Portfolio and realized
| return table. The investor chooses a represented
by the vector n = n1 , . . . , nN , where ni is the number of units (shares) asset held
for i = 1, . . . , N . The initial values of the assets are the components of a vector v,
and the is n| v = W0 .
The (1+ rate of return) on the i-th asset in the state s is Zsi = Ysi /vi . The state
space table of return is
Z = Y D 1
(5.1.3)
v
where D v is the diagonal matrix with i-th element vi and A1 indicates the inverse
matrix of a generic square matrix A. We assume that vi 6= 0 to ensure that D v is
invertible.
The i-th column of Z represents the (vector of) S possible returns on the asset i,
corresponding to all possible states ranging from 1 to S. The row vector z s | , which
will denote the s-th row of Z has N components, each of which represents the return
on each of the N assets, in a given state s = 1, . . . , S. The matrix Z is often referred
to as an .
e a random vector whose possible realizations are the (transposed)
We denote by z
rows of Z, z s . When we say we mean that each state s = 1, . . . , S has a certain
probability (which may be known or unknown to the investor) of occurring. In
e, denoted by zi , is a scalar-valued
particular, this means that each component of z
random variable defined on the {1, . . . , S}.

54

MTA (G5078) Autumn 2009 Lecture Notes

In the return space, investments are characterized by a vector , called the , where
each component is given by
i = ni vi .
(5.1.4)
If the investment involves a total net amount, called the total commitment,
|

W0 = n v =

N
X

i = C> 0,

(5.1.5)

i=1

then we can normalize into a vector w by setting wi = i /W0 .


In this case, the investment is a and w is the vector of . Note that vectors w,
bear the same relative (i.e., percentage) information as long as vi is non-zero (which
we assume to be true, unless otherwise stated).
5.1.3. Arbitrage. If the i-the weight happens to be negative (i.e., wi < 0), we
say that the asset has been sold short. If some of the assets have been sold short
(and some not) then it may occur that W0 = 0. In this case, it is not possible to
normalize . This motivates the following.
An arbitrage portfolio is defined as a vector of commitments summing up to 0.
Arbitrage portfolios will be denoted by . Though the sum of s components is 0,
we usually assume that 6= 0.1
In general financial terms, arbitrage is defined as the possibility of making a profit
without investing. For example, price differentials in certain goods (or currency
purchase power differences with respect to their market exchange rate) provide the
opportunity for arbitrage. As a rule of thumb, every situation that allows you to
obtain something out of nothing represents an arbitrage opportunity.
5.1.4. Exercise (short selling). Find the definition of short sale and some application of such activity.
5.1.5. Example. Consider the economy with two assets and three states:

1 3
(5.1.6)
Z = 2 1 .
3 2
|
An investment commitment of = 1, 3 has state-by-state outcomes of
|
Z = 10, 5, 9
(5.1.7)
|
and an original commitment of 4. The corresponding portfolio is w = 1/4, 3/4
|
with returns Zw = 2.5, 1.25, 2.25 .
5.1.6. Riskless portfolios and linear programming. To gain some insight into
how an arbitrage portfolio may be financed, we define a riskless portfolio to be a
portfolio w with the same return in any state, i.e., w satisfies
Zw = R1 and 1| w = 1,

(5.1.8)

for some constant R, called the riskless return. The constant R 1 defines the
risk-free rate.
We are assuming here that the return R is a constant with respect to s = 1, . . . , S,
but in general this may not be true. Indeed, for a given w it may be that the vector
Zw had non-constant entries.
1Make

sure you distinguish the latin letter double you w from the greek letter omega .

MTA (G5078) Autumn 2009 5.2. Terminology and notation

55

This observation prompts us to introduce the following returns which will be useful
later:


|
R := max
min Zw = max
min z s w ,
(5.1.9)
|
|
1 w=1

1 w=1 s=1,...,S

and


|
R := min
max Zw = max
min z s w
|
|
1 w=1

1 w=1 s=1,...,S

(5.1.10)

we may well get R 6= R. The minimax and maximin problems to be solved to obtain
these returns are known as linear programming problems.
5.1.7. Remark. In reality, the Arbitrage Principle in economics can be phrased as
follows:
There is never an opportunity to make a risk-free profit that gives a greater
return than that provided by the interest from a bank deposit.
A simple common-sense argument that supports this principle is that if there
were an opportunity to make a risk-free profit greater than that provided by a bank
deposit, then no one would be depositing in the bank. This would force the bank
to raise the interest rate (for example by investing in that risk-free profit scheme)
and the situation would end up being balanced. In other words, if the market is out
of equilibrium (because of an arbitrage opportunity) then it will be driven to the
equilibrium. This is called the arbitrage-free condition under which economists and
financial theorists operate.
In real life, arbitrageurs make profit by borrowing bank money to invest, they take on
risks to get better return. So they may well lose the money as well as making more.
The bottom line is that a successful arbitrageur must rely on extra-information (e.g.,
insider trading) to perform real arbitrage. Without such infomation, the probability
of winning or loosing is 50% each.
5.1.8. Exercise (can you spot arbitrage?). For each of the following, state whether
or not there is an arbitrage opportunity:
(a) An individual needs change and is offering you a 10 pound note in return for 9
pounds change.
(b) England football team is playing Croatia tonight, a friend says that he will pay
you 20 pounds if England loses and if you support Croatia during the game.
(c) Mr A offers to purchase a car for 12000 pounds, Mr B offers to buy the same
car for 15000 pounds.
(d) You and your friend are betting on the weather tomorrow. If it does not rain,
your friend will pay you 2 pounds, if it rains, you pay your friend 2 pounds.
(no, gamble 6= arbitrage)
5.2. Redundant Assets
5.2.1. Duplicable portfolios. Consider a given return table Z. Let w1 denote a
specific portfolio for this table. This portfolio is duplicable if and only if there exists
w2 6= w1 such that Zw2 = Zw1 .
Note that if there exists a pair of duplicable portfolios w1 6= w2 , then any vector w
is duplicable. Indeed, for x := w + (w1 w2 ), we have
Zx = Zw + Zw1 Zw2 = Zw.

(5.2.1)

What really matters thus, when identifying redundant assets, is their difference
w1 w2 , which constitutes an arbitrage portfolio. This prompts the following.

56

MTA (G5078) Autumn 2009 Lecture Notes

5.2.2. Definition (redundant assets). A redundant asset is one of the assets (a


column of Z) that contributes to the duplicability. Or equivalently, one or more of
the primitive assets in Z is redundant if there exists a nontrivial arbitrage portfolio
such that
Z = 0, 1| = 0, and 6= 0.
(5.2.2)
5.2.3. Exercise (redundant assets). Are any

1 1
Z=
1 2

of the assets in the economy



2
(5.2.3)
3

redundant?
5.2.4. Problem
(redudant assets and linear algebra). What can be said of the
 
Z
matrix | if one of the assets in Z is redundant?
1
5.3. Contingent Claims and Derivative Assets
A contingent claim or derivative assets end of term payoff can be precisely determined by the payoff on one or more of the other assets.
Common types of derivative assets are put options and call options, warrants, rights
offerings, futures and forward contracts, and convertible or more exotic types of
bonds. Let us define some of these.
5.3.1. Definition (forward contracts, options, call and put). A forward contract is
the obligation to purchase or sell, at the end of the contracts period (also known as
expiration time), a set number of units (shares) of a particular primitive asset at a
price specified at the beginning of the period. This is a one-period forward contract
or option.
It is important to distinguish the buyer/seller of the contract and buyer/seller of
the shares. This is not the same, as we explain next.
Call option: known briefly as call.
The buyer of the option has the right, but not the obligation, to buy an
agreed quantity of units (shares) from the seller of the option at a certain time
(the expiration date or period) for a certain price (the strike price). The seller
(also known as writer) is obligated to sell the commodity or financial instrument
should the buyer so decide. The buyer pays a fee (called a premium) to acquire
this right. The strike price is sometimes referred to as exercise price.
Put option: known briefly as put.
The put allows the buyer the right but not the obligation to sell an agreed
quantity of units (shares) to the writer of the option at a certain time for
a certain price (the strike price). The writer (seller of the option) has the
obligation to purchase the underlying asset at that strike price, if the buyer
exercises the option.
5.3.2. Essay. Options come in different styles. There are European style options
where the expiration time (period) is fixed, and there are American style options
where the expiration time can be decided by the buyer of the option. Take a Library
(or Wikipedia) tour and explore the various different flavours. Write a report.
5.4. Insurable portfolios and states
An insurable portfolio, with respect to a given economy Z, is a portfolio that pays
off only in a particular state.

MTA (G5078) Autumn 2009 5.4. Insurable portfolios and states

57

5.4.1. Definition (insurable state). The state s is insurable if there exists a vector
s such that
(
6= 0 if = s,
|
z s
(5.4.1)
= 0 if 6= s.
Moreover, by rescaling s appropriately, we may assume
z s | s = 1.

(5.4.2)
|
In matrix parlance, this means that Z s = es where es := 0, . . . , 0, 1, 0, . . . , 0 ,
that is the s-th column of the S S identity matrix.
In this case the scalar 1| s is the cost per currency unit (say ) of the insurance
policy against the occurrence of state s and s is the corresponding insurable portfolio.
5.4.2.
Theorem
(characterization of insurable states). For a given economy Z =

|
z 1 . . . z S and a state s = 1, . . . , S, the following are equivalent.
(i) State s is insurable.
(ii) The vector of asset returns in this state, z s is linearly independent of the vectors
of assets returns on in the other states, z , 6= s.
(iii) Denoting by es the s-th column of the identity matrix we have


(5.4.3)
rank Z es = rank Z.
Proof
[(i) (ii)] Suppose s is insurable, we want to show that z s cannot be expressed as
a linear combination of the other state vectors, z s.
Suppose, by contradiction that z s is linearly dependent on the z s , i.e.,
X
z ,
(5.4.4)
zs =
6=s

it follows that, for all ,


zs| =

z | .

(5.4.5)

6=s

In particular for = s given in Definition 5.4.1, we obtain


z |s = 0

(5.4.6)

(5.4.7)

and thus
1 = zs|s =

s z | s = 0,

6=s

which is absurd. Hence z s is linearly indepdendent of the other states.


[(ii) (i)] Viceversa, suppose z s is linearly independent of the other state return
vectors z , 6= s. We want to show that s is insurable. Let Z be the matrix Z
without its row z s | . Because of our assumption we know that
rank Z (= dim span Z | ) < rank Z(= dim span Z | ),

(5.4.8)

where the span of a matrix is understood as the span of its columns, namely,
span Z | = span {z : 6= s} , and span Z | = span {z : = 1, . . . , S} . (5.4.9)
Introduce now an orthonormal basis of the space span Z | ,
{l span Z | : l = 1, . . . , L}

(5.4.10)

58

MTA (G5078) Autumn 2009 Lecture Notes

(this is always possible for L = rank Z thanks to the Gramm-Schmidt procedure),


and define
L
X
s := z s
(z s | l ) l .
(5.4.11)
l=1

It follows that
2

z s s = |z s |

L
X

(z s | l )2 > 0

(5.4.12)

l=1

because s 6 span { l }l=1,...,L . Moreover, for each fixed = 1, . . . , S, 6= s, we have


z =

L
X

l l

(5.4.13)

l=1

for some (l )l=1,...,L RL , and thus


|

z s =

L
X

l l z s

l=1

L
X

l z s | l = 0.

(5.4.14)

l=1

Thus s (after rescaling) constitutes an insurable portfolio for s.


[(i) (iii)] By elementary linear algebra non-homogenous linear system Z
 = es
for has a non-trivial solution s if and only if the system matrixes Z and Z es
have equal rank.

5.4.3. Example. In the economy

3 2
Z = 1 2 ,
(5.4.15)
2 4
|
|
the portfolio 2, 1 gives a return 4, 0, 0 , which means that 1 is an insurable
state corresponding to the insurable portfolio


1 := 1/2, 1/4 .
(5.4.16)
So insurance against state 1 is possible at cost 1/4 per unit. The returns in states
2 and 3 are collinear (i.e., z 2 linearly depends on z 3 and viceversa), so insurance is
not possible for either.
5.5. Dominance and arbitrage
5.5.1. Definition (dominant portfolio). We say that portfolio w1 dominates w2 if
and only of
Zw1 > Zw2 ,
(5.5.1)
where given two vectors x and y we define
x > y xi yi , for all i = 1, . . . , N and xi > yi , for at least one i . (5.5.2)
Thus the > may be a non-strict inequality for some components but it must be
strict on one component at least. We denote this relation by w1  w2 .
Since the dominated portfolio never outperforms the dominating one, investor should
prefer to hold the dominating one.
5.5.2. Proposition (dominance and unbounded portfolios). If a dominated portfolio exists, then there is no unbounded optimal portfolio.

MTA (G5078) Autumn 2009 5.5. Dominance and arbitrage

59

Proof Let w1  w2 and define x = w1 w2 . By Definition 5.5.1, it follows that


Zx > 0.
Consider now the arbitrary portfolio w. The vector y := w + x is a portfolio for
any R:
1| y = 1| w + 1| x = 1 + (1 1) = 1.
(5.5.3)
Also, by choosing > 0 we obtain y that dominates w. Indeed, > 0 implies that
Zy Zw = Zx > 0.

(5.5.4)


5.5.3. Definition (arbitrage opportunities). If there is a solution to the system of


inequalities
1| 0 and Z > 0
(5.5.5)
we call the solution an arbitrage opportunity of the first type. When the first equation
in (5.5.5) happens to be an equality, then the solution is an arbitrage portfolio
creating a zero investment arbitrage opportunity.
We say that a negative investment arbitrage or an arbitrage opportunity of the second
type exists if there is a solution to
1| < 0 and Z 0,

(5.5.6)

where the non-strict inequality between two vectors is defined by


x y xi yi , for all i.

(5.5.7)

5.5.4. Remark (arbitrage opportunity types). The principal distinction between


the two types of arbitrage is that arbitrage of the first type guarantees positive
returns with non-positive commitments (e.g., borrowing and lending at different
rates) whereas the second kind guarantees non-negative returns with negative commitments (a free lunch).
5.5.5. Example. An economy in which there are arbitrage opportunities only of
the second type is given by the table


5
2
Z=
.
(5.5.8)
5 2
|
Indeed, for any , posing Z =: r1 , r2 we have
  

r1
51 + 22
=
,
(5.5.9)
r2
51 22
and thus r1 , r2 0 is true if and only if r1 = r2 = 0, which excludes arbitrage
opportunities of the first kind.
|
On the other hand, 2, 5 satisfies:
 

 

 2
5
2
2
1 1
= 3 < 0 and
= 0.
(5.5.10)
5
5 2 5
|
Thus 2, 5 is an arbitrage opportunity of the second type for Z.
5.5.6. Example. An economy with arbitrage opportunities only of the first type is

1
0
Z = 1 1
(5.5.11)
1
1

60

MTA (G5078) Autumn 2009 Lecture Notes


|
The pay-off for any investment = 1 , 2 is then

1
Z = 1 2 .
(5.5.12)
1 + 2
For 1 = 1 and 2 = 1 we see that
1| = 0

(5.5.13)

while


1
0

Z = 0 > 0 .
(5.5.14)
0
0
This creates an arbitrage opportunity of the first type. As an exercise show that
there is no arbitrage of second type for Z.
5.5.7. Problem. Show that if there exists a positive investment portfolio, say w >
0, with semi-positive return, that is, Zw 0, then an arbitrage opportunity of the
second type guarantees the existence of an arbitrage opportunity of the first type.
5.6. Pricing under NAP
As we have remarked, investment equilibrium can only be achieved in the absence
of arbitrage. This is why a proper pricing theory works (i.e., for it to have realistic
relevance) only under the No-Arbitrage Principle (NAP). The goal of this section is
to understand how to price a given economy.
5.6.1. Definition (pricing). Given an economy in the form of a table Z, a vector
p RS is said to be a pricing vector supporting the economy Z if and only if
Z | p = 1,

(5.6.1)

p| Z = 1 | .

(5.6.2)

which is equivalent to
5.6.2. Theorem (characterisation of pricing vectors). There exists a non-negative
pricing vector, p RS , supporting an economy Z if and only is there are no arbitrage
opportunities of the second type for Z.
Proof Consider the linear programming problem of finding 0 such that
1| 0 = m = min {1| : Z 0} .

(5.6.3)

Because 0 is an admissible then the minimum (value) is negative.


If there are no arbitrage opportunities of the second type, by (5.5.6) it follows that
the minimum value is 0.
Now from the theory of duality, a finite objective minimum for a primal linear
programming problem guarantees that the dual is also feasible. The dual of (5.6.3)
is
m = max {0| p : Z | p = 1 and p > 0} ,
(5.6.4)
with the same objective m as in (5.6.3). It follows that m = 0 and that there exists
p solving (5.6.4) which means that p is a pricing vector supportin matZ.
Viceversa, if there is nonnegative pricing support vector, then the dual problem
of the dual problem (5.6.4), is given by (5.6.3), is also feasible. By necessity, the
objective of (5.6.4) is m = 0 and thus, by the Theorem of Duality, the primal problem
(5.6.3) has a minimal objective of zero. This means that no arbitrage opportunities
exist.


MTA (G5078) Autumn 2009 5.6. Pricing under NAP

61

5.6.3. Example. This theorem does not prohibit the existence of pricing vectors
with some negative prices. For example, for the economy

1 3
Z = 2 1
(5.6.5)
3 2
the pricing support equations are
p1 + 2p2 + 3p3 = 1
3p1 + p2 + 2p3 = 1

(5.6.6)

p1 = 0.2(1 )
p2 = 0.2 1.4
p3 = .

(5.6.7)

with solution

|
This problem has thus a positive solution p = 0.18, 0.26, 0.1 . Hence no arbitrage
|
opportunities of 2nd type available. Nevertheless, the pricing vector 0.1, 0.3, 0.5
also supports the economy.
5.6.4. Example. Consider the economy

3 2
Z = 1 2
2 4

(5.6.8)

where state 1 is insurable. The pricing support equations are


3p1 + p2 + 2p3 = 1
2p1 + 2p2 + 4p3 = 1

(5.6.9)

with solutions p1 = 0.25, p2 = 0.25 2p3 . Thus there is no arbitrage opportunity


here.
5.6.5. Example. For the economy



5
2
Z=
5 2

(5.6.10)

5p1 5p2 = 1
2p1 2p2 = 1

(5.6.11)



rank Z 6= rank Z | 1

(5.6.12)

the price support equations are

By comparing
(or by trying to solve it), this system is seen to have no solution. Hence arbitrage
opportunities of second type exist for this economy.
5.6.6. Exercise (arbitrage via pricing). Derive pricing equations for the economy

5 2
Z = 3 1
(5.6.13)
2 1
and find out if there are any arbitrage opportunities of first or second type.

62

MTA (G5078) Autumn 2009 Lecture Notes

5.6.7. Exercise. Find out whether there are arbitrage opportunities (first or second
type) in an economy Z such that its transposed matrix is


2 1 3
|
Z =
.
(5.6.14)
1 1 3
5.6.8. Theorem. There exists a positive pricing vector p which supports an economy Z if and only if there are no arbitrage opportunities of first nor second type.
Proof Necessity Assume is an arbitrage opportunity of the first type. Then
Z 0 and p > 0 imply
p| Z > 0.
(5.6.15)
Since p is a pricing support vector, p| Z = 1| . and we get
1| > 0

(5.6.16)

which implies that arbitrage of first type does not exist.


[Sufficiency] Omitted.

5.6.9. Remark. The positivity of the pricing vector assures the absence of arbitrage
and the linearity guarantees the absence of any monopoly power in the financial
market.
5.7. Riskless Issues
From Section 5.1.6 we know that an economy Z has a riskless asset if there exists a
portfolio w1 with 1| w1 = 1, and Zw1 = R1.
5.7.1. Proposition. If there is a riskless asset or portfolio with return R, then in
the absence of arbitrage, the sum of the state prices of any valid supporting pricing
vector is equal to 1/R.
Proof By assumption, there exists a portfolio w1 with
1| w1 = 1,
Zw1 = R1.

(5.7.1)

Then for all valid supporting price vectors p, recalling that p| Z = 1| , we have
R

S
X

ps = Rp| 1 = p| R1 = p| (Zw1 ) = (p| Z) w1 = 1| w1 = 1,

(5.7.2)

s=1

which implies
S
X

ps =

s=1

1
.
R

(5.7.3)


More generally, if there is no way to construct a riskless portfolio, then we have the
following result.
5.7.2. Proposition. The sum of the state prices is bounded below by 1/R and
above by 1/R where R and R are defined by
R := max
min Zw
|

(5.7.4)

R := min
max Zw.
|

(5.7.5)

1 w

and
1 w

MTA (G5078) Autumn 2009 5.8. Riskless Issues

63

Proof By (5.7.5) there exists a portfolio w1 such that R = max Zw1 and thus
Zw1 R1.
For any princing vector p we have p| Z = 1| and thus

p| 1R p| Zw1 = 1| w1 = 1,
from which

S
X

ps

s=1

1
.
R

(5.7.6)
(5.7.7)

(5.7.8)

Likewise, by the definition (5.7.4) we have that


min Zw2 = R,

(5.7.9)

and thus
R1 Zw2 .
By the same argument as above it follows that
p| (1R) p| Zw2 = 1| w2 = 1,
from which

S
X
s=1

ps

1
.
R

(5.7.10)
(5.7.11)

(5.7.12)


5.7.3. Example (bounds are tight). Consider the economy

1 3
Z = 2 1 .
3 2

(5.7.13)

Then the two pricing constraint equations are given by


p1 + 2p2 + 3p3 = 1,

(5.7.14)

3p1 + p2 + 2p3 = 1.

(5.7.15)

and
Solve these equations to get
p1 = 1/7 + (1/7)p2
p3 = 2/7 (5/7)p2

(5.7.16)

All the values of p2 between 0 and 2/5 give valid pricing vectors.
The sum of the state prices takes on all values between 3/5 and 3/7 which are the
reciprocals of R and R for this Z.
5.7.4. The single price law of markets. Two investments with the same payoff in every state must have the same current value. That is, if n1 and n2 are two
vectors of asset holdings and Y n1 = Y n2 , then v | n1 = v | n2 or, equivalently,
Y m = 0 v | m = 0.

(5.7.17)

Using portfolio returns, let = D v m and D v is a diagonal matrix with entries


Dii = vi . Then we have
Y D v 1 D v m = Z = 0
(5.7.18)
and thus
v | D v 1 D v m = 1| = 0.
(5.7.19)
Any violation of this last condition implies the existence of an arbitrage opportunity
of the second type.

64

MTA (G5078) Autumn 2009 Lecture Notes


5.8. Applications

5.8.1. Example. Consider a security market consisting of two securities, A and B.


At time t = 0, they have prices of P A(0) and P B(0). At t = 1, we look at two
possibilities:
Security Time t = 0 price Market up at t = 1 Market down at t = 1
P (0)
P (1)
P (1)
A
5
6
4
B
9
12
8
There is an arbitrage opportunity here. I can buy 1 unit of B and sell 2 units of
A at time t = 0, this way, I have 1 pounds at t = 0. At time t = 1, I can do the
reverse, sell 1 unit of B and buy 2 units of A. Summarising
? When market is up, this brings me 0 pounds.
? When market is down, this brings me also 0 pounds.
? Overall, I made 1 pound without any risk of lose.
However, if we want to do the same thing but with
Security Time t = 0 price Market up at t = 1 Market down at t = 1
P (0)
P (1)
P (1)
A
5
6
4
B
10
12
8
Then we get no profit. The opportunity for arbitrage has been eliminated.- the
prices are now consistent.
5.8.2. Example. Consider the following situation:
Security Time t = 0 price Market up at t = 1 Market down at t = 1
P (0)
P (1)
P (1)
A
7
14
3
B
7
14
2
In this case, an arbitrage opportunity exists. Buy 1 unit of A and sell 1 unit of B
will achieve no risk profit.
As the tendency of buying A and selling B increases, arbitrage opportunities will
be eliminated. In this case, we have P A > P B.
5.8.3. Example (Forward contracts). At time t = 0, say, an agreement is made
between two parties A and B that A will buy from B a specified amount of an asset
S and a specified price on a specified future date.
The person selling is said to hold a short forward position. The person buying is
said to hold a long forward position.
Let S(r) be the price of the underlying asset price at time rthis is usually unknown
a prioriK be the price agreed at t = 0 for asset S.
It is usually assumed that there is a risk-free force of interest over the term of
the contract.
Ignoring other factors such as inflation and interest rate, we use the following example to explain the concept of forward contract: Investor I agrees to sell 1000 BP
shares in 6 monthss time to Investor II at a price of 7 per share. The current
price is 6.50 per share: 7 is the forward price. In order to gain the right to sell
in 6 months time, Investor I pays Investor II an initial sum of 0.02 per share (i.e.,
20).
Six months later, the share price is 6.90. Since Investor I, having paid 20 to
guarantee the put option, chooses to sell (put) 1000 shares at 7 a piece, i.e., for a

MTA (G5078) Autumn 2009 5.8. Applications

65

total 7000, but the shares are, according to the market price, really worth 6900,
so Investor I makes a profit of 100 20 = 80. Investor II, by contrast, is the writer
of the contract and is obliged to buy made a loss of 100 20 = 80 in this situation.
5.8.4. Example. Forward price of a security without income or cost. Consider the
following two portfolios:
Portfolio A: At t = 0, enter a forward contract to buy one unit of an asset S,
with forward price K maturing at time T . Simultaneously invest an amount
KeT in the risk-free investment the price of the portfolio is KeT .
Portfolio B: At t = 0, buy one unit of the asset, at the current price S(0) the
price of the portfolio is S(0).
At time t = T , the price of the portfolio A is: receiving K from the risk-free
investment, pay K to buy one unit of Sthe payout of the portfolio is one unit of
S share.
The price of the portfolio B is one unit of S share. If we follow the principle of no
arbitrage, the two portfolio has the same payout, hence the initial price is the same,
KeT = S(0), which implies K = S(0)eT . This gives us a guide on the price of
the value of K.
5.8.5. Exercise. A three-year forward contract exists in a zero-coupon corporate
bond with a current price per 100 nominal of 50. The yield available on threeyear government securities is 6% p.a. effective. Calculate the forward price.
In the above problem, what if the three-year forward contract is replaced by sixmonth forward contract ?
(Note: a zero-coupon bond is a security that people purchase at time t = 0 for a
specified lump sum at some specified future date.)
5.8.6. Example (Forward price of a security with fixed cash income). Consider
the following problem: at a time t1 where 0 < t1 < T , the security underlying the
forward contract provides a fixed amount c to the holder.
Portfolio A: Enter a forward contract to buy one unit of S, with forward price
K, maturing at T , simultaneously invest an amount KeT + ce(t1) in the
risk-free investmentthe price at t = 0 is hence KeT + ce(t1) the payout
at t = T is hence K + ce(T t1) K + one unit of asset with price
S(T ) = ce(T t1) + one unit of asset with price S(T ).

(5.8.1)

Portfolio B: Buy one unit of the asset, at the current price S(0). At time t1,
invest c in the risk-free investment the price at t = 0 is hence S(0) the
payout at t = T is hence ce(T t1) + one unit of asset with price S(T )
Since yield from A and B are the same, no arbitrage assumption gives
KeT + ce(t1) = S(0),

(5.8.2)

K = S(0)eT ce(T t1) .

(5.8.3)

and thus
5.8.7. Example (forward price for a security with known dividend yield). The
dividend yield for an equity is defined to be:
Dividend per share
Dividend yield =
.
(5.8.4)
Price per share
Let D be the known dividend yield per annum. Assume that dividends are received
continuously, and are immediately reinvested in the security of S. (Note: The actual
amount of dividend varies with the price).

66

MTA (G5078) Autumn 2009 Lecture Notes

Start with one unit of S at time t = 0, the accumulated holding at time T would be
eDT units of the security. Consider the following two portfolios:
Portfolio A: Enter a forward contract to buy one unit of S, with forward price
K, maturing at time T ; simultaneously invest an amount KeT in the risk-free
investment - At t = 0, the price of Portfolio A is KeT .
Portfolio B: Buy eDT units of S, at the current price S(0). Reinvest dividend
income in S immediately after it is received At t = 0, the price of Portfolio
B is S(0)eDT .
At t = T , the net portfolio A is: 1 unit of S.
At t = T , the net portfolio B is: 1 unit of S.
Using the no-arbitrage assumption the prices must also be the same, that is,
K eT = S(0)eDT

(5.8.5)

thus
K = S(0) e(D)T .
(5.8.6)
The following is a concrete numerical case: Assume that the dividend yield of a
portfolio of shares with current price of 1 000 is 4% p.a. Calculate the forward
price of a one-year forward contract, based on the portfolio, assuming dividends are
received continuously and the risk-free rate of interest is 4.602 8% p.a. effective.
4.602 8% p.a. effective is equivalent to a force of interest = 4.5%. Therefore
K = 1000 e0.0450.04 = 1005.012521.

(5.8.7)

5.8.8. Exercise. The current share price of a stock is 100. Dividends are paid
continuously and the current dividend is 2 per annum. Calculate the forward price
of a five-year contract on the asset if the risk-free force of interest is 5% per annum
and the dividend yield remains constant.
5.8.9. Exercise. Deduce a formula for the forward price, K, for an equity forward
contract in T years time (T is an integer). Assume a constant dividend yield D,
and that dividends are received in the middle of each year and are immediately
reinvested.
5.8.10. Definition (portfolio hedging). Hedging is a general term which describes
the use of financial instruments (including stocks, bonds, forward contracts and
more complex financial contracts such as options) to reduce or eliminate a future
risk of loss.
5.8.11. Exercise (hedging). In the above example, is it possible to calculate K if
the dividend is fixed amount of cash provided
1. The dividend is reinvested in shares of S.
2. The dividend is reinvested in risk-free investment.
5.8.12. Example. An investor agrees to sell an asset with no income in 2 years
time. The current price of the asset is 100 and the risk-free force of interest is 10%.
We assume that the investor borrows and invests money at the risk-free rate.
The forward price must be 100e2x0.1 = 122.1402758. This is the amount that the
investor will receive at time 2 for selling the asset.
In order the sell the asset, the investor first needs to buy the asset.
Suppose the investor chooses to buy the asset now and so the accumulated profit at
time 2 is 0.
If the price of the asset at time 2 turns out to be 110, then the investor would have
made a profit if she had waited and purchased the asset at time 2. The accumulated
profit would have been 122.1402758 110.

MTA (G5078) Autumn 2009 5.7. Applications

67

If the price of the asset at time 2 turns out to be 130, then the investor would have
made a lose if she had waited and purchased the asset at time 2. The accumulated
lose would have been 130 122.1402758.
This is called a static hedge since the hedge portfolio, which consists of the asset
to be sold plus the borrowed risk-free investment, does not change over the term of
the contract. For more complex financial instruments, such as options, the hedge
portfolio is more complex, and requires (in principle) continuous rebalancing to
maintain. This is called a dynamic hedge.
5.8.13. Exercise. Assume that there is no interest or dividend income and consider a forward contract agreed at time t = 0, with a forward price K, for buying
one unit of S. The maturity date of the contract is time T . Find the value of the
contract at an arbitrary intermediate time t. Construct a numerical example to
illustrate the point.
Hint. These values can be found using the no arbitrage assumption and similar
techniques to those used to find the forward price.
Exercises and problems on Arbitrage and portfolios
Exercise 5.1 (short selling). Find the definition of short sale and some application
of such activity.
Exercise 5.2 (can you spot arbitrage?). For each of the following, state whether
or not there is an arbitrage opportunity:
(a) An individual needs change and is offering you a 10 pound note in return for 9
pounds change.
(b) England football team is playing Croatia tonight, a friend says that he will pay
you 20 pounds if England loses and if you support Croatia during the game.
(c) Mr A offers to purchase a car for 12000 pounds, Mr B offers to buy the same
car for 15000 pounds.
(d) You and your friend are betting on the weather tomorrow. If it does not rain,
your friend will pay you 2 pounds, if it rains, you pay your friend 2 pounds.
(no, gamble 6= arbitrage)
Exercise 5.3. Are any of the assets in the economy


1 1 2
Z=
1 2 3

(5.3.1)

redundant?
 
Z
Problem 5.4. What can be said of the matrix | if one of the assets in Z is
1
redundant?
Problem 5.5. Show that if there exists a positive investment portfolio, say w > 0,
with semi-positive return, that is, Zw 0, then an arbitrage opportunity of the
second type guarantees the existence of an arbitrage opportunity of the first type.
Exercise 5.6 (arbitrage detection via pricing). Derive pricing equations for the
economy

5 2
Z = 3 1
(5.6.1)
2 1
and find out if there are any arbitrage opportunities of first or second type.

68

MTA (G5078) Autumn 2009 Lecture Notes

Exercise 5.7 (arbitrage detection via pricing). Find out whether there are arbitrage opportunities (first or second type) in an economy Z such that its transposed
matrix is


2 1 3
|
Z =
.
(5.7.1)
1 1 3
Exercise 5.8. A three-year forward contract exists in a zero-coupon corporate
bond with a current price per 100 nominal of 50. The yield available on threeyear government securities is 6% pa effective. Calculate the forward price.
In the above problem, what if the three-year forward contract is replaced by sixmonth forward contract?
(Note: a zero-coupon bond is a security that people purchase at time t = 0 for a
specified lump sum at some specified future date.)
Exercise 5.9. The current share price of a stock is 100. Dividends are paid
continuously and the current dividend is 2 per annum. Calculate the forward
price of a five-year contract on the asset if the risk-free force of interest is 5% per
annum and the dividend yield remains constant.
Exercise 5.10. Deduce a formula for the forward price, K, for an equity forward
contract in T years time (T is an integer). Assume a constant dividend yield D,
and that dividends are received in the middle of each year and are immediately
reinvested.
Exercise 5.11 (hedging). In the above example, is it possible to calculate K if
the dividend is fixed amount of cash provided
1. The dividend is reinvested in shares of S.
2. The dividend is reinvested in risk-free investment.
Exercise 5.12. Assume that there is no interest or dividend income and consider
a forward contract agreed at time t = 0, with a forward price K, for buying one unit
of S. The maturity date of the contract is time T . Find the value of the contract at
an arbitrary intermediate time t. Construct a numerical example to illustrate the
point.
Hint. These values can be found using the no arbitrage assumption and similar
techniques to those used to find the forward price.

APPENDIX A

A review of optimisation
1.1. Homotheticity and homogeneity
1.1.1. Definition (homogeneous functions). A function F : Rd R is said to be
homogeneous of degree k, for some given k R, at the point x0 if for all 6= 0 we
have
F ((x x0 )) = k F (x x0 ).

(1.1.1)

If no reference is made about x0 then it is considered x0 = 0 to keep the notation


simpler. All the results for 0 can be adapted for a generic x0 .
1.1.2. Exercise. Verify that if a function F : Rd R is differentiable and homogeneous of degree k, then its first order derivatives are homogeneous of degree k 1;
namely check that (1.1.1), with x0 = 0 for simplicity, implies
F ((x)) = k1 F (x).

(1.1.2)

Hint. Differentiate both sides of (1.1.1) and use the chain rule.
This is a consequence of the following more precise statement.
1.1.3. Theorem (Euler). If F is homogeneous of degree k, then
x Rd ,

x| F (x) = kF (x),

(1.1.3)

where
|

x F (x) =

d
X
i=1

xi

F
(x).
xi

(1.1.4)

1.1.4. Theorem (homothetic functions). A function F (x) is said to be homothetic


if F (x) = h(g(x)) where g : Rd R is homogeneous and h : R R is continuous,
non-decreasing and positive.
1.2. General optimisation (also known as nonlinear programming)
Optimisation (also known as nonlinear programming) is an important branch of
applied mathematics. It is one of the most useful when it comes to Economics and
Management, especially the subbranch known as linear programming. We give here
an overview of nonlinear programming.
1.2.1. Uncontrained optimisation. We recall, from basic calculus texts [Fle77],
that to find a local maximum (resp. minimum) x of a differentiable real-valued
function f : Rd R, it is necessary that the following EulerLagrange equation be
satisfied
f (x) = 0.

(1.2.1)

70

MTA (G5078) Autumn 2009 Lecture Notes

If the function f is twice differentiable, then condition (1.2.1) becomes sufficient if


the Hessian1 of f at x is moreover negative (resp. positive), i.e.,
z Rd : z | D2 f (x)z 0

(resp. 0) .

(1.2.3)

1.2.2. Equality constrained optimisation and Lagrange multipliers.


Problem (constrained minimisation). Given a function f : Rd R, a (possibly
scalar) vector-field g : Rd Rl , and a (possibly scalar) constant vector a, find
points x such that


f (x ) = min f (x) : x Rd and g(x) = a .
(1.2.4)
Solution. Introduce a vector Lagrange multipliers Rl , as an extra unknown,
via the Lagrangian function
L(x, ) := f (x) | (g(x) a),

(1.2.5)

and calculate the critical points of L, i.e., as those pairs (x , ) that solve the
following constrained EulerLagrange equations (also known as first-order optimality
conditions)
L(x, ) = 0,
(1.2.6)
which can be more explicitly written as a (usually coupled) system of (usually nonlinear) equations:
f (x) g(x) = 0,
g(x) = a.

(1.2.7)

(Note that the gradient of g, g is the transpose of the Jacobian matrix Dg.)
The following second-order optimality condition, makes the first-order optimality
condition sufficient: let x be a critical point of f constrained to g, define the
bordered Hessian at x as being the matrix given by
 2

D f (x ) g(x )
H :=
.
(1.2.8)
Dg(x )
0
If H is positive definite, then the critical point x is a local minimum of f constrained to g = a.
1.2.3. Exercise (constrained maximisation). Adapt 1.2.2 for a maximisation
problem.
1.2.4. Exercise (simplifying the constraints constant). Explain the sentence without loss of generality we may consider a = 0 in 1.2.2.
1.2.5. Exercise (gradient calculus). Recall that the gradient of a field g is the
transpose of its Jacobian (i.e., the derivative) and show that for constant with
respect to x we have
x [| g(x)] = g(x).
(1.2.9)
1The

Hessian of f is defined as the matrix-valued function such that for each x

H11 . . . H1d
2

.. where H = f (x) for 1 i, j d.


..
D2 f (x) = ...
ij
.
.
xi xj
Hd1 . . . Hdd

(1.2.2)

MTA (G5078) Autumn 2009 1.2. General optimisation (also known as


nonlinear programming)
71
1.2.6. Inequality constrained optimisation and the KKT method.
Problem (inequality constrained minimisation). Given a function f : Rd R, a
(possibly scalar) fields g : Rd Rl and h : Rd Rm , find points x such that


f (x ) = min f (x) : x Rd and g(x) = 0 and h(x) 0 ,
(1.2.10)
where inequality among vectors is defined by


v1 , . . . , vm w1 , . . . , wm vi wi

i = 1, . . . , m.

(1.2.11)

Solution (KarushKuhnTucker (KKT) method). Introduce the Lagrange multiplier , the KarushKuhnTucker multiplier and the KKT function
M (x, , ) := f (x) + | g(x) + | h(x).

(1.2.12)

For x to be a minimum it is necessary that, for some Rl and Rm , it


satisfy
(i) the stationarity condition
x M (x , , ) = 0;

(1.2.13)

(ii) the primal feasability condition


M (x , , ) = 0, and M (x , , ) 0;

(1.2.14)

(iii) the dual feasibility condition


0

(1.2.15)

with active j-th constraint if j > 0 and inactive j-th constraint if j = 0 for
j = 1, . . . , m;
(iv) and the complementary slackness condition
| h(x ) = 0.

(1.2.16)

Note that the Lagrange multiplier method is a submethod of KKT. Indeed, the
Lagrange multiplier method is the case where h 0 and where (iii)(iv), as well as
the second equation in (1.2.14) are simply dropped.
Conditions (i)(iv) are only necessary. Second order conditions are possible, but
stating them here is beyond our scope. The interested reader might want to check
some textbook on the matter [NW99, e.g.].
1.2.7. Exercise (KKT for maximisation problems). Consider a maximisation problem max f (x) with the same constraints as in (1.2.10) and show that (i) (iv) are
necessary, but with the KKT function defined as
M (x, , ) := f (x) + | g(x) | h(x).

(1.2.17)

Hint. Rewrite the maximisation problem for f (x) as a minimisation problem for
f(x) = f (x) and apply KKT.
1.2.8. Example. Let f : Rd R be a generic function. The necessary first-order
optimality conditions for maximising f (x) subject to the constraint x x0 (which
can be writen as h(x) := x0 x 0), can be obtained by taking h(x) = x0 x
and g(x) 0. Hence we need to solve
f (x) + = 0,

(1.2.18)

x0 x 0, 0 and | (x0 x) = 0.

(1.2.19)

f (x) 0, x0 x, and (x x0 )| f (x) = 0.

(1.2.20)

This is equivalent to

72

MTA (G5078) Autumn 2009 Lecture Notes

1.2.9. Exercise. Find the points of the form (x, y) that realize min(x2 + y 2 + xy)
and max(x2 + y 2 + xy), subject to x2 + y 2 8.
Solution. Let f (x, y) := x2 + y 2 + xy, then


2x + y
f (x, y) =
.
2y + x

(1.2.21)

Furthermore, we introduce the KKT functions (with + for the minimum and for
the maximum)
M (x, y, ) := f (x, y) (x2 + y 2 8),
(1.2.22)
and derive:
(i) the stationarity condition
  

  

0
2x + y
2x
2(1 )x + y
=

=
0
2y + x
2y
x + 2(1 )y
which happens to be linear in (and easily solved) for (x, y) as follows
(
(0, 0)
for 6= 1/2,
(x, y) =
(t, t), t R for = 1/2;

(1.2.23)

(1.2.24)

(ii) the primal feasibility


x2 + y 2 8 0;

(1.2.25)

0,

(1.2.26)

(iii) the dual feasibility


which excludes the case = 1/2 and implies (x, y) = (0, 0) as the unique minimum
candidate, while it is clearly satisfied for the maximum;
(iv) the complementary slackness

x2 + y 2 8 = 0,
(1.2.27)
which, based on the previous findings, implies = 0 for the minimums and x2 +y 2
8 = 0 with = 1/2 for the maximum and hence x2 = 4, i.e., (2, 2) and (2, 2).
In effect, evaluating f (2, 2) = 12 = f (2, 2) it turns out that both (2, 2) are
constrained maximums with active constraint (in the sense that the equality holds).
On the other hand, (0, 0) is the only constrained minimum with inactive constraint;
in fact, this could have been deduced without going through the whole optimisation
song and dance by noting that
(
 > 0 for (x, y) 6= (0, 0),
1 2
2
2
2
f (x, y) = x + y + xy x + y
(1.2.28)
2
= 0 for (x, y) = (0, 0).
1.2.10. Exercise. Find the points of the form (x, y) that realise min x2 + y 2 and
max x2 + y 2 subject to (x 10)2 + y 2 < 4.
1.3. Linear programming (LP) and duality
Linear programming (LP) is a subbranch of Nonlinear Programming, roughly speaking, like Linear Functional Analysis is a subbranch of Nonlinear Analysis. Like
NP, LP involves linear target functionals and constraints, but these must be linear
(affine, to be precise), i.e., for some constant vector k0
k(x + y) = k(x) + k(y) + (1 )k0

, R and x, y Rd (1.3.1)

MTA (G5078) Autumn 2009 1.3. Linear programming (LP) and duality
73
Figure 1. Geometric representation of the LP problem in Example 1.3.2.
|P +
lues c
rget va
a
t
4

4
3

1
(0,4)

(0,6)

c
(3/2,3)
(3,0)

set P
feasible

(6,0)

or, more simply put, all the involved functionals (f ) and constraints (g and h) are
of the form
k(x) = Kx + k0 for x0 Rd ,
(1.3.2)
for some fixed matrix K and vector k0 .
We used LP in Chapter 5. It is widely used in Economics and Business Management.
In this section we define is the linear optimisation problem (abusively called also
LP) which the LP method can solve and then explain how the LP method works.
1.3.1. Problem (linear programming (LP)). Given vectors c Rd and b Rk , a
matrix A Rkd and a scalar R, find x Rd such that
c| x + = max {c| x + : Ax b} .

(1.3.3)

1.3.2. Example. An example of linear programming problem, as introduced in 1.3.1,


is given by the following data


6
2
1
 

2
3
12
1


d = 2, k = 4, A =
1 0 , b = 0 , c = 1/2 , = 1.
0 1
0
An example is given in Figure 1.
1.3.3. Analysis of the LP Problem 1.3.1.
Without loss of generality we will

assume that = 0 and drop it. Let P := x Rd : Ax b be the feasible set,
which is a polyhedron in Rd . Finding the maximum above can be seen as shifting
the hyper-plane orthogonal to the vector c, as long as it contains points in P .
Suppose the maximum is finite, say its value is , and attained by the element
x P . Let J be the subset of [1 : k] for which the inequalities in Ax b are
actual equalities
[A]j x = bj j J
(1.3.4)
Here [M ]j indicates the j-th row of a matrix M . Geometric intuition implies that
J is not empty because a linear function cannot realise its maximum in an internal
point of P , unless it is constant. The set J is known in jargon as the set of active
constraints.

74

MTA (G5078) Autumn 2009 Lecture Notes

Elementary geometry implies that = c| x is a nonnegative-coefficient linear combination of [A]j x = bj , for j J , i.e., there exist j 0, for j J , such that
P
= J j bj . Posing j := 0 when j 6 J , we obtain a vector Rk such that
|

0 and c x = = b =

k
X

j [A]j x = | Ax .

(1.3.5)

j=1

(Note that although Ax need not equal b, in the above equation we do have an
equality.) It follows that
A | c x .
(1.3.6)
Say c = 1 a1 + . . . + k ak and = 1 1 + . . . + k k , with 1 , . . . , k 0.
Since 1 1 + . . . + k k = b, we also know that 0 and A = c from above. This
implies
max {cx|Ax b} = = 1 1 + . . . + k k = b min{yb|y 0, yA = c}.
The inequality
max {cx|Ax b} min {yb|y 0, yA = c}
is trivial. Hence the equality
max {cx|Ax b} = min {yb|y 0, yA = c}.
Second, lets introduce complementary slackness: Let A be a matrix, let b be a
column vector, and let c be a row vector. Consider the LP duality equation
max {cx : Ax b} = min {yb : y 0 and yA = c} .

(1.3.7)

Assume that both optima are finite, and let x0 and y0 be feasible solutions, then the
following are equivalent:
(i) x0 and y0 are optimum solutions in (1.3.7);
(ii) cx0 = y0 b
(iii) if a component of y0 is positive, the corresponding inequality in Ax b is
satisfied by x0 with equality, i.e. y0 (b Ax0 ) = 0.
Condition (iii) is called complementary slackness. The equivalence of (i) and (ii)
follows directly from the Duality Theorem. The equivalence of (ii) and (iii) follows
from the having that cx0 = y0 Ax0 y0 b and y0 Ax0 = y0 b are true if and only (iii)
is true.
The primal-dual iteration method for LP was established by Dantzig, Ford and
Fulkerson [DFF56]. The idea is as follows: starting with a dual feasible solution y,
the method searches for a primal feasible solution x satisfying
y(b Ax) = 0.

(1.3.8)

If such a solution has been found, x and y are optimum primal and dual solutions.
If no such primal solution, the method prescribes a modification of the dual feasible
solution y, after which we iterate the procedure (start anew).
Suppose we want to solve the LP problem (where c = (1 , 2 , . . . , n ) is a given
vector)
min{cx|x 0, Ax = b}
(1.3.9)
where A is a m n matrix, b is a m-vector and c is an Rn vector. The dual LP
problem is
max{yb|yA c}
(1.3.10)

MTA (G5078) Autumn 2009 1.3. Linear programming (LP) and duality
75
1.3.4. Primal-dual iteration. Suppose we have a feasible solution y0 for the dual
problem (1.3.10). Let A0 be the submatrix of A consisting of those columns aj of A
for which y0 aj = j . To find a feasible primal solution for which the complementary
slackness condition holds, solve the restricted linear program
min{|x0 , 0, A0 x0 + b = b} = max{yb|yA0 0; yb 1}

(1.3.11)

x00 ,

If the optimum value in (1.3.11) is 0, let


be an optimal solution for the
0
0 0
minimum. So x0 0, A x0 = b and = 0. Hence by adding zero components, we
get x0 0 such that Ax0 = b and (x0 )j = 0 if y0 aj < j . By the complementary
slackness it follows that x0 and y0 are optimum solutions for (1.3.9) and (1.3.10).
If the optimum value in (1.3.11) is strictly positive, then it is 1. Let y be an optimum
solution for the maximum. Let be the largest rational number such that
(y0 + y)A c.

(1.3.12)

Note that > 0, since yA0 0, y0 A c and y0 aj < j , if a is not in A0 . Note


also that is easy to calculate. In the case = , the max (2) is unbounded, as
then (y0 + y)A c and (y0 + y)b if . Reset y0 = y0 + y, and start
iteration again.

APPENDIX B

Brush-up of Probability Theory


This is a very quick sketch of some basic probability theory, not a courses lecture
notes, nor a substitute! To study probability theory properly you should check
some textbook, or take (unless you already have taken) such a course. A handy and
concise reference, for those having some grounding in the topic, is the monograph
by Jacod & Protter [JP03].

2.1. Probability space and measure


Probability theory models the measuring the likelihood of certain events to happen.
The starting point is a set that contains all the elementary events called probability
space. Consider a space , if you are not familiar with probability theory, the best
example of probability space is the interval := [0, 1] = {x R : 0 x 1}.
A probability measure P is, roughly speaking, a real-valued set-argument function
P : F R
A 7 P A

(2.1.1)

where P A models the probability of the event A happening. F is the collection of


all possible events (including, but not only, elementary events), which are subsets
of the probability space .
So, F is a subset of P (), the power set of . Ideally, we would like F to
contain all subsets of , but this is not always possible without destroying the nice
properties of P and F which we list next. So in general we have
F ( P () .

(2.1.2)

2.1.1. Sum-algebra (also known as -algebra). The collection F enjoys the


following properties:
(i)
(ii)
(iii)
(iv)

F 3 ;
A, B F A B S
F;
{Ai : i N} F i Ai F ;
A F r A F.

Such a collection goes by the somewhat unlucky name of -algebra, or -field. Less
frequently we talk about sum-algebra/field, or you may want to strike the middle
ground we could use sigmalgebra.1
1Part

of building up jargon barriers common in science, but a bit less so in mathematics, is


to come up with intimidating terms like -algebra, K-theory, -method and A-stability. In all
such names (surprisingly for mathematicians are picky about symbols) the letters , K, and A
are devoid of any significance and are just acronyms. In our case the comes from contracting
Summe, which is German for sum, because, in early days of set theory, the union of sets was
though of as the sum of sets.

78

MTA (G5078) Autumn 2009 Lecture Notes

2.1.2. Probability measure. The probability measure P enjoys the following


properties
(i)
(ii)
(iii)
(iv)

P = 0 and P = 1;
A, B F and A B P A P B;
P (A
P A + P B P (A B);
S B) = P
P i=1 Ai
i=1 P Ai .

It follows that 0 P (A) 1 for any A F . Other interesting facts like the
Inclusion-Exclusion Principle follow. The concept of indepedent event is maybe the
deepest in Probability Theory and you should check some good reference on the
topic [JP03, e.g.].2
2.2. Random variables
Given a probability measure (, F , P ). A random variable (also known as random
number) is a function X,
X : R,
(2.2.1)
such that the counterimage of any open interval in R is an element of F . In symbols
we write this as
{ : a < X() < b} := {a < X < b} F ,

(2.2.2)

for all a, b R.
In practice, you need not worry whether a given function X defined on is a random
variable: you may safely assume it is.3
2.2.1. Expectation. Given a random variable, its expectation is its integral against
the probability measure P over :
Z
E[X] = E X :=
X()P d.
(2.2.3)

Probability spaces, such as , come in two flavours: discrete, or countable and


uncountable. Discrete probability spaces arise in simple applications where we have
finite spaces (e.g., toss of a pair of dice) or infinite discrete spaces (the life-span in
years of a chemical pollutant, which is virtually infinite).
On discrete probability spaces of the symbols appearing in (2.2.3) the integral is
interpreted as a sum or a series (depending on whether is finite or infinite) and
the elementary increment d can be replaced by singletons, i.e., sets of the type
{}, containing a single element . Thus, equation (2.2.3), in the case of a
discrete probability space reads
X
E[X] =
X()P {} .
(2.2.4)

On non-discrete spaces a whole theory of integration (or measure) must be developped, which is beyond this brief summary of probability. We will assume that
we talk about a probability space we actually mean a triple (, F , P ), not only which
is just a set, devoid of structure, without F and P . Often, though, F and are understated and
all that is mentioned is the probability measure P . In fact, given P we may obtain F as Dom P
and as the union of the collection F , so mentioning the triple is redundant.
3Although functions that are not measurable are shown to exist, using the Axiom of Choice,
you need to work quite hard to find one in practical applications.
2When

MTA (G5078) Autumn 2009 2.2. Random variables

79

we know how to integrate. By integral we mean definite integrals. In financial


endeavours, the space of events can be extremely complicated.4
2.2.2. Example (random variables). (a) A good way to think of random variables, especially if your aim is financial mathematics, is to attach the meaning
of price to it. For example, we may imagine of attaching the price of 1 to each
single event .
This yields the extremely simple random variable X defined on such that
X() = 1 for all . The probability of having X = 1 is given by
P {X = 1} = P = 1,

(2.2.5)

and the expectation (in practice thought as the average price) is given by
Z
EX =
P d = 1.
(2.2.6)

The reason behind the last equality (beyond the intuitively clear fact that the
average of 1 always, must be 1) is that the integral, being the area under the
graph, can be, in this particular situation calculated as
Z
P d = 1 P = 1 1 = 1.
(2.2.7)

(b) Given an event A F , its indicator function is defined by


(
1 if A,
1A () =
0 otherwise.

(2.2.8)

An indicator function is a random variable. A random variable, when multiplied


with a scalar (i.e., a number in R) yields a random variable. The expectation of
such random variables can be also easily calculated in this case. Suppose, that for
some c R and A F , we define
X() := c1A ()

(2.2.9)

then
E X = E[c1A ] = c E 1A = c

Z
P d = cP A.

(2.2.10)

(c) A simple function is a function that is the indicator function of an event A, or


the linear combination of simple functions. For example, let us pose
X() := a1A () + b1B (),

(2.2.11)

where A and B are two events in F , and a, b R is a simple function. The linear
combination of two random variables is a random variable. Thus, simple functions
are random variables and
E X = E[a1A + b1B ] = aP A + bP B.

(2.2.12)

finance (, F , P ) is supposed to model all of the universe, and how eventss occurence
would affect the uncertain prices in stock markets, so describing would be mad. Of course,
events such as the birth of a star, or the fall of a comet onto a distant black hole, wont affect
financial markets on Earth as much as other events, such as the closure of the straight of Hormuz,
or the fall of a comet on Earth. In practice, one is not interested in describing , but simply uses
it to calculate quantities of interest, e.g., the expectation of the price of a stock.
4In

80

MTA (G5078) Autumn 2009 Lecture Notes

(d) Suppose a stock is priced at 3 at time 0 and that its price at time 1 is given by
the random number
(
5 with probability 30%,
S=
(2.2.13)
2 with probability 70%.
This means that S is a random variable of the form a1A + b1B with a = 5, A =
{S = 5}, b = 2, b = {S = 2}. We can thus calculate the stocks expected price as
E S = 5 0.3 + 2 0.7 = 2.9 .

(2.2.14)

Random variables taking only 2 possible value, such as S in this example, are called
binomial.
2.2.3. Proposition (CauchyBunyakovskiiSchwarz inequality). If X and Y are
two random variables on a space (, F , P ) then
   
E[XY ]2 E X 2 E Y 2 .
(2.2.15)
Note that standard operator precedence in notation means E[XY ]2 = (E[XY ])2 in
the above result.
2.2.4. Variance, standard deviation and covariance. Suppose X is a random
variable on the probability space (, F , P ). Its expected value E X, when finite,
is also known as the average. C.F. Gau, in proving his celebrated Law of Large
Numbers realised the importance of the variance of X, which he defined to be




var X := E (X E X)2 = E[X 2 ] E[X]2 = E X 2 E[X]2 .
(2.2.16)
The second and third identities are a result of simple algebra and properties of E.
Many authors use the notation = X for E X and the variance is thus expressed
as
var X = E[X 2 ] 2 .
(2.2.17)
The standard deviation of the random variable X, often denoted by X (or just
if the only random variable being discussed is X), is the square root of its variance,
i.e.,

1/2
X := var X = E[X 2 ] 2
.
(2.2.18)
If Y is another random variable then the covariance of the pair (X, Y ) is defined as
cov[X, Y ] := E[(X E X)(Y E Y )] .

(2.2.19)

Covariance plays an important role in the study of independence of random variables.


2.3. Distribution and density
Let X be a random variable. Instead of integrating X on , which is in general
some abstract probability space, it is often much more handy to integrate some
composition of X over R. To do this, roughly speaking, we look for smart change
of variables from into R.
2.3.1. Cumulative distribution function (cdf ). A natural choice for such a
change of variables is to consider the cumulative distribution function of X, denoted
by cdf X or FX or F (when X is understood from the context), defined for each fixed
real number x as the probability for the random real number X of being no more
than x. In symbols, we have
cdf X (x) = FX (x) = F (x) := P {X x} .
The function so defined is a monotonic nondecreasing function.

(2.3.1)

MTA (G5078) Autumn 2009 2.3. Distribution and density

81

2.3.2. Law (also known as probability distribution) of a random variable.


The law, or probability distribution of the random variable X is a probability measure
defined on the Borel sum-algebra, B, in R. This measure is defined, for each B B,
by

lawX B := PX (B) := P {X B} = P X 1 B ,
(2.3.2)
where X 1 B is the counterimage of B (the last equality above is just an equivalent
expression of the same object). If FX = cdf X , then we have that
PX (a, b] = FX (b) FX (a)

a < b R.

(2.3.3)

Also, we have
PX [a, b] = FX (b) FX (a) and PX (a, b) = FX (b) FX (a),

(2.3.4)

where F (x) := lim0<0 F (x ) is the left limit of F at x.


Conversely, it can be shown that the law PX is a probability measure on R such that
Z
FX (x) =
PX ( d) x R.
(2.3.5)
(,x]

As we shall see next, somebut not all!probability measures on R can be represented using a combination of a density function and a pointwise mass distribution.
2.3.3. Lebesgues Decomposition and RadonNikodym Theorems. 5 Although the following results are valid for general measurable spaces, we concentrate
on the sum-algebra B of Borel/Lebesgue measurable sets in R.
Denote by L the Lebesgue Measure on R, and L( dx) simply by dx. The Lebesgue
Decomposition Theorem says that any measure, call it M , on R can be decomposed
into two measures: a regular part, M reg , and a singular part, M sng such that
M (A) = M reg (A) + M sng (A)
L(A) = 0 M

reg

(A) = 0 (M

reg

A B,

absolutely continuous w.r.t. L, M reg  L) ,

M sng (A) 6= 0 L(A) = 0 (M sng and L are singular, L M sng ) .


(2.3.6)
Another important result of Measure Theory, the RandonNikodym Theorem, states
that an absolutely continuous measure, such as M reg above, has a density function.
This means that there exists a function fM such that
Z
reg
M (A) =
fM (x) dx.
(2.3.7)
A

By combining these two results it is possible to represent the probability distribution


of a random variable as the sum of a density function and a singular measure.
2.3.4. Masses (also known as Dirac delta functions). In applications, many
random variables have a zero singular measure, which makes them very handy.
However, many useful random variables have nonzero singular measures. Tho understand nonzero singular measures, we discuss point masses here and their use in
representing probability distributions next.
Given a point a R, the unit mass (also known as Dirac delta function 6) located
at a, is denoted by a , and defined as the functional7 that operates on any given
5Skip

unless you have some measure theoretical foundations


spite of its name, the Dirac delta is not a function in the usual sense of the word. A proper
mathematical name for such objects as a is a Schawrtz distribtion or a Gelfand generalised
function. A nice introduction to the topic is given by Lieb & Loss [LL01]
7Functional is the standard name given to real or complex-valued function of functions.
6In

82

MTA (G5078) Autumn 2009 Lecture Notes

continuous function g as follows


ha | gi := g(a).

(2.3.8)

In simple words, a is the evaluation at the point a. The singular part of fX


stems from the jumps of FX . Although not a proper integral, the expression ha | gi
behaves like one in many respects, in particular we have
hca + db | gi = c ha | gi + d hb | gi ,
ha | cg + dhi = c ha | gi + d ha | hi ,

(2.3.9)

ha | gi sup g =: kgk .
R
(As an exercise you should check the above properties.)
For this reason, the duality pairing ha | gi is often denoted as an integral:
Z
a (x)g(x) dx := ha | gi = g(a).
(2.3.10)
R
We stress that the notation above is not an integral, but merely a notation that
looks like and often (but not always!) behaves like an integral. It is possible to
intergrate a against a continuous function g also on a subset A of R, by using As
indicator function. In particular we may define
(
Z


g(a), if a b,
a (x)g(x) dx := a | 1(,b] g =
(2.3.11)
0,
if a > b.
(,b]
While the first equality is a definition, the second equality, which follows from the
properties of a and 1(,b] , is left as an exercise to check. Again, we warn against
the abuse of the integral notation. For instance, you should convince yourself that
in general
Z
Z
a (x)g(x) dx 6=
(,a]

a (x)g(x) dx.

(2.3.12)

(,a)

(Thats because g(a) 6= 0 in general.) In particular, when Dirac delta functions


Ra
are involved we cannot use the usual notation for integrals on intervals: :=
R
R
= (,a] because the second equality may fail!
(,a)
2.3.5. Example (Dirac delta is the derivative of a step). The main purpose for
introducing impulses, is to be able to differentiate classically nondifferentiable functions. As an example, consider Heavisides step-function given by
(
1 if x > 0,
H(x) := 1R+ (x) =
(2.3.13)
0 if x 0.
We want to differentiate H. However, from basic calculus we know that H is not
continuous at 0, let alone differentiable. Nevertheless we may define a generalised derivative of a function f as being that distribution f 0 (i.e., bounded-linear functional
acting on differentiable functions with compact support g) such that
Z
0
hf | gi = f (x)g 0 (x) dx, g C1c (R).
(2.3.14)
R
where the integral on the right-hand side is a classical integral (no cheating here).
The reason for this definition is that it works for differentiable functions f and in
this case the generalised derivative f 0 coincides with the classical derivative of f
(vindicating thus the prime notation). Further, the expression on the right hand
side makes sense even if f is not differentiable in the classical sense.

MTA (G5078) Autumn 2009 2.3. Distribution and density

83

This makes it possible to define the derivative of a nondifferentiable function, such


as H, as a distribution (or generalised function, if you prefer).
Exercise. You should take several minutes now to check that H 0 = 0 .
2.3.6. Probability density function (pdf ). It can be shown that FX has a
countable number of discontinuities. Because FX is increasing these discontinuities
consist of jumps in the value of FX .
The probability density function of X, denoted by pdf X , or fX , or just f , is defined
as the (generalised) derivative of FX = cdf X . Generally it is not easy to give a
description of fX , but in many cases of practical importance an easy description is
available.
If we assume that FX is differentiable at all points where it is not discontinuous (such
functions are called piecewise differentiable), then results from Functional Analysis
imply that fX = pdf X is the sum of a positive function fXreg , called the regular part
of fX , and a singular part consisting of a countable sum of point masses (scaled
Dirac delta functions) of positive intensity, i.e.,
X
fXsng =
f j x j ,
(2.3.15)
jJ

where J is a countable index set (e.g., some subset of Z), {xj : j J} R and
f j > 0.8
In particular we have that
Z x
X
P {X x} = FX (x) =
fXreg (y) dy +
f j x j ,
(2.3.16)

xj x

for any x R. Using the abuse of notation, whereby we integrate the xj s, this
may also be written as
Z
FX (x) =
fX (y) dy.
(2.3.17)
(,x]

2.3.7. Remark (existence of density function not always possible). Not all random
variables X have a piecewise differentiable distribution function cdf X . An example
is given by the Inverted Devils Staircase, originating in Georg Cantors work, and
described in Example 2.3.9.
2.3.8. Example (distribution and density of a binomial). Consider the binomial
random variable given by the price of a stock
(
5 with probability 70%,
S=
(2.3.18)
2 with probability 30%.
The cumulative distribution of S is easily described as follows

for x < 2,
0
cdf S (x) = 0.3 for 2 x < 5,

1
for 5 x.
8Note

(2.3.19)

that in spite of its name the density may in general fail to be a classical function. Indeed,
it is a function if and only if its singular part is 0.

84

MTA (G5078) Autumn 2009 Lecture Notes

The law of S applied on a Borel set B R is also easily desribed as follows

0
if 2 and 5 6 B,

0.3 if 2 B and 5 6 B,
(2.3.20)
lawS B =

0.7 if 2 6 B and 5 B,

1
if 2 and 5 B.
The density of S is not a classical function, it is the sum of two impulses, namely
pdf S = 0.32 + 0.75 .

(2.3.21)

In this case the regular part of S is 0, and the expression above consists of the
singular part.
2.3.9. Example (inverted Devils Staircase). 9 Consider to be the space of infinite
sequences of independent fair coin tosses, where each is a sequence of symbols
in {H, T } (head or tail), i.e.,
= ((1), (2), . . . , (k), . . .) and (k) = H or T

k N,

(2.3.22)

with the probability measure P induced by the elementary binomial toss probability p on {H, T } where p {H} = 1/2 for heads and p {T } = 1/2 for tails. For
example, the probability of the event where the first toss is a head, the 5th a tail,
and the 6th or 7th toss is a head, can be calculated as follows
P { : (1) = H and (5) = T and ((6) = H or (T ) = H)}
(2.3.23)
113
3
= p {H} p {T } P {(6) = H or (T ) = H} =
= .
224
16
The space (, P ) constructed here is a common model probability space found in
many standard textbooks [Bil95, JP03, e.g.].
Consider playing the following game now. A chocolate bar C0 , of length 2 metres
is divided into three pieces, A1 , B1 and C1 , of equal length. You toss the coin then
bar B1 goes to the Bank, bar C1 is saved for the next round and bar A1 goes to you
if the outcome is T , otherwise (if the toss produces H) A1 goes to the bank. The
next step is played with C1 divided into three pieces of equal length, A2 , B2 , C2 ,
you toss the coin, B2 to the bank, C2 kept for next toss, and A2 to you if T or to
Bank if H. Consider now the random variable X that gives the length of chocolate
obtained after infinitely many tosses.
To model this game, introduce the basic random variable V : {H, T } R, where
V (H) := 0, V (T ) := 1, and let X : R be the random variable defined as follows

X
V ((k))
X() := 2
.
(2.3.24)
k
3
k=1
For example we have
2
X(T, T, T, T . . .) = 1, X(T, H, H, H . . .) = ,
3
1
2
X(H, T, T, T . . .) = , and X(H, T, T, T . . .) = .
3
9
Proposition. Let K N. For each integer of the form
j := 2

K1
X

k 3k + l, with 0 l 3 and (1 , . . . , K1 ) {0, 1}K1 ,

(2.3.25)

(2.3.26)

k=1
9You

may safely skip this example if you do not know Measure and Integration Theory.

MTA (G5078) Autumn 2009 2.3. Distribution and density

85

we have that

P

j
X K
3


=

K
X

!
k 2k + dl/2e

k=1

1
.
2K

(2.3.27)

Proof The complete proof of this result is not too hard and left as an exercise.
As an example take K = 3, then we may produce the following schedule
(0, 0)
(0, 1)
(1, 0)
(1, 1)
(1 , 2 )
l 0 1 2 3 0 1 2 3 0 1 2 3 0 1 2 3
j 0 1 2 3 6 7 8 9 18 19 20 21 24 25 26 27
P {X j/33 } 0 18 18 14 14 83 38 12 12 58 58 34 34 87 78 1
(2.3.28)

It follows that the cumulative distribution function, FX , of X is the so-called Devils
Staircase, which is a well-known fractal structure. The function FX is continuous (it
has no jumps), it is differentiable almost everywhere, with derivative 0, but it has
infinitely many points where it is differentiable only on one side. So its generalised
derivative is not a function and neither a countable sum of mass points, but a diffuse
singular measure concentrated on a set, denoted [0, 1), and known as the Cantor
set.
An approximation of the Devils Staircase can be plotted as follows
1
0.9
0.8
0.7
0.6
0.5
0.4
0.3
0.2
0.1
0

0.1

0.2

0.3

0.4

0.5

0.6

0.7

0.8

0.9

The Cantor set can be shown to consist of all those real numbers between 0 and 1
with a base-3 expansion that has no 1s (for those numbers that admit two different
expansions such as 1/3 = 0.1 = 0.0222 . . . , it is enough to have one expansion
with no 1s). A diagonal argument shows then that the Cantor set is uncountable.
Furthermore, it is possible to show that is closed and has Lebesgue measure 0.
Thus the distribution measure PX of X, defined by the elementary blocks
PX (a, b] := P {a < X b} = FX (b) FX (a),

(2.3.29)

is a singular measure with respect to the Lebesgue measure and the RadonNikodym
Theorem does not apply.

86

MTA (G5078) Autumn 2009 Lecture Notes

Using the terminology introduced in this section, if fX denotes the generalised density function of X, then the decomposition fX = fXreg + fXsng satisfies
Z
reg
sng
(x) dFX (x),
(2.3.30)
fX = 0 and hfX | i =

where the last integral is the RiemannStieltjes integral. The fact that fXreg = 0
means that the only contribution to the generalised probability density function in
this case comes from its singular part. For more details, you could see the book by
Stroock [Str99].
2.3.10. Expectation rule. Laws, c.d.f. and p.d.f. are useful tools. One the main
purposes for them is to get rid of . This means, for example, that by using the
law of a random variable on , integration on can be replaced by integration on
R (this is some kind of change-of-variable formula). This is an advantage, especially
in practical applications of probability, because generally the space is very hard
to model and manipulate, whereas R is (as you know) quite an easy place to work
in.
Proposition (expectation rule). If X is a real-valued random variable, with PX =
lawX and FX = cdf X . Let g be some function whose domain includes the set X().
Then g(X) is a random variable too and
Z
Z
E[g(X)] = g(x)PX ( dx) = g(x) dFX (x),
(2.3.31)
R
R
where the last integral is understood as a Riemann-Stieltjes integral.
2.3.11. Densitymass only distributions. Most of what we describe next works
for general random variables. Nonetheless, unless otherwise stated, we will consider
from now on only random variables which have a generalised probability distribution function consisting of the sum of a density function and a (possibly infinite)
countable sum of modulated Dirac masses. That is, given a random variable X we
assume that
X
fX (= pdf X ) = f reg +
f i xi
(2.3.32)
iJ

for some function f reg and a point mass distribution fi R+ for all i J, a finite
or countably infinite set. In particular, if is a smooth function, then we have
Z
Z
X
(x) dFX (x) = (x)f reg (x) dx +
f i (xi ).
(2.3.33)
iJ

For example, in this case the expectation rule becomes


Z
Z
X
E[g(X)] = g(x) dFX (x) = g(x)f reg (x) dx +
f i g(xi ).
R
R
iJ

(2.3.34)

2.4. Moments and characteristic functions


2.4.1. P
Moments. Let X be a random variable with F := cdf X and pdf X =: f =
f reg + iJ fi xi . Its expectation (or average, or mean) can be calculated using the
expectation rule (2.2.3) with g(x) = x:
 Z
 Z
Z
X
E X :=
X()P ( d) = x dF (x) = xf reg (x) dx +
f i xi .
(2.4.1)

R
R
iJ

MTA (G5078) Autumn 2009 2.4. Moments and characteristic functions


87
Similarly, if we denote by := E X, the variance of X can be calculated as follows
Z


2
var X = E (X ) = (x )2 dF (x).
(2.4.2)
R
The mean and the variance are often called first and second moment respectively.
The variance is called also second central moment.
More generally the n-th moment of X about some value a R is defined as
Z
0
n
(x a)n dF (x).
(2.4.3)
n (a) := E[(X a) ] =

provided the integral exists. If a = , the mean of X, then the moments are called
central moments and are conventionally written without the prime. For any other
a, the moments are non-central moments. The most common alternative is a = 0.
2.4.2. Characteristic function. Let X be a random variable and let F := cdf X
and (possibly) f = pdf X . The characteristic function of X, denoted by chf X , X , or
simply , whenever X is clear from the context, is the Fourier transform of dF ,
namely,
Z
Z
i tx
(t) := E[exp(i tX)] = e dF (x) = (possibly) ei tx f (x) dx, for t R. (2.4.4)
R
R

2
Here i is the unit imaginary number such that i = 1 (or i = 1 if you prefer).
The characteristic funtion and the n-th moment of f (x) about the origin are related
by the relation,
0n (0) = in (n) (0).
(2.4.5)
In the sequel we shall give some examples of characteristic functions and illustrate
their use to find moments. For a full development of the subject and more examples,
however, you should consult a standard textbook [JP03, Ch.13].
2.4.3. Exercise. Verify the characteristicmoment relation (2.4.5).
Hint. By the Lebesgue Dominated Convergence Theorem, you may take the
under the integral.

d
dt

sign

2.4.4. Example (characteristic function of a mass). Let c R be fixed and consider


a random variable : c R such that
10

c () = c for P -almost all ,

(2.4.6)

c () = c with chance 100%.

(2.4.7)

Hence we know that

Call the distribution Fc := cdf c , then we have


(
0 if x < c,
Fc (x) =
1 if x c,

(2.4.8)

and the generalised derivative of Fc is a Dirac mass located at c, c . Therefore the


characteristic function of c , denoted c , is given by
c (t) = hc | exp(i t)i = exp(i ct) = cos(ct) + i sin(ct).
In particular if c = 0, then the charactersic function is the constant 1.
10If

you dont know what P -almost all means just think all.

(2.4.9)

88

MTA (G5078) Autumn 2009 Lecture Notes

2.4.5. Example (characterstic function of a binomial). Consider the binomial random variable S : R such that
(
a with chance p,
S=
(2.4.10)
b with chance 1 p,
for some p (0, 1).
As seen in Example 2.3.8 we have
pdf S = pa + (1 p)b .

(2.4.11)

Thus Ss characteristic function is found to be


(t) := chf S (t) = p exp(i at) + (1 p) exp(i bt)
= (p cos(at) + (1 p) cos(bt)) + i(p sin(at) + (1 p) sin(bt)) .
(2.4.12)
For instance, if we had a = 0 and b = 1 we obtain
(t) = p + (1 p) cos(bt) + i(1 p) sin(bt).
Calculating the first derivative we have


d
(t)
= [p i a exp(i at) + (1 p) i b exp(i bt)]t=0 = i(ap + b(1 p))
dt
t=0

(2.4.13)

(2.4.14)

so E S = ap + b(1 p), as expected. The second derivative yields the second moment



d d
0
2 (0) =
(t)
= pa2 + (1 p)b2 ,
(2.4.15)
dt dt
t=0
and thus, with = E S, the variance can be computed as follows:
 
var S = E S 2 (E S)2 = 00 (0) 2

= pa2 + (1 p)b2 (ap + b(1 p))2 = p(1 p) a2 + b2 2p(1 p)ab
= p(1 p)(b a)2 .
(2.4.16)
For instance, if we had a = 0 and b = 1 we would have the standard deviation
p
(2.4.17)
S := p(1 p).
2.4.6. Characterisation Theorem. The characteristic functions of a distribution are always defined (because probability measures sum to 1 and the RiemannLebesgue Lemma guarantees convergence of the integral in (2.4.4)) and uniquely determine the distribution, in the following sense, suppose X1 and X2 are two random
variables, and let i and Fi indicate their characteristic and cumulative distribution
functions, respectively, for i = 1, 2. Then, 1 = 2 if and only if F1 = F2 , i.e., X1
and X2 are identically distributed. (Note that this does not mean that X1 and X2
are equal, nor equal P -almost everywhere.)
For a proof of uniqueness we refer to Jacod & Protter [JP03, Ch.14].
2.4.7. Moment generating function and Laplace transform. The characteristic function of a random variable is the Fourier transform of its probability
distribution measure. Similarly, one may consider the Laplace transform,
Z
Z
rx
L(r) = e
dF (x) = (possibly) erx f (x) dx,
(2.4.18)
R
R
provided the integral makes sense, which is the case when the random variable X
(for which F = cdf X and f = pdf X ) is positive.

MTA (G5078) Autumn 2009 2.6. Moments and characteristic functions


89
Related to the Laplace transfrom is Xs moment generating function
Z
Z
qx
M (q) := L(q) = e dF (x) = (possibly) eqx f (x) dx.
(2.4.19)
R
R
Both the Laplace transform and the moment generating function are real valued
functions (whereas the characterstic function is generally complex valued). The moment generating function is also useful in determining the moments of a distribution
via the identity
 n

d
M (q)
= 0n (0),
(2.4.20)
n
dq
q=0
which is left as a useful exercise to verify.
2.4.8. Theorem (Chebyshevs inequality). Let X be a random variable on the
probability space (, F , P ) with = E X and 2 = var X, then
P {|X | t} t2

t > 0.

(2.4.21)

2.5. Normal distribution


If X is a normally distributed random variable with mean and variance 2 , its
density function and characteristic function are given by


(x )2
1
exp
,
(2.5.1)
f (x) =
2 2
2
and


2 t2
(t) = exp i t
,
(2.5.2)
2
respectively. Normally distributed random variables are called also Gaussian and it
is customary to write


X N , 2
or X N , 2 .
(2.5.3)
The higher central moments of a normal random variable are
(
0
if n is odd,
n = E[(x )n ] =
n
(n 1)(n 3) 3
if n is even.

(2.5.4)

2.5.1. Definition (standard normal deviates). If X is a normally distributed random variable with mean = 0 and variance 2 = 1, it is said to be a standard normal
deviate. The density and distribution functions of a standard normal deviate are
often denoted by n(x) and N (x).
2.6. Limit Theorems
2.6.1. Weak law of large numbers. Let (Xi )iN be a sequence of independent
random variables11 on a probability space (, F , P ), which are identically distributed, i.e., there is one distribution F such that cdf Xi = F for all i N. Denote
by the, then for any constant > 0,

)
( n

1 X


Xi < = 1.
lim P
n

n
i=1

11Independence

is the single most important probability concept, but it is not covered in this
appendix. For a proper discussion you should consult one of the recommended texts.

90

MTA (G5078) Autumn 2009 Lecture Notes

This relation is also written as


#
n
1X
lim
Xi = ,
n n
i=1
P

"

where the symbol limP means convergence in measure with respect to P .


If the Xi are not identically distributed, but each has finite expectation i and a
finite variance i2 , then the following similar, albeit more general, relation holds:
" n
#
n
P
1X
1X
lim
Xi
i = 0.
(2.6.1)
n n
n
i=1
i=1
These results go by the name of weak law of large numbers.
2.7. Central limit-theorem
If the Xi are identically distributed with common finite expectation and finite
variance 2 , then the Central Limit-Theorem applies. This theorem indicates that
the approach to the limits above is asymptotically normal; that is, the sample mean
of the xi is approximately normal with mean and finite variance 2 /n, or
(
!
)

n
n 1X
lim P a <
(2.7.1)
xi < b = N (b) N (a)
n
n i=1
2.8. Examples of random variables
2.8.1. Bivariate Normal Variables. Let x1 andx2 be bivariate normal random
variables with means i variance i2 and covariance 12 . The weighted sum w1 x1 +
w2 x2 is normal with mean and variance
= w1 1 + w2 2 , 2 = w12 i2 + 2w1 w2 12 + w22 22 ,
For such x1 andx2 and for a differentiable function h(x), cov[x1 , h(x2 )] = E[h0 (x2 )]12
This property can be proved as follows. If x1 andx2 are bivariate normals, then from
our understanding of regression relationships we may write
x1 = a + bx2 + e,
where b = 12 /22 and e is independent of x2 . Therefore,
cov[x1 , h(x2 )] = cov[a + bx2 + e, h(x2 )]
= b cov[x2 , h(x2 )]
= bE[(x2 2 )h(x2 )]
Z

(2.8.1)

(x2 2 )h(x2 )f (x2 )dx2 ,

=b

where f (x2 ) is the univariate normal density. Now


df (x2 )
x2 2
=
f (x2 ).
dx2
22

(2.8.2)

So (2.8.1) may be rewritten as


cov[x1 , h(x2 )] =

b22

h(x2 ) df (x2 )

b22 [h(x2 )f (x2 )]

b22

(2.8.3)

h (x2 )f (x2 ) dx2

MTA (G5078) Autumn 2009 2.8. Examples of random variables

91

upon integrating by parts. Then if h(x2 ) = o(exp(22 x2 )), the first term vanishes at
both limits, and the remaining term is just E[h0 (x2 )]12 .
2.8.2. Lognormal variables. If x is normally distributed, then z = ex is said to
be lognormal. The lognormal density function is

(ln z )2
f (z) = ( 2z)1 exp(
)
2 2

(2.8.4)



n2 2
n = exp n +
2


2

z = exp +
2


var(z) = exp 2 + 2 exp 2 1

(2.8.5)

The moments are

The truncated mean


Z
E[z|z > a] =
a



ln a
2
+ .
zf (z) dz = exp( + )N
2

(2.8.6)

2.8.3. Exercise. Introduce change of variables z = ex and dz = zdx to calculate


E[z|z > a].
2.8.4. Fair Games. If the conditional mean of one random variable does not depend on the realization of another, then the first random variable is said to be
conditionally independent of the second.
That is, x is conditionally independent of y if E[x|y] = E[x] for all realizations y.
If x has zero mean, then x is a noise or a fair game with respect to y : E[x|y] = 0.
Conditional independence is intermediate between independence and zero correlation:
1. x and y are uncorrelated if cov(x, y) = 0.
2. They are independent if cov(f (x), g(y)) = 0 for all pairs f and g.
3. x is conditionally independent of y if and only if cov(x, g(y)) = 0 for all functions
g(y).
2.8.5. Theorem (Jensens inequality). If X is a random variable with positive
dispersion and density function f and G : R D R is a concave function,
G00 (x) < 0 for all x D, then
E[G(X)] < G(E[X]).
:= E[X] R, Taylors formula for G about X
yields
Proof Let X
+ (x X)G
0 (X)
+ 1 (x X)
2 G00 (x )
G(x) = G(X)
2
and x.
where x belongs to the interval with endpoint X

x D,

(2.8.7)

92

MTA (G5078) Autumn 2009 Lecture Notes

Then
Z

E[G(X)] =

G(x)f (x) dx
Z
Z
0

(x) dx
= G(X)
f (x)dx + G (X)
(x X)f

Z
1
2 f (x)dx
+
G00 (x )(x X)
2
Z
1

2 f (x)dx
= G(X) +
G00 (x )(x X)
2

< G(X),

which is what we are required to show.

2.9. Stochastic Processes


2.9.1. Definition. A stochastic process is a sequence of random variables X0 , . . . , XN ,
with realizations x0 , . . . , xN . The probability density function of Xn as measured
after observing the i-th outcome is given by fni (Xn ; x0 , . . . , xi ).
One example of a stochastic process is the random walk with drift
Xn = + Xn1 + n .

(2.9.1)

Here the n are independent and identically distributed. The parameter , which
is the expected change per period, is called the drift. Another stochastic process is
the autoregressive process
Xn = (1 a) + aXn1 + n

(2.9.2)

where again the error terms are independent and identically distributed.
2.9.2. Definition (Markov process). A Markov process is a stochastic process for
which everything that we know about its future is summarized by its current state.
In terms of the distributions of Xn we have
fni (Xn ; x0 , . . . , xi ) = fni (Xn ; xi )

(2.9.3)

2.9.3. Definition (martingale). A martingale is a stochastic process {Xn } with


the properties:
E[|Xn |] <
(2.9.4)
and
E[Xn+1 |x0 , . . . , xn ] = xn .
(2.9.5)
More generally, the stochastic process {Xn } is a martingale with respect to the
information in the in the stochastic process {Yn } if and only if
E[|Yn |] < ,

(2.9.6)

E[Xn+1 |Y0 , . . . , Yn ] = xn .

(2.9.7)

and
The martingales property can also be stated as Xn+1 Xn is a fair game with respect
to {Yi }.
2.9.4. Proposition. A time series of updated conditional expectations is always
a martingale in this generalized sense (provided that the expectations are finite).
That is, let X be a random variable, let {Yi } be any stochastic process. Then
xn = E[X|y0 , . . . , yn ] are the realizations of a martingale.

MTA (G5078) Autumn 2009 2.10. Stochastic Processes

93

Proof Use law of iterated expectations


E[Xn+1

|y0 , . . . , yn ] = E[E[Xn+1 |Y0 , . . . , Yn+1 ]|y0 , . . . , yn ]

(2.9.8)

But Xn+1 is a function of y0 , . . . , yn+1 so the inner expectation is just the realization
xn+1 which is also, by definition
E[X|y0 , . . . , yn+1 ],
therefore
E[E[Xn+1

|Y0 , . . . , Yn+1 ]|y0 , . . . , yn ] = E[E[X

|y0 , . . . , yn+1 ]|y0 , . . . , yn ]

= E[X|y0 , . . . , yn ] = xn
This is the martingale property.

(2.9.9)

(2.9.10)


2.9.5. Remark. One important property of martingales is that there is no expected


change in the level over any interval. Over one interval, this is follows from
E[Xn+1

|y0 , . . . , yn ] = xn

(2.9.11)

Over many intervals, we have


E[Xn+1 ] = E[E[Xn+1

|Y0 , . . . , Yn ]] = E[Xn ]

(2.9.12)

Then by induction, we have E[Xn+m ] = E[Xn ] for all m > 0.


2.10. Regression and analysis of variance
Statistics can be defined as the art of fitting by chance. More precisely, given some
collected data, the job of a statistician is to find underlying probability distributions.
Here we describe a common and widely used in finance statistical technique which
is known as the least-square fitting.
2.10.1. Single Variable. Many scientific experiments are concerned with examining the way things behave under different conditions. An experiment must have
a clearly measurable outcome condition: e.g., in a medical trial the reduction in
blood pressure vs. the strength of drug, in a manufacturing process the purity of
the product vs. the temperature in the combustion chamber, in house prices the
percentage changes vs. the interest rate changes.
The simplest mathematical model where the out come depends on a single variable
is
y = a + bx + e
(2.10.1)
Here a and b are constants and e is a noise, mathematically thought of as a random
variable.
2.10.2. Fitting a model. Set up criteria to measure the extent to which the model
fits the observed data {(xi , yi ) : i = 1, . . . , N }.
The most popular way is to consider the squares sum function
S(a, b) := (y1 a bx1 )2 + (y2 a bx2 )2 + . . . + (yn a bxn )2

(2.10.2)

and minimize S(a, b) with respect to a and b. Using the usual rules of finding
minimum, we get
s(x, y)
a
= y b
x and b =
,
(2.10.3)
S(x, x)
where
n
n
X
1X
1X
S(x, y) =
(
x xi )(
y yi ), x =
xi and y =
yi
(2.10.4)
n
n
i
i=1
i=1

94

MTA (G5078) Autumn 2009 Lecture Notes

The random variables are assumed to satisfy var(ei ) = 2 , after substituting the
expressions of a
and b into (2.10.2), we obtain
S(a, b) = S(y, y)

S(x, y)2
S(x, x)

(2.10.5)

It can be shown that an unbiased estimator of 2 is


2 = S(a, b)/(n 2).

(2.10.6)

2.10.3. Remark (extending to multiple variables). In this case we have


y = a + bx1 + cx2 + e

(2.10.7)

where e is the error term, assumed to be N (0, 2 ) distributed. Again, we fix the
parameters a, b and c by minimising the squares sum function
S(a, b) = (y1 a bx11 cx12 )2 + (y2 a bx21 cx22 )2 + . . . + (yn a bxn1 cxn2 )2
(2.10.8)

APPENDIX C

Common indices in Economics and the Housing Market


This chapter is a collection of sources, that may be useful for essay writing.
According to the dictionary, index is a system of numbers used for comparing values
of things which vary against each other or against a fixed standard.
3.1. FTSE 100 Index (also known as Footsie)
A number which expresses the value of the share prices of the one hundred most
important British companies, which is published by the Financial Times (a British
newspaper for people interested in business and finance):
The FTSE 100 Index is a capitalization-weighted index of the 100 most highly
capitalized companies traded on the London Stock Exchange. The equities use an
investibility weighting in the index calculation. The index was developed with a
base level of 1000 as of January 3, 1984.
Remark: investibilty: the freely available for trade shares in the designated market.
For example, if X is a FTSE 100 constituent, it has 50% of its shares floating in
London, 50% floating in New York, then only half of X companys capital value
should be use when calculating its weighting in FTSE.
3.1.1. Problem. Explain when FTSE 100 constituents change every quarterly, how
do you recalculate the index.
3.2. Halifax House Price Index
3.2.1. Background. The Halifax House Price Index is the UKs longest running
monthly house price series covering the whole country from January 1983. The UK
Index is derived from the mortgage data of the countrys largest mortgage lender,
which provides a robust and representative sample of the entire UK market.
There are a number of national indices covering different categories of houses (all,
new and existing) and buyers (all, first-time buyers and home-movers). These
indices are also adjusted to allow for seasonal variations. The most commonly used
and quoted Halifax Index is the UK seasonally adjusted index covering all houses
and all buyers. Regional indices for the 12 standard planning regions of the UK are
produced on a quarterly basis.
3.2.2. Methodology. The indices calculated are standardised and represent the
price of a typically transacted house. The need for standardisation arises because
no two houses are identical and may differ according to a variety of characteristics
relating to the physical attributes of the houses themselves or to their locations.
In summary, prices are disaggregated into their constituent parts using a commonly
used statistical technique called multivariate regression analysis. This allows values
to be attributed to the various qualitative characteristics (type of property, region,
etc.) and quantitative characteristics (age of property, number of habitable rooms,
garages, bathrooms, etc.) of a property.
As a result, the technique allows us to track the value of a typical house over
time on a like-for-like basis (i.e. with the same characteristics). This prevents the

96

MTA (G5078) Autumn 2009 Lecture Notes

possibility of short-term changes in the set of properties sold from month to month
(for example, shifts in the regional complexion of the market or a change towards
more large properties being sold) giving a misleading impression of the change in
the price of a typical house.
Analyses of house prices are based on simple arithmetic average prices.
3.2.3. Data. The Halifax House Price Indices are derived from information on the
following house characteristics:
? Purchase price.
? Location (region).
? Type of property: house, sub-classified according to whether detached, semidetached or terraced, bungalow, flat.
? Age of the property.
? Tenure: freehold, leasehold, feudal.
? Number of rooms: habitable rooms, bedrooms, living-rooms, bathrooms.
? Number of separate toilets.
? Central heating: none, full, partial.
? Number of garages and garage spaces.
? Garden.
? Land area if greater than one acre.
? Road charge liability.
Although one hundred per cent coverage of all house purchase transactions financed
by the Halifax is obtained, those transactions that do not constitute a fully consistent
body of data for the purpose of house price analysis are excluded from the Indices.
These exclusions primarily cover property sales that are not for private occupation
and those that are likely to have been sold at prices which may not represent free
or normal market prices, for example, most council house sales, sales to sitting
tenants, etc. Only mortgages to finance house purchase are included; remortgages
and further advances are excluded.
The data refers to mortgage transactions at the time they are approved rather
than when they are completed. Whilst this may cover some cases which may never
proceed to completion, it has the important advantage that the price information
is more up-to-date as an indicator of price movements and is on a more consistent
time-base than completions data (such as the ODPM Index) given the variable time
lags between approval and completion.
The monthly indices cover transactions during the full calendar month and the
regional quarterly indices cover transactions over the entire quarter. Properties over
1 million have been included since December 2002 to reflect the increasing number
of this hitherto tiny market segment.
3.2.4. Seasonality. House prices are seasonal with prices varying during the course
of the year irrespective of the underlying trend in price movements. For example,
prices tend to be higher in the spring and summer months when more people are
looking to buy. We therefore produce seasonally adjusted series to remove this
effect and to allow us to concentrate on the underlying trend in house prices. These
seasonal factors are updated monthly.
The Halifax House Price Index is prepared from information that we believe is
collated with care, but we do not make any statement as to its accuracy or completeness. We reserve the right to vary our methodology and to edit or discontinue
the indices at any time for regulatory or other reasons. Persons seeking to place

MTA (G5078) Autumn 2009 3.3. Halifax House Price Index

97

reliance on the indices for their own or third party commercial purposes do so at
their own risk.
Full Technical Details are available free on request - contact the HBOS Economics
Help-line on 01422 333558.
3.3. Nationwide House price Index
There are several methods that could be used to calculate the trend in house prices,
ranging from a simple average of purchase price to a statistical method of averaging.
Then there is the matter of making sure that the different mixture of properties
sold in each month does not give a false impression of the actual change in house
prices. The next few sections explain the way we do this as well as providing some
background to the Nationwide house price series and the current methodology that
we employ to calculate average house prices.
3.3.1. Background to Nationwide House Price Information. Nationwide
Building Society has a long history of recording and analysing house price data and
has published average house price information since 1952. The following provides a
short chronology of publish series and developments in Nationwides methodology
of calculating average house prices:
1952: annual publication of house price data;
1974: quarterly data is published for the first time;
1989: development of new house price methodology (a statistical regression
technique was introduced under guidance of Fleming and Nellis Loughborough University and Cranfield Institute of Technology);
1993: the house price system was further improved following publication of the
Census 1991 data, and frequency for UK series increased to monthly.
The monthly figure measures the mix adjusted average house price for all houses in
the UK. Every quarter the Nationwide also publishes a more detailed breakdown of
house prices. These include both UK and 13 regional estimates for:
5 types of house: detached, semi-detached, terraced, flats and bungalows;
2 types of buyer: first time buyer and former owner occupiers;
3 property ages: new, modern and older.
This makes a total of 154 separate series, all of which are published in the Nationwides Quarterly Review.
3.3.2. Data source. All house price information is derived using Nationwide mortgage data. This data is extracted monthly for mortgages that are at the approvals
stage and after the corresponding building survey has been completed.
3.3.3. Data processing (also known as cleaning). Nationwide house price series utilise only owner occupied property information. In addition, properties that
are not typical and may distort the series are also removed from the data set. Therefore, the following criteria is used to select which properties to include:
? house purchasesremortgages and further advances are excluded;
? owner occupied properties;
? purchase prices below 1 million;
? properties sold at true market pricesright to buy sales at discounted price are
excluded;
? floor size has to be within specified limits for a give type of property, e.g., a
detached house has to have at least 400 square foot [sic] floor area.

98

MTA (G5078) Autumn 2009 Lecture Notes

3.3.4. Sample Size. The number of cases that are used to calculate the average
price for a given month will depend on the volume of monthly mortgage activity and
out of these the cases that meet the criteria in the cleaning process. The monthly
sample size will therefore vary from month to month. Nationwide has sufficient
sample size to produce a representative house price series. N.B. Net lending figures
quoted at our half yearly and annual results are not a guide to our sample size.
Sample size is based on the number of new loans we write i.e. the amount of gross
lending for house purchase(remortgage cases are excluded).
The Nationwide Building Society is the 5th largest mortgage lender in the UK. Our
share of the gross house purchase market has averaged 9% over the last 3 years
(reaching 11% during our financial year 2000/01. This allows us to be confident
that the series based on Nationwide mortgage data is representative of the whole
house market.
The quarterly UK series for all houses uses 3 months of data and hence a much
larger sample than at the month. The samples sizes for the other quarterly series
will depend on what it is they are measuring, for example the series for first time
buyers only considers properties being brought by first time buyers and hence this
will have a smaller sample size than that used for the whole of the UK. It is for this
reason that detailed breakdown of house prices are produced quarterly.
3.3.5. Mix Adjustment Process. The purpose of mix adjustment is to simply
isolate pure prices changes. The simple example below illustrates how the changes in
the mixture of properties sold each month could give a misleading picture of what is
actually happening to house prices. The set of properties sold from month to month
will vary by location and design etc. and some adjustment is necessary to make
sure all of these do not give a false impression of the actual changes to house prices.
A mix-adjusted or standardised index is not affected by such changes because the
relative weight given to each characteristic of a property in the mix (or basket,
to use an analogy with retail prices) is fixed from one period to the next.

3.3.6. A simple example: benefits of mix adjustment. Suppose that the


price of both detached houses and flats increased at the same rate for five periods,
with flats being cheaper.
Further suppose that the proportion of flats and detached sold in each period varied
considerably, as the table shows.

MTA (G5078) Autumn 2009 3.4. Nationwide House price Index

99

The simple average of both kinds of properties will be influenced by the proportion
of each property sold. In periods 3 and 4 the simple average shows a decrease,
whereas the actual prices of both increased!
The mix adjusted average uses a consistent measure of the proportion of each type
of property and is able to better reflect the true change in prices.
Time Period P1 P2 P3 P4 P5
% Flats
50 30 70 30 70
% Detached
50 70 30 70 30
The mix-adjusted price represents the price for an average or typical house. This
should not be confused with the average price of all houses. The latter is usually
higher because even though there are fewer more expensive houses sold, their price
is such that they bias the simple average to be greater than the price of the typical
house.
3.4. Calculating the price of a typical house
The price of a property will depend on the characteristics of the property. These
characteristics could include physical properties of the house, like its design, but
other aspects such as the type of neighbourhood the house is located in will also
contribute to the price someone is willing to pay. Using mortgage data, the Nationwide house price system can relate all the observed combinations of these factors and
relate them to the price of which the house was sold for. From this, the model can
estimate how much on average a house would cost given a set values for these characteristics, in particular a set of characteristics that describes the typical house.
This typical house does not physically exist, it is an average house across all the
characteristics that the model uses. This method is repeated on data sets at different points in time and changes in the price of this typical house reflect only the
price changes over the same time periods, and not the mixture of properties sold in
the current or pervious periods.
3.4.1. Factors that affect the price of a house. The following are the items
that are used to describe the characteristics of a property. There is no set order that
these contribute most to the price of the house, although UK location, the type of
neighbourhood and house size are consistently the three most important followed
by the design of the house.
Geographic location: Type of neighbourhood. The Nationwide index uses an
established demographic system that classifies areas in the UK into 54 categories
based on the type of people that live there, two examples include retirement
and council areas.
floor size: Property design (detached house, semi-detached house, terraced house,
bungalow, flat, etc.)
tenure: freehold/leasehold/feudal, except for flats, which are nearly all leasehold;
number of bathrooms: 1 or more than 1;
type of central heating: full, part or none;
type of garage: single garage, double garage or none;
number of bedrooms: 1,2,3,4 or above 4;
new/old: property is either new or not.

100

MTA (G5078) Autumn 2009 Lecture Notes

3.4.2. Seasonal Adjustment. House prices are slightly seasonal - that is, prices
are higher at certain times of year irrespective of the overall trend. This tends to
be in spring and summer, when more buyers are in the market and hence sellers
do not need to discount prices so heavily, in order to achieve a sale. The effect on
prices over the year is of the order of 2%; however this is much smaller than the
change in volume of property transactions. The seasonal effect is estimated twice a
year using established statistical methods.
For the monthly house price index where changes can be as little as 0.1%, seasonal
factors are important. The Nationwide therefore produce a seasonally adjusted series
for UK house prices which seeks to remove this effect so that the overall trend in
prices is more readily apparent.
Seasonal adjustment shows that June is generally the strongest month for house
prices (raw prices are 1.3% above their SA level) and January is the weakest (raw
prices are 1.9% below their SA level).
3.5. Pooled Property Fund Indices
Details of this index can be found on http://www.ipdindex.co.uk
Published by HSBC and The Association of Property Unit Trusts
The Pooled Property Fund Indices are jointly published by the Association and
HSBC. The data is compiled and calculated by IPD.
The index is designed to bring greater transparency to the PUT and managed pension fund industry; provide fuller and more timely information to both existing and
potential investors (to a target of 10 working days); allow investors to compare
performance of individual vehicles with the rest of the Pooled Property sector.
3.5.1. Important Information.
1. Following a consultation exercise the decision has been made to change the calculation basis of the Pooled Property Fund Indices to a net asset value basis,
from an offer price basis.
2. The new Indices no longer contains an All Property Unit Trusts Index - this
has been replaced by an All Balanced Funds Index.
It splits the 29 participating Funds into three categories:
balanced funds: holding a wide mix of property assets,
specialist vehicles: focusing on a particular type of property,
managed property funds: managed mainly by insurance companies.
Their performance is measured by NAV to NAV unit returns. These are compiled
from quarterly unit price and distribution records, supplied to IPD by individual
fund managers.
One of the unique advantages of the new service is that it enables investors to judge
the unit price performance of a fund in the context of gearing percentages and
independently verified IPD information relating to portfolio asset allocation.
3.5.2. Problem. Discuss the differences in the three property related indices.

Bibliography
[Bil95]

Patrick Billingsley. Probability and measure. Wiley Series in Probability and Mathematical Statistics. John Wiley & Sons Inc., New York, third edition, 1995. ISBN 0-471-007102. xiv+593 pp. A Wiley-Interscience Publication.
[CZ03] Marek Capi
nski and Tomasz Zastawniak. Mathematics for finance. Springer Undergraduate Mathematics Series. Springer-Verlag London Ltd., London, 2003. ISBN 1-85233330-8. x+310 pp. An introduction to financial engineering.
[DFF56] G. B. Dantzig, L. R. Ford, Jr., and D. R. Fulkerson. A primal-dual algorithm for linear
programs. In Linear inequalities and related systems, Annals of Mathematics Studies, no.
38, pages 171181. Princeton University Press, Princeton, N. J., 1956.
[Eat]
John W. Eaton. GNU Octave. URL http://www.gnu.org/software/octave/.
[Fle77] Wendell Fleming. Functions of several variables. Springer-Verlag, New York, second edition, 1977. xi+411 pp. Undergraduate Texts in Mathematics.
[JP03]
Jean Jacod and Philip Protter. Probability essentials. Universitext. Springer-Verlag,
Berlin, second edition, 2003. ISBN 3-540-43871-8. x+254 pp.
[LL01] Elliott H. Lieb and Michael Loss. Analysis, volume 14 of Graduate Studies in Mathematics. American Mathematical Society, Providence, RI, second edition, 2001. ISBN
0-8218-2783-9. xxii+346 pp.
[MCS86] J. J. Mc Cutcheon and W. F. Scott. An Introduction to the Mathematics of Finance.
ButterworthHeinemann, 1986. ISBN 0-7506-0092-6.
[NW99] Jorge Nocedal and Stephen J. Wright. Numerical optimization. Springer Series in Operations Research. Springer-Verlag, New York, 1999. ISBN 0-387-98793-2. xxii+636 pp.
URL http://dx.doi.org/10.1007/b98874.
[Str99] Daniel W. Stroock. A concise introduction to the theory of integration. Birkhauser Boston
Inc., Boston, MA, third edition, 1999. ISBN 0-8176-4073-8. xiv+253 pp.

Index

(total) return on the portfolio, 48


annuity schedule, 14
annuity-due, 12
arbitrage, 48
asset, 45, 53
return, 46
risk-free, 45
risky, 45
asset price, 46
asset types, 45
binomial random variable, 45
bond, 29, 45
budget constraint, 53
capital redemption, 29
cash flow model, 15
commitment vector, 54
continuous time annuity, 13
currency, 40
deferred annuity-certain, 13
deferred annuity-due, 13
discrete
time, 46
economy, 53
equity, 45
fixed interest, 29
fixed interest security, 15
float
bonds, 29
hedging, 66
inflation, 40
annual, 40
initial investment, 29
instant, 46
interest payment, 29
linear programming, 94
loan/mortgage schedule, 14
long on, 47
long position, 47
market matrix, 47
market vector, 47

NAP, 48
net cash flow, 15
No-Arbitrage Principle, 48
one-step binomial, 46
perpetuity, 34
portfolio, 47, 53, 54
portfolio weights, 54
price index, 40
purchasing power, 40
random number, 45, 100
random variable, 45, 100
random vector, 53
realized return, 53
redeem
a bond, 45
redemption
bond, 45
return, 46
cash, 40
money, 40
on an asset, 46
real, 40
risk, 51
risk-free
asset, 45
risky
asset, 45
sample space, 53
security, 45
short on, 47
short position, 47
state, 53
stock, 45
time
discrete, 46

Vous aimerez peut-être aussi